Π€ΠΎΡ€ΠΌΡƒΠ»Π° сСчСния ΠΏΡ€ΠΎΠ²ΠΎΠ΄Π°: Как ΠΎΠΏΡ€Π΅Π΄Π΅Π»ΠΈΡ‚ΡŒ ΠΏΠ»ΠΎΡ‰Π°Π΄ΡŒ сСчСния ΠΏΡ€ΠΎΠ²ΠΎΠ΄Π° β€” Π Π΅ΠΌΠΎΠ½Ρ‚ Π² ΠΊΠ²Π°Ρ€Ρ‚ΠΈΡ€Π΅

Π‘ΠΎΠ΄Π΅Ρ€ΠΆΠ°Π½ΠΈΠ΅

ΠŸΠ»ΠΎΡ‰Π°Π΄ΡŒ сСчСния ΠΏΡ€ΠΎΠ²ΠΎΠ΄ΠΎΠ² ΠΈ ΠΊΠ°Π±Π΅Π»Π΅ΠΉ Π² зависимости ΠΎΡ‚ силы Ρ‚ΠΎΠΊΠ°, расчСт Π½Π΅ΠΎΠ±Ρ…ΠΎΠ΄ΠΈΠΌΠΎΠ³ΠΎ сСчСния кабСля

Если старая ΠΏΡ€ΠΎΠ²ΠΎΠ΄ΠΊΠ° Π²Ρ‹ΡˆΠ»Π° ΠΈΠ· строя Π½ΡƒΠΆΠ½ΠΎ Π΅Ρ‘ Π·Π°ΠΌΠ΅Π½ΠΈΡ‚ΡŒ, Π½ΠΎ ΠΏΡ€Π΅ΠΆΠ΄Π΅ Ρ‡Π΅ΠΌ ΠΌΠ΅Π½ΡΡ‚ΡŒ Π½Π° Π°Π½Π°Π»ΠΎΠ³ΠΈΡ‡Π½ΡƒΡŽ, ΡƒΠ·Π½Π°ΠΉΡ‚Π΅, ΠΏΠΎΡ‡Π΅ΠΌΡƒ ΠΏΡ€ΠΎΠΈΠ·ΠΎΡˆΠ»Π° ΠΏΡ€ΠΎΠ±Π»Π΅ΠΌΠ° со старой. Π’ΠΎΠ·ΠΌΠΎΠΆΠ½ΠΎ, Ρ‡Ρ‚ΠΎ Π±Ρ‹Π»ΠΎ просто мСханичСскоС ΠΏΠΎΠ²Ρ€Π΅ΠΆΠ΄Π΅Π½ΠΈΠ΅, ΠΈΠ»ΠΈ изоляция ΠΏΡ€ΠΈΡˆΠ»Π° Π² Π½Π΅Π³ΠΎΠ΄Π½ΠΎΡΡ‚ΡŒ, Π° Π΅Ρ‰Π΅ Π±ΠΎΠ»Π΅Π΅ вСсомой ΠΏΡ€ΠΎΠ±Π»Π΅ΠΌΠΎΠΉ являСтся – Π²Ρ‹Ρ…ΠΎΠ΄ ΠΈΠ· строя ΠΏΡ€ΠΎΠ²ΠΎΠ΄ΠΊΠΈ ΠΈΠ·-Π·Π° ΠΏΡ€Π΅Π²Ρ‹ΡˆΠ΅Π½ΠΈΡ допустимой Π½Π°Π³Ρ€ΡƒΠ·ΠΊΠΈ.

Π§Π΅ΠΌ отличаСтся кабСльная продукция, ΠΊΠ°ΠΊΠΈΠ΅ основныС характСристики?

НачнСм с Ρ‚ΠΎΠ³ΠΎ, Ρ‡Ρ‚ΠΎ опрСдСляСтся, ΠΊΠ°ΠΊΠΎΠ΅ напряТСниС Π² сСти, Π² ΠΊΠΎΡ‚ΠΎΡ€ΠΎΠΉ Π±ΡƒΠ΄ΡƒΡ‚ Ρ€Π°Π±ΠΎΡ‚Π°Ρ‚ΡŒ кабСля. Для Π±Ρ‹Ρ‚ΠΎΠ²Ρ‹Ρ… сСтСй часто ΠΏΡ€ΠΈΠΌΠ΅Π½ΡΡŽΡ‚ΡΡ кабСля ΠΈ ΠΏΡ€ΠΎΠ²ΠΎΠ΄Π° Ρ‚ΠΈΠΏΠ° Π’Π’Π“, ΠŸΠ£Π“ΠΠŸ (Ρ‚ΠΎΠ»ΡŒΠΊΠΎ ΠΎΠ½ Π·Π°ΠΏΡ€Π΅Ρ‰Π΅Π½ соврСмСнными трСбованиями ПУЭ ΠΈΠ·-Π·Π° Π±ΠΎΠ»ΡŒΡˆΠΈΡ… допусков ΠΏΠΎ ΡΠ΅Ρ‡Π΅Π½ΠΈΡŽ ΠΏΡ€ΠΈ производствС, Π΄ΠΎ 30%, ΠΈ допустимой Ρ‚ΠΎΠ»Ρ‰ΠΈΠ½Π΅ ΠΈΠ·ΠΎΠ»ΠΈΡ€ΡƒΡŽΡ‰Π΅Π³ΠΎ слоя 0.3ΠΌΠΌ, ΠΏΡ€ΠΎΡ‚ΠΈΠ² 0.4 Π² ПУЭ), Π¨Π’Π’ΠŸ ΠΈ Π΄Ρ€ΡƒΠ³ΠΈΠ΅.

Если ΠΎΡ‚ΠΎΠΉΡ‚ΠΈ ΠΎΡ‚ ΠΎΠΏΡ€Π΅Π΄Π΅Π»Π΅Π½ΠΈΠΉ ΠΏΡ€ΠΎΠ²ΠΎΠ΄ ΠΎΡ‚ кабСля отличаСтся минимально, Π² основном ΠΏΠΎ ΠΎΠΏΡ€Π΅Π΄Π΅Π»Π΅Π½ΠΈΡŽ Π² Π“ΠžΠ‘Π’Π΅ ΠΈΠ»ΠΈ Π’Π£ ΠΏΠΎ ΠΊΠΎΡ‚ΠΎΡ€ΠΎΠΌΡƒ ΠΎΠ½ производится.

Π’Π΅Π΄ΡŒ Π½Π° Ρ€Ρ‹Π½ΠΊΠ΅ Π΅ΡΡ‚ΡŒ большоС количСство ΠΏΡ€ΠΎΠ²ΠΎΠ΄ΠΎΠ² с 2-3 ΠΆΠΈΠ»Π°ΠΌΠΈ ΠΈ двумя слоями изоляции, Π½Π°ΠΏΡ€ΠΈΠΌΠ΅Ρ€ Ρ‚ΠΎΡ‚ ΠΆΠ΅ ΠŸΠ£Π“ΠΠŸ ΠΈΠ»ΠΈ ПУНП.

ДопустимоС напряТСниС опрСдСляСтся изоляциСй кабСля

Для Π²Ρ‹Π±ΠΎΡ€Π° кабСля ΠΊΡ€ΠΎΠΌΠ΅ напряТСния ΠΏΡ€ΠΈΠ½ΠΈΠΌΠ°ΡŽΡ‚ Π²ΠΎ Π²Π½ΠΈΠΌΠ°Π½ΠΈΠ΅ ΠΈ условия, Π² ΠΊΠΎΡ‚ΠΎΡ€Ρ‹Ρ… ΠΎΠ½ Π±ΡƒΠ΄Π΅Ρ‚ Ρ€Π°Π±ΠΎΡ‚Π°Ρ‚ΡŒ, для ΠΏΠΎΠ΄ΠΊΠ»ΡŽΡ‡Π΅Π½ΠΈΡ двиТущСгося инструмСнта ΠΈ оборудования ΠΎΠ½ Π΄ΠΎΠ»ΠΆΠ΅Π½ Π±Ρ‹Ρ‚ΡŒ Π³ΠΈΠ±ΠΊΠΈΠΌ, для ΠΏΠΎΠ΄ΠΊΠ»ΡŽΡ‡Π΅Π½ΠΈΡ Π½Π΅ΠΏΠΎΠ΄Π²ΠΈΠΆΠ½Ρ‹Ρ… элСмСнтов, Π² ΠΏΡ€ΠΈΠ½Ρ†ΠΈΠΏΠ΅, всС Ρ€Π°Π²Π½ΠΎ, Π½ΠΎ Π»ΡƒΡ‡ΡˆΠ΅ ΠΏΡ€Π΅Π΄ΠΏΠΎΡ‡Π΅ΡΡ‚ΡŒ кабСль с ΠΌΠΎΠ½ΠΎΠ»ΠΈΡ‚Π½ΠΎΠΉ ΠΆΠΈΠ»ΠΎΠΉ.

Π Π΅ΡˆΠ°ΡŽΡ‰ΠΈΠΌ Ρ„Π°ΠΊΡ‚ΠΎΡ€ΠΎΠΌ ΠΏΡ€ΠΈ ΠΏΠΎΠΊΡƒΠΏΠΊΠ΅ являСтся ΠΏΠ»ΠΎΡ‰Π°Π΄ΡŒ ΠΏΠΎΠΏΠ΅Ρ€Π΅Ρ‡Π½ΠΎΠ³ΠΎ сСчСния ΠΆΠΈΠ»Ρ‹, ΠΎΠ½Π° измСряСтся Π² ΠΌΠΌ2, ΠΎΡ‚ Π½Π΅Ρ‘ ΠΈ зависит ΡΠΏΠΎΡΠΎΠ±Π½ΠΎΡΡ‚ΡŒ ΠΏΡ€ΠΎΠ²ΠΎΠ΄Π½ΠΈΠΊΠ° Π²Ρ‹Π΄Π΅Ρ€ΠΆΠΈΠ²Π°Ρ‚ΡŒ Π΄Π»ΠΈΡ‚Π΅Π»ΡŒΠ½ΡƒΡŽ Π½Π°Π³Ρ€ΡƒΠ·ΠΊΡƒ.

Π§Ρ‚ΠΎ влияСт Π½Π° допустимый Ρ‚ΠΎΠΊ Ρ‡Π΅Ρ€Π΅Π· кабСль?

Для Π½Π°Ρ‡Π°Π»Π° обратимся ΠΊ основам Ρ„ΠΈΠ·ΠΈΠΊΠΈ. Π•ΡΡ‚ΡŒ Ρ‚Π°ΠΊΠΎΠΉ Π·Π°ΠΊΠΎΠ½ ДТоуля-Π›Π΅Π½Ρ†Π°, ΠΎΠ½ Π±Ρ‹Π» ΠΎΡ‚ΠΊΡ€Ρ‹Ρ‚ нСзависимо Π΄Ρ€ΡƒΠ³ ΠΎΡ‚ Π΄Ρ€ΡƒΠ³Π° двумя ΡƒΡ‡Π΅Π½Π½Ρ‹ΠΌΠΈ ДТСймсом Π”ΠΆΠΎΡƒΠ»Π΅ΠΌ (Π² 1841) ΠΈ Π­ΠΌΠΈΠ»ΠΈΠ΅ΠΌ Π›Π΅Π½Ρ†ΠΎΠΌ (Π² 1842), поэтому ΠΈ ΠΏΠΎΠ»ΡƒΡ‡ΠΈΠ» Π΄Π²ΠΎΠΉΠ½ΠΎΠ΅ Π½Π°Π·Π²Π°Π½ΠΈΠ΅.

Π’Π°ΠΊ Π²ΠΎΡ‚ этот Π·Π°ΠΊΠΎΠ½ количСствСнно описываСт Ρ‚Π΅ΠΏΠ»ΠΎΠ²ΠΎΠ΅ дСйствиС элСктричСского Ρ‚ΠΎΠΊΠ° ΠΏΡ€ΠΎΡ‚Π΅ΠΊΠ°ΡŽΡ‰Π΅Π³ΠΎ Ρ‡Π΅Ρ€Π΅Π· ΠΏΡ€ΠΎΠ²ΠΎΠ΄Π½ΠΈΠΊ.

Если Π²Ρ‹Ρ€Π°Π·ΠΈΡ‚ΡŒ Π΅Π³ΠΎ Ρ‡Π΅Ρ€Π΅Π· ΠΏΠ»ΠΎΡ‚Π½ΠΎΡΡ‚ΡŒ Ρ‚ΠΎΠΊΠ° получится такая Ρ„ΠΎΡ€ΠΌΡƒΠ»Π°:

Π Π°ΡΡˆΠΈΡ„Ρ€ΠΎΠ²ΠΊΠ°: w – ΠΌΠΎΡ‰Π½ΠΎΡΡ‚ΡŒ выдСлСния Ρ‚Π΅ΠΏΠ»Π° Π² Π΅Π΄ΠΈΠ½ΠΈΡ†Π΅ объСма, Π²Π΅ΠΊΡ‚ΠΎΡ€ j – ΠΏΠ»ΠΎΡ‚Π½ΠΎΡΡ‚ΡŒ Ρ‚ΠΎΠΊΠ° Ρ‡Π΅Ρ€Π΅Π· ΠΏΡ€ΠΎΠ²ΠΎΠ΄Π½ΠΈΠΊ измСряСтся Π² АмпСрах Π½Π° ΠΌΠΌ2. Для ΠΌΠ΅Π΄Π½ΠΎΠ³ΠΎ ΠΏΡ€ΠΎΠ²ΠΎΠ΄Π° ΠΏΡ€ΠΈΠ½ΠΈΠΌΠ°ΡŽΡ‚ ΠΎΡ‚ 6 Π΄ΠΎ 10 А Π½Π° ΠΌΠΈΠ»Π»ΠΈΠΌΠ΅Ρ‚Ρ€ ΠΏΠ»ΠΎΡ‰Π°Π΄ΠΈ, Π³Π΄Π΅ 6 – рабочая ΠΏΠ»ΠΎΡ‚Π½ΠΎΡΡ‚ΡŒ, Π° 10 кратковрСмСнная. Π²Π΅ΠΊΡ‚ΠΎΡ€ E – Π½Π°ΠΏΡ€ΡΠΆΠ΅Π½Π½ΠΎΡΡ‚ΡŒ элСктричСского поля. Οƒ – ΠΏΡ€ΠΎΠ²ΠΎΠ΄ΠΈΠΌΠΎΡΡ‚ΡŒ срСды.

Π’Π°ΠΊ ΠΊΠ°ΠΊ ΠΏΡ€ΠΎΠ²ΠΎΠ΄ΠΈΠΌΠΎΡΡ‚ΡŒ ΠΎΠ±Ρ€Π°Ρ‚Π½ΠΎ ΠΏΡ€ΠΎΠΏΠΎΡ€Ρ†ΠΈΠΎΠ½Π°Π»ΡŒΠ½Π° ΡΠΎΠΏΡ€ΠΎΡ‚ΠΈΠ²Π»Π΅Π½ΠΈΡŽ: Οƒ=1/R

Если Π²Ρ‹Ρ€Π°Π·ΠΈΡ‚ΡŒ Π·Π°ΠΊΠΎΠ½ ДТоуля-Π›Π΅Π½Ρ†Π° Ρ‡Π΅Ρ€Π΅Π· количСство Ρ‚Π΅ΠΏΠ»ΠΎΡ‚Ρ‹ Π² ΠΈΠ½Ρ‚Π΅Π³Ρ€Π°Π»ΡŒΠ½ΠΎΠΉ Ρ„ΠΎΡ€ΠΌΠ΅, Ρ‚ΠΎ:

Π’Π°ΠΊΠΈΠΌ ΠΎΠ±Ρ€Π°Π·ΠΎΠΌ, dQ – количСство Ρ‚Π΅ΠΏΠ»ΠΎΡ‚Ρ‹, ΠΊΠΎΡ‚ΠΎΡ€ΠΎΠ΅ выдСлится Π·Π° ΠΏΡ€ΠΎΠΌΠ΅ΠΆΡƒΡ‚ΠΎΠΊ Π²Ρ€Π΅ΠΌΠ΅Π½ΠΈ dt Π² Ρ†Π΅ΠΏΠΈ, Π³Π΄Π΅ ΠΏΡ€ΠΎΡ‚Π΅ΠΊΠ°Π΅Ρ‚ Ρ‚ΠΎΠΊ I, Ρ‡Π΅Ρ€Π΅Π· ΠΏΡ€ΠΎΠ²ΠΎΠ΄Π½ΠΈΠΊ сопротивлСниСм R.

Π’ΠΎ Π΅ΡΡ‚ΡŒ количСство Ρ‚Π΅ΠΏΠ»Π° прямо ΠΏΡ€ΠΎΠΏΠΎΡ€Ρ†ΠΈΠΎΠ½Π°Π»ΡŒΠ½ΠΎ Ρ‚ΠΎΠΊΡƒ ΠΈ ΡΠΎΠΏΡ€ΠΎΡ‚ΠΈΠ²Π»Π΅Π½ΠΈΡŽ. Π§Π΅ΠΌ большС Ρ‚ΠΎΠΊ ΠΈ сопротивлСниС – Ρ‚Π΅ΠΌ большС выдСляСтся Ρ‚Π΅ΠΏΠ»Π°. Π­Ρ‚ΠΎ опасно Ρ‚Π΅ΠΌ, Ρ‡Ρ‚ΠΎ Π² ΠΎΠΏΡ€Π΅Π΄Π΅Π»Π΅Π½Π½Ρ‹ΠΉ ΠΌΠΎΠΌΠ΅Π½Ρ‚ количСство Ρ‚Π΅ΠΏΠ»Π° достигнСт Ρ‚Π°ΠΊΠΎΠ³ΠΎ значСния, Ρ‡Ρ‚ΠΎ Ρƒ ΠΏΡ€ΠΎΠ²ΠΎΠ΄ΠΎΠ² плавится изоляция. Π’Ρ‹ ΠΌΠΎΠ³Π»ΠΈ Π·Π°ΠΌΠ΅Ρ‡Π°Ρ‚ΡŒ, Ρ‡Ρ‚ΠΎ ΠΏΡ€ΠΎΠ²ΠΎΠ΄Π° Π΄Π΅ΡˆΠ΅Π²Ρ‹Ρ… ΠΊΠΈΠΏΡΡ‚ΠΈΠ»ΡŒΠ½ΠΈΠΊΠΎΠ² ΠΎΡ‰ΡƒΡ‚ΠΈΠΌΠΎ Ρ‚Π΅ΠΏΠ»Π΅ΡŽΡ‚ Π²ΠΎ врСмя Ρ€Π°Π±ΠΎΡ‚Ρ‹, это ΠΎΠ½ΠΎ ΠΈ Π΅ΡΡ‚ΡŒ.

Если выдСляСтся ΠΌΠΎΡ‰Π½ΠΎΡΡ‚ΡŒ Π½Π° ΠΊΠ°Π±Π΅Π»Π΅, Π·Π½Π°Ρ‡ΠΈΡ‚, ΠΏΠ°Π΄Π°Π΅Ρ‚ ΠΈ напряТСниС Π½Π° Π΅Π³ΠΎ ΠΊΠΎΠ½Ρ†Π°Ρ…, ΠΏΠΎΠ΄ΠΊΠ»ΡŽΡ‡Π΅Π½Π½Ρ‹Ρ… ΠΊ Π½Π°Π³Ρ€ΡƒΠ·ΠΊΠ΅.

Π’ ΠΊΠ°Π»ΡŒΠΊΡƒΠ»ΡΡ‚ΠΎΡ€Π°Ρ… для расчСта сСчСний кабСля, ΠΎΠ±Ρ‹Ρ‡Π½ΠΎ Π·Π°Π΄Π°ΡŽΡ‚ΡΡ Ρ‚Π°ΠΊΠΈΠ΅ ΠΏΠ°Ρ€Π°ΠΌΠ΅Ρ‚Ρ€Ρ‹:

Π§Π΅ΠΌ большС сопротивлСниС – Ρ‚Π΅ΠΌ большС ΡƒΠΏΠ°Π΄Π΅Ρ‚ напряТСниС ΠΈ нагрССтся кабСль, ΠΏΠΎΡΠΊΠΎΠ»ΡŒΠΊΡƒ Π½Π° Π½Π΅ΠΌ выдСлится ΠΌΠΎΡ‰Π½ΠΎΡΡ‚ΡŒ (P=UI, Π³Π΄Π΅ U ΠΏΠ°Π΄Π΅Π½ΠΈΠ΅ напряТСния Π½Π° ΠΊΠ°Π±Π΅Π»Π΅, I – Ρ‚ΠΎΠΊ, ΠΏΡ€ΠΎΡ‚Π΅ΠΊΠ°ΡŽΡ‰ΠΈΠΉ Ρ‡Π΅Ρ€Π΅Π· Π½Π΅Π³ΠΎ).

ВсС расчСты свСлись ΠΊ Ρ‚ΠΎΠΊΡƒ ΠΈ ΡΠΎΠΏΡ€ΠΎΡ‚ΠΈΠ²Π»Π΅Π½ΠΈΡŽ. Π‘ΠΎΠΏΡ€ΠΎΡ‚ΠΈΠ²Π»Π΅Π½ΠΈΠ΅ ΠΏΡ€ΠΎΠ²ΠΎΠ΄Π½ΠΈΠΊΠ° вычисляСтся ΠΏΠΎ Ρ„ΠΎΡ€ΠΌΡƒΠ»Π΅:

Π—Π΄Π΅ΡΡŒ: ρ (Ρ€ΠΎ) – ΡƒΠ΄Π΅Π»ΡŒΠ½ΠΎΠ΅ сопротивлСниС, l – Π΄Π»ΠΈΠ½Π° кабСля, S – ΠΏΠ»ΠΎΡ‰Π°Π΄ΡŒ ΠΏΠΎΠΏΠ΅Ρ€Π΅Ρ‡Π½ΠΎΠ³ΠΎ сСчСния.

УдСльноС сопротивлСниС зависит ΠΎΡ‚ структуры ΠΌΠ΅Ρ‚Π°Π»Π»Π°, Π²Π΅Π»ΠΈΡ‡ΠΈΠ½Ρ‹ ΡƒΠ΄Π΅Π»ΡŒΠ½Ρ‹Ρ… сопротивлСний ΠΌΠΎΠΆΠ½ΠΎ ΠΎΠΏΡ€Π΅Π΄Π΅Π»ΠΈΡ‚ΡŒ ΠΈΠ· Ρ‚Π°Π±Π»ΠΈΡ†Ρ‹.

Π’ ΠΏΡ€ΠΎΠ²ΠΎΠ΄ΠΊΠ΅ Π² основном ΠΈΡΠΏΠΎΠ»ΡŒΠ·ΡƒΡŽΡ‚ΡΡ алюминий ΠΈ мСдь. Π£ ΠΌΠ΅Π΄ΠΈ сопротивлСниС 1.68*10-8 Ом*ΠΌΠΌ2/ΠΌ., Π° Ρƒ аллюминия Π² 1.8 Ρ€Π°Π·Π° большС Ρ‡Π΅ΠΌ Ρƒ ΠΌΠ΅Π΄ΠΈ, равняСтся 2.82*10-8 Ом*ΠΌΠΌ2/ΠΌ. Π­Ρ‚ΠΎ Π·Π½Π°Ρ‡ΠΈΡ‚, Ρ‡Ρ‚ΠΎ Π°Π»ΡŽΠΌΠΈΠ½ΠΈΠ΅Π²Ρ‹ΠΉ ΠΏΡ€ΠΎΠ²ΠΎΠ΄ нагрССтся ΠΏΠΎΡ‡Ρ‚ΠΈ Π² 2 Ρ€Π°Π·Π° сильнСС, Ρ‡Π΅ΠΌ ΠΌΠ΅Π΄Π½Ρ‹ΠΉ ΠΏΡ€ΠΈ ΠΎΠ΄ΠΈΠ½Π°ΠΊΠΎΠ²ΠΎΠΌ сСчСнии ΠΈ Ρ‚ΠΎΠΊΠ΅. ΠžΡ‚ΡΡŽΠ΄Π° слСдуСт, Ρ‡Ρ‚ΠΎ для ΠΏΡ€ΠΎΠΊΠ»Π°Π΄ΠΊΠΈ ΠΏΡ€ΠΎΠ²ΠΎΠ΄ΠΊΠΈ придСтся ΠΏΠΎΠΊΡƒΠΏΠ°Ρ‚ΡŒ Π±ΠΎΠ»Π΅Π΅ толстый Π°Π»ΡŽΠΌΠΈΠ½ΠΈΠ΅Π²Ρ‹ΠΉ ΠΏΡ€ΠΎΠ²ΠΎΠ΄, ΠΊ Ρ‚ΠΎΠΌΡƒ ΠΆΠ΅ ΠΆΠΈΠ»Ρ‹ Π»Π΅Π³ΠΊΠΎ ΠΏΠΎΠ²Ρ€Π΅Π΄ΠΈΡ‚ΡŒ.

ΠŸΠΎΡΡ‚ΠΎΠΌΡƒ ΠΌΠ΅Π΄Π½Ρ‹Π΅ ΠΏΡ€ΠΎΠ²ΠΎΠ΄Π° вытСснили с домашнСй ΠΏΡ€ΠΎΠ²ΠΎΠ΄ΠΊΠΈ ΠΌΠ΅Π΄Π½Ρ‹Π΅, Π° ΠΏΡ€ΠΈΠΌΠ΅Π½Π΅Π½ΠΈΠ΅ аллюминия Π² ΠΏΡ€ΠΎΠ²ΠΎΠ΄ΠΊΠ΅ Π·Π°ΠΏΡ€Π΅Ρ‰Π΅Π½ΠΎ, Ρ€Π°Π·Ρ€Π΅ΡˆΠ°Π΅Ρ‚ΡΡ Ρ‚ΠΎΠ»ΡŒΠΊΠΎ ΠΏΡ€ΠΈΠΌΠ΅Π½Π΅Π½ΠΈΠ΅ Π°Π»ΡŽΠΌΠΈΠ½ΠΈΠ΅Π²Ρ‹Ρ… ΠΊΠ°Π±Π΅Π»Π΅ΠΉ для ΠΌΠΎΠ½Ρ‚Π°ΠΆΠ° ΠΎΡ‡Π΅Π½ΡŒ ΠΌΠΎΡ‰Π½Ρ‹Ρ… элСктроустановок, ΠΏΠΎΡ‚Ρ€Π΅Π±Π»ΡΡŽΡ‰ΠΈΡ… большой Ρ‚ΠΎΠΊ, Ρ‚ΠΎΠ³Π΄Π° ΠΈΡΠΏΠΎΠ»ΡŒΠ·ΡƒΡŽΡ‚ ΠΏΡ€ΠΎΠ²ΠΎΠ΄ ΠΈΠ· аллюминия сСчСниСм большС 16 ΠΌΠΌ2 (смотритС —Β ΠŸΠΎΡ‡Π΅ΠΌΡƒ Π°Π»ΡŽΠΌΠΈΠ½ΠΈΠ΅Π²Ρ‹ΠΉ кабль нСльзя ΠΈΡΠΏΠΎΠ»ΡŒΠ·ΠΎΠ²Π°Ρ‚ΡŒ Π² элСктропроводкС)

Как ΠΎΠΏΡ€Π΅Π΄Π΅Π»ΠΈΡ‚ΡŒ сопротивлСниС ΠΏΡ€ΠΎΠ²ΠΎΠ΄Π° ΠΏΠΎ Π΄ΠΈΠ°ΠΌΠ΅Ρ‚Ρ€Ρƒ ΠΆΠΈΠ»Ρ‹?

Π‘Ρ‹Π²Π°ΡŽΡ‚ случаи, ΠΊΠΎΠ³Π΄Π° ΠΏΠ»ΠΎΡ‰Π°Π΄ΡŒ ΠΏΠΎΠΏΠ΅Ρ€Π΅Ρ‡Π½ΠΎΠ³ΠΎ сСчСния ΠΆΠΈΠ»Ρ‹ Π½Π΅ извСстна, поэтому ΠΌΠΎΠΆΠ½ΠΎ ΠΏΠΎΡΡ‡ΠΈΡ‚Π°Ρ‚ΡŒ ΠΏΠΎ Π΄ΠΈΠ°ΠΌΠ΅Ρ‚Ρ€Ρƒ.

Для опрСдСлСния Π΄ΠΈΠ°ΠΌΠ΅Ρ‚Ρ€Π° ΠΌΠΎΠ½ΠΎΠ»ΠΈΡ‚Π½ΠΎΠΉ ΠΆΠΈΠ»Ρ‹ ΠΌΠΎΠΆΠ½ΠΎ ΠΈΡΠΏΠΎΠ»ΡŒΠ·ΠΎΠ²Π°Ρ‚ΡŒ ΡˆΡ‚Π°Π½Π³Π΅Π½Ρ†ΠΈΡ€ΠΊΡƒΠ»ΡŒ, Ссли Π΅Π³ΠΎ Π½Π΅Ρ‚, Ρ‚ΠΎ Π²ΠΎΠ·ΡŒΠΌΠΈΡ‚Π΅ ΡΡ‚Π΅Ρ€ΠΆΠ΅Π½ΡŒ, Π½Π°ΠΏΡ€ΠΈΠΌΠ΅Ρ€ ΡˆΠ°Ρ€ΠΈΠΊΠΎΠ²ΡƒΡŽ Ρ€ΡƒΡ‡ΠΊΡƒ ΠΈΠ»ΠΈ гвоздь, Π½Π°ΠΌΠΎΡ‚Π°ΠΉΡ‚Π΅ ΠΏΠ»ΠΎΡ‚Π½ΠΎ 10 Π²ΠΈΡ‚ΠΊΠΎΠ² ΠΏΡ€ΠΎΠ²ΠΎΠ΄Π° Π½Π° Π½Π΅Π³ΠΎ, ΠΈ ΠΈΠ·ΠΌΠ΅Ρ€ΡŒΡ‚Π΅ Π»ΠΈΠ½Π΅ΠΉΠΊΠΎΠΉ Π΄Π»ΠΈΠ½Ρƒ ΠΏΠΎΠ»ΡƒΡ‡ΠΈΠ²ΡˆΠ΅ΠΉΡΡ спирали, Ρ€Π°Π·Π΄Π΅Π»ΠΈΠ² эту Π΄Π»ΠΈΠ½Ρƒ Π½Π° 10 – Π²Ρ‹ ΠΏΠΎΠ»ΡƒΡ‡ΠΈΡ‚Π΅ Π΄ΠΈΠ°ΠΌΠ΅Ρ‚Ρ€ ΠΆΠΈΠ»Ρ‹.

Для опрСдСлСния ΠΎΠ±Ρ‰Π΅Π³ΠΎ Π΄ΠΈΠ°ΠΌΠ΅Ρ‚Ρ€Π° ΠΌΠ½ΠΎΠ³ΠΎΠΏΡ€ΠΎΠ²ΠΎΠ»ΠΎΡ‡Π½ΠΎΠΉ ΠΆΠΈΠ»Ρ‹, ΠΈΠ·ΠΌΠ΅Ρ€ΡŒΡ‚Π΅ Π΄ΠΈΠ°ΠΌΠ΅Ρ‚Ρ€ ΠΊΠ°ΠΆΠ΄ΠΎΠΉ ΠΆΠΈΠ»Ρ‹ ΠΈ ΡƒΠΌΠ½ΠΎΠΆΡŒΡ‚Π΅ Π½Π° ΠΈΡ… количСство.

Π”Π°Π»ΡŒΡˆΠ΅ ΡΡ‡ΠΈΡ‚Π°ΡŽΡ‚ ΠΏΠΎΠΏΠ΅Ρ€Π΅Ρ‡Π½ΠΎΠ΅ сСчСниС ΠΏΠΎ этой Ρ„ΠΎΡ€ΠΌΡƒΠ»Π΅:

И вновь Π²ΠΎΠ·Π²Ρ€Π°Ρ‰Π°ΡŽΡ‚ΡΡ ΠΊ этой Ρ„ΠΎΡ€ΠΌΡƒΠ»Π΅ для расчСта сопротивлСния ΠΏΡ€ΠΎΠ²ΠΎΠ΄Π°:

Как ΠΎΠΏΡ€Π΅Π΄Π΅Π»ΠΈΡ‚ΡŒ Π½Π΅ΠΎΠ±Ρ…ΠΎΠ΄ΠΈΠΌΡƒΡŽ ΠΏΠ»ΠΎΡ‰Π°Π΄ΡŒ сСчСния ΠΏΡ€ΠΎΠ²ΠΎΠ΄Π°?

Π‘Π°ΠΌΡ‹ΠΉ простой Π²Π°Ρ€ΠΈΠ°Π½Ρ‚ – ΠΎΠΏΡ€Π΅Π΄Π΅Π»ΠΈΡ‚ΡŒ ΠΏΠ»ΠΎΡ‰Π°Π΄ΡŒ сСчСния ΠΆΠΈΠ» ΠΏΠΎ Ρ‚Π°Π±Π»ΠΈΡ†Π΅. Он ΠΏΠΎΠ΄Ρ…ΠΎΠ΄ΠΈΡ‚ для расчСта Π½Π΅ слишком Π΄Π»ΠΈΠ½Π½Ρ‹Ρ… Π»ΠΈΠ½ΠΈΠΉ ΠΏΡ€ΠΎΠ»ΠΎΠΆΠ΅Π½Π½Ρ‹Ρ… Π² Π½ΠΎΡ€ΠΌΠ°Π»ΡŒΠ½Ρ‹Ρ… условиях (с Π½ΠΎΡ€ΠΌΠ°Π»ΡŒΠ½ΠΎΠΉ Ρ‚Π΅ΠΌΠΏΠ΅Ρ€Π°Ρ‚ΡƒΡ€ΠΎΠΉ ΠΎΠΊΡ€ΡƒΠΆΠ°ΡŽΡ‰Π΅ΠΉ срСды). Π’Π°ΠΊΠΆΠ΅ Ρ‚Π°ΠΊ ΠΌΠΎΠΆΠ½ΠΎ ΠΏΠΎΠ΄ΠΎΠ±Ρ€Π°Ρ‚ΡŒ ΠΏΡ€ΠΎΠ²ΠΎΠ΄ для удлинитСля. ΠžΠ±Ρ€Π°Ρ‚ΠΈΡ‚Π΅ Π²Π½ΠΈΠΌΠ°Π½ΠΈΠ΅, Ρ‡Ρ‚ΠΎ Π² Ρ‚Π°Π±Π»ΠΈΡ†Π΅ ΡƒΠΊΠ°Π·Π°Π½Ρ‹ сСчСния ΠΏΡ€ΠΈ ΠΎΠΏΡ€Π΅Π΄Π΅Π»Π΅Π½Π½ΠΎΠΌ Ρ‚ΠΎΠΊΠ΅ ΠΈ мощности Π² ΠΎΠ΄Π½ΠΎΡ„Π°Π·Π½ΠΎΠΉ ΠΈ Ρ‚Ρ€Ρ‘Ρ…Ρ„Π°Π·Π½ΠΎΠΉ сСти для аллюминия ΠΈ ΠΌΠ΅Π΄ΠΈ.

ΠŸΡ€ΠΈ расчСтС Π΄Π»ΠΈΠ½Π½Ρ‹Ρ… Π»ΠΈΠ½ΠΈΠΉ (большС 10 ΠΌΠ΅Ρ‚Ρ€ΠΎΠ²) Ρ‚Π°ΠΊΠΎΠΉ Ρ‚Π°Π±Π»ΠΈΡ†Π΅ΠΉ Π»ΡƒΡ‡ΡˆΠ΅ Π½Π΅ ΠΏΠΎΠ»ΡŒΠ·ΠΎΠ²Π°Ρ‚ΡŒΡΡ. НуТно провСсти расчСты. БыстрСС всСго Π²ΠΎΡΠΏΠΎΠ»ΡŒΠ·ΠΎΠ²Π°Ρ‚ΡŒΡΡ ΠΊΠ°Π»ΡŒΠΊΡƒΠ»ΡΡ‚ΠΎΡ€ΠΎΠΌ. Алгоритм расчСта Ρ‚Π°ΠΊΠΎΠΉ:

Π‘Π΅Ρ€ΡƒΡ‚ допустимыС ΠΏΠΎΡ‚Π΅Ρ€ΠΈ ΠΏΠΎ Π½Π°ΠΏΡ€ΡΠΆΠ΅Π½ΠΈΡŽ (Π½Π΅ Π±ΠΎΠ»Π΅Π΅ 5%), это Π·Π½Π°Ρ‡ΠΈΡ‚ Ρ‡Ρ‚ΠΎ ΠΏΡ€ΠΈ напряТСнии Π² сСти 220Π’ ΠΈ допустимым потСрям напряТСния Π² 5% Π½Π° ΠΊΠ°Π±Π΅Π»Π΅ ΠΏΠ°Π΄Π΅Π½ΠΈΠ΅ напряТСния (ΠΎΡ‚ ΠΊΠΎΠ½Ρ†Π° Π΄ΠΎ ΠΊΠΎΠ½Ρ†Π°) Π½Π΅ Π΄ΠΎΠ»ΠΆΠ½ΠΎ ΠΏΡ€Π΅Π²Ρ‹ΡˆΠ°Ρ‚ΡŒ:

5%*220=11Π’.

Π’Π΅ΠΏΠ΅Ρ€ΡŒ, зная Ρ‚ΠΎΠΊ, ΠΊΠΎΡ‚ΠΎΡ€Ρ‹ΠΉ Π±ΡƒΠ΄Π΅Ρ‚ ΠΏΡ€ΠΎΡ‚Π΅ΠΊΠ°Ρ‚ΡŒ, ΠΌΡ‹ ΠΌΠΎΠΆΠ΅Ρ‚ Π²Ρ‹Ρ‡ΠΈΡΠ»ΠΈΡ‚ΡŒ сопротивлСниС кабСля. Π’ Π΄Π²ΡƒΡ… ΠΏΡ€ΠΎΠ²ΠΎΠ΄Π½ΠΎΠΉ Π»ΠΈΠ½ΠΈΠΈ сопротивлСниС ΡƒΠΌΠ½ΠΎΠΆΠ°ΡŽΡ‚ Π½Π° 2, Ρ‚Π°ΠΊ ΠΊΠ°ΠΊ Ρ‚ΠΎΠΊ Ρ‚Π΅Ρ‡Π΅Ρ‚ ΠΏΠΎ Π΄Π²ΡƒΠΌ ΠΏΡ€ΠΎΠ²ΠΎΠ΄Π°ΠΌ, ΠΏΡ€ΠΈ Π»ΠΈΠ½ΠΈΠΈ Π΄Π»ΠΈΠ½ΠΎΠΉ Π² 10ΠΌ, общая Π΄Π»ΠΈΠ½Π° ΠΏΡ€ΠΎΠ²ΠΎΠ΄Π½ΠΈΠΊΠΎΠ² – 20ΠΌ.

ΠžΡ‚ΡΡŽΠ΄Π° ΠΏΠΎ Π²Ρ‹ΡˆΠ΅ΠΏΡ€ΠΈΠ²Π΅Π΄Π΅Π½Π½Ρ‹ΠΌ Ρ„ΠΎΡ€ΠΌΡƒΠ»Π°ΠΌ Π²Ρ‹Ρ‡ΠΈΡΠ»ΡΡŽΡ‚ Π½Π΅ΠΎΠ±Ρ…ΠΎΠ΄ΠΈΠΌΠΎΠ΅ ΠΏΠΎΠΏΠ΅Ρ€Π΅Ρ‡Π½ΠΎΠ΅ сСчСниС кабСля.

Π’Ρ‹ ΠΌΠΎΠΆΠ΅Ρ‚Π΅ ΡΠ΄Π΅Π»Π°Ρ‚ΡŒ это автоматичСски со своСго смартфона, с ΠΏΠΎΠΌΠΎΡ‰ΡŒΡŽ ΠΏΡ€ΠΈΠ»ΠΎΠΆΠ΅Π½ΠΈΠΉ Β«ΠœΠΎΠ±ΠΈΠ»ΡŒΠ½Ρ‹ΠΉ элСктрик» ΠΈ electroDroid. Волько Π² ΠΊΠ°Π»ΡŒΠΊΡƒΠ»ΡΡ‚ΠΎΡ€Π΅ задаСтся Π½Π΅ общая Π΄Π»ΠΈΠ½Π° ΠΏΡ€ΠΎΠ²ΠΎΠ΄ΠΎΠ², Π° ΠΈΠΌΠ΅Π½Π½ΠΎ Π΄Π»ΠΈΠ½Π° Π»ΠΈΠ½ΠΈΠΈ ΠΎΡ‚ источника питания ΠΊ ΠΏΡ€ΠΈΠ΅ΠΌΠ½ΠΈΠΊΡƒ элСктричСства.

Π—Π°ΠΊΠ»ΡŽΡ‡Π΅Π½ΠΈΠ΅

ΠŸΡ€Π°Π²ΠΈΠ»ΡŒΠ½ΠΎ рассчитанная ΠΏΡ€ΠΎΠ²ΠΎΠ΄ΠΊΠ° это ΡƒΠΆΠ΅ 50% Π·Π°Π»ΠΎΠ³ Π΅Ρ‘ ΡƒΡΠΏΠ΅ΡˆΠ½ΠΎΠ³ΠΎ функционирования, вторая ΠΏΠΎΠ»ΠΎΠ²ΠΈΠ½Π° зависит ΠΎΡ‚ ΠΏΡ€Π°Π²ΠΈΠ»ΡŒΠ½ΠΎΡΡ‚ΠΈ ΠΌΠΎΠ½Ρ‚Π°ΠΆΠ°. Π‘Π»Π΅Π΄ΡƒΠ΅Ρ‚ ΡƒΡ‡ΠΈΡ‚Ρ‹Π²Π°Ρ‚ΡŒ всС особСнности ΠΏΡ€ΠΎΠ²ΠΎΠ΄ΠΊΠΈ, ΠΌΠ°ΠΊΡΠΈΠΌΠ°Π»ΡŒΠ½ΡƒΡŽ ΠΏΠΎΡ‚Ρ€Π΅Π±Π»ΡΠ΅ΠΌΡƒΡŽ ΠΌΠΎΡ‰Π½ΠΎΡΡ‚ΡŒ всСми потрСбитСлями. ΠŸΡ€ΠΈ этом Π²Π²Π΅Π΄ΠΈΡ‚Π΅ запас ΠΏΠΎ допустимому Ρ‚ΠΎΠΊΡƒ Π½Π° 20-40% Β«Π½Π° всякий случай».

ΠžΠΏΡ€Π΅Π΄Π΅Π»ΡΠ΅ΠΌ ΠΏΠ»ΠΎΡ‰Π°Π΄ΡŒ свСчСния

Π—Π°Ρ‡Π΅ΠΌ ΠΆΠ΅ всС Ρ‚Π°ΠΊΠΈ ΠΏΡ€Π°Π²ΠΈΠ»ΡŒΠ½ΠΎ ΠΎΠΏΡ€Π΅Π΄Π΅Π»ΡΡ‚ΡŒ сСчСниС кабСля? Π§Ρ‚ΠΎ случится, Ссли произвСсти ΠΌΠΎΠ½Ρ‚Π°ΠΆ нСподходящСго кабСля?Β ΠŸΡ€Π΅Π΄ΠΏΠΎΠ»ΠΎΠΆΠΈΠΌ, Π²Ρ‹ рассчитали, Ρ‡Ρ‚ΠΎ номинальная Π½Π°Π³Ρ€ΡƒΠ·ΠΊΠ° Π΄Π°Π½Π½ΠΎΠΉ Π»ΠΈΠ½ΠΈΠΈΒ 25 А, Π·Π½Π°Ρ‡ΠΈΡ‚, для ΠΌΠΎΠ½Ρ‚Π°ΠΆΠ° элСктропроводки Π²Π°ΠΌ Π½ΡƒΠΆΠ΅Π½ кабСль Π΄ΠΈΠ°ΠΌΠ΅Ρ‚Ρ€ΠΎΠΌΒ 2,5 ΠΌΠΌ. ΠΊΠ².Β Π£ вас имССтся кабСль Π±Π΅Π· ΠΌΠ°Ρ€ΠΊΠΈΡ€ΠΎΠ²ΠΊΠΈ (ΠΌΠ°Ρ€ΠΊΠΈΡ€ΠΎΠ²ΠΊΠ° ΠΌΠΎΠ³Π»Π° ΡΡ‚Π΅Ρ€Π΅Ρ‚ΡŒΡΡ, ΡΠΌΠ°Π·Π°Ρ‚ΡŒΡΡ) ΠΏΠΎΡ…ΠΎΠΆΠΈΠΉ Π½Π°Β 2,5 ΠΌΠΌ ΠΊΠ²., Π½ΠΎ Π½Π° самом Π΄Π΅Π»Π΅ ΠΎΠ½ мСньшС —Β 1,5 ΠΌΠΌ. ΠΊΠ². КабСль с сСчСниСм 1,5 ΠΌΠΌΒ Π½Π΅ Π²Ρ‹Π΄Π΅Ρ€ΠΆΠΈΡ‚ ΠΏΠΎΠ΄ΠΎΠ±Π½ΠΎΠΉ Π½Π°Π³Ρ€ΡƒΠ·ΠΊΠΈ, Ρ‚Π°ΠΊ ΠΊΠ°ΠΊ ΠΎΠ½ ΠΏΡ€Π΅Π΄Π½Π°Π·Π½Π°Ρ‡Π΅Π½ для Π»ΠΈΠ½ΠΈΠΈ с Π½Π°Π³Ρ€ΡƒΠ·ΠΊΠΎΠΉΒ 10-12 А. Π’ Ρ€Π΅Π·ΡƒΠ»ΡŒΡ‚Π°Ρ‚Π΅ изоляция нагрССтся, оплавится, Ρ‡Ρ‚ΠΎ Π³Ρ€ΠΎΠ·ΠΈΡ‚ Π·Π°ΠΌΡ‹ΠΊΠ°Π½ΠΈΠ΅ΠΌ ΠΈ ΠΊΠ°ΠΊ слСдствиС — ΠΏΠΎΠΆΠ°Ρ€ΠΎΠΌ.

Β Π‘Ρ‹Π²Π°Π΅Ρ‚, Ρ‡Ρ‚ΠΎ ΠΏΡ€ΠΈΠΎΠ±Ρ€Π΅Ρ‚Π΅Π½Π½Ρ‹ΠΉ кабСль Π½Π° самом Π΄Π΅Π»Π΅ ΠΈΠΌΠ΅Π΅Ρ‚ сСчСниС мСньшС ΡƒΠΊΠ°Π·Π°Π½Π½ΠΎΠ³ΠΎ ΠΏΡ€ΠΎΠΈΠ·Π²ΠΎΠ΄ΠΈΡ‚Π΅Π»Π΅ΠΌ.  Допустим, Π²Ρ‹ ΠΏΡ€ΠΈΠΎΠ±Ρ€Π΅Π»ΠΈ ΠΏΡ€ΠΎΠ²ΠΎΠ΄ сСчСниСм 4 ΠΌΠΌ, хотя Π½Π° самом Π΄Π΅Π»Π΅ сСчСниС составляСт 3,5 ΠΌΠΌ. Π’ Ρ€Π΅Π·ΡƒΠ»ΡŒΡ‚Π°Ρ‚Π΅ Ρ‡Π΅Π³ΠΎ нагрузочная ΡΠΏΠΎΡΠΎΠ±Π½ΠΎΡΡ‚ΡŒ Ρ‚Π°ΠΊΠΆΠ΅ ΡƒΠΌΠ΅Π½ΡŒΡˆΠ°Π΅Ρ‚ΡΡ, Ρ‡Ρ‚ΠΎ Π²Π»Π΅Ρ‡Π΅Ρ‚ Π·Π° собой Π½Π΅Π³Π°Ρ‚ΠΈΠ²Π½Ρ‹Π΅ послСдствия.Β ΠŸΠΎΡ‡Π΅ΠΌΡƒ сСчСниС кабСля ΠΌΠΎΠΆΠ΅Ρ‚ Π±Ρ‹Ρ‚ΡŒ мСньшС ΡƒΠΊΠ°Π·Π°Π½Π½ΠΎΠ³ΠΎ? Π’Π°ΠΊ Π½Π΅ΠΊΠΎΡ‚ΠΎΡ€Ρ‹Π΅ ΠΊΠΎΠΌΠΏΠ°Π½ΠΈΠΈ хотят ΡΡΠΊΠΎΠ½ΠΎΠΌΠΈΡ‚ΡŒ ΠΊΡ€ΡƒΠ³Π»ΡƒΡŽ сумму Π΄Π΅Π½Π΅Π³, Π²ΠΎΡ‚ ΠΈ ΠΏΠΎΠ½ΠΈΠΆΠ°ΡŽΡ‚ сСчСниС ΠΏΡ€ΠΎΠ²ΠΎΠ΄Π°.Π’Π°ΠΊ Ρ‡Ρ‚ΠΎ Π·Π½Π°Ρ‚ΡŒ Π΄Π΅ΠΉΡΡ‚Π²ΠΈΡ‚Π΅Π»ΡŒΠ½ΠΎΠ΅ сСчСниС ΠΏΡ€ΠΎΠ²ΠΎΠ΄Π° Π½Π΅ΠΎΠ±Ρ…ΠΎΠ΄ΠΈΠΌΠΎ для бСзопасной ΠΈ ΠΏΡ€ΠΎΠ΄ΠΎΠ»ΠΆΠΈΡ‚Π΅Π»ΡŒΠ½ΠΎΠΉ эксплуатации.


РасчСт сСчСния кабСля ΠΏΠΎΠ΄Ρ€ΡƒΡ‡Π½Ρ‹ΠΌΠΈ срСдствами

Π§Ρ‚ΠΎΠ±Ρ‹ Ρ€Π°ΡΡΡ‡ΠΈΡ‚Π°Ρ‚ΡŒ ΠΏΠ»ΠΎΡ‰Π°Π΄ΡŒ сСчСния ΠΆΠΈΠ»Ρ‹, для Π½Π°Ρ‡Π°Π»Π° Π½ΡƒΠΆΠ½ΠΎ ΡƒΠ·Π½Π°Ρ‚ΡŒ Π΅Π΅ Π΄ΠΈΠ°ΠΌΠ΅Ρ‚Ρ€. Π’ этом ΠΏΠΎΠΌΠΎΠΆΠ΅Ρ‚ ΠΌΠΈΠΊΡ€ΠΎΠΌΠ΅Ρ‚Ρ€ — особый ΠΏΡ€ΠΈΠ±ΠΎΡ€, ΠΈΠ·ΠΌΠ΅Ρ€ΡΡŽΡ‰ΠΈΠΉ Π΄ΠΈΠ°ΠΌΠ΅Ρ‚Ρ€ ΠΆΠΈΠ»Ρ‹ ΠΏΡ€ΠΎΠ²ΠΎΠ΄Π° с высокой Ρ‚ΠΎΡ‡Π½ΠΎΡΡ‚ΡŒΡŽ. Для этой ΠΆΠ΅ Π·Π°Π΄Π°Ρ‡ΠΈ ΠΏΠΎΠ΄ΠΎΠΉΠ΄Π΅Ρ‚ ΡˆΡ‚Π°Π½Π³Π΅Π½Ρ†ΠΈΡ€ΠΊΡƒΠ»ΡŒ. ΠŸΡ€ΠΎΡ„Π΅ΡΡΠΈΠΎΠ½Π°Π»ΡŒΠ½ΠΎΠΌΡƒ элСктрику ΠΏΡ€ΠΈΠΎΠ±Ρ€Π΅Ρ‚Π΅Π½ΠΈΠ΅ ΠΌΠΈΠΊΡ€ΠΎΠΌΠ΅Ρ‚Ρ€Π° Π½Π΅ΠΎΠ±Ρ…ΠΎΠ΄ΠΈΠΌΠΎ Π² силу спСцифики Ρ€Π°Π±ΠΎΡ‚Ρ‹. Для простого Ρ‡Π΅Π»ΠΎΠ²Π΅ΠΊΠ°, ΠΊΠΎΡ‚ΠΎΡ€ΠΎΠΌΡƒ понадобилось произвСсти Π·Π°ΠΌΠ΅Ρ€ 1 Ρ€Π°Π·, Π½Π΅Ρ‚ смысла ΠΏΠΎΠΊΡƒΠΏΠ°Ρ‚ΡŒ ΠΌΠΈΠΊΡ€ΠΎΠΌΠ΅Ρ‚Ρ€.

 Но Ρ‡Ρ‚ΠΎ Π΄Π΅Π»Π°Ρ‚ΡŒ, Ссли Π΄Π°ΠΆΠ΅ ΡˆΡ‚Π°Π½Π³Π΅Π½Ρ†ΠΈΡ€ΠΊΡƒΠ»ΡŒ Π΄ΠΎΠΌΠ° Π½Π΅ нашСлся? Π•ΡΡ‚ΡŒ Π²ΠΏΠΎΠ»Π½Π΅ достойный, Π°Π»ΡŒΡ‚Π΅Ρ€Π½Π°Ρ‚ΠΈΠ²Π½Ρ‹ΠΉ ΠΌΠ΅Ρ‚ΠΎΠ΄. Β Π’Π°ΠΌ понадобится Π»ΠΈΠ½Π΅ΠΉΠΊΠ°, простой ΠΊΠ°Ρ€Π°Π½Π΄Π°Ρˆ ΠΈΠ»ΠΈ Ρ€ΡƒΡ‡ΠΊΠ°. Волько Π½Π΅ ΠΏΡ‹Ρ‚Π°ΠΉΡ‚Π΅ΡΡŒ ΠΈΠ·ΠΌΠ΅Ρ€ΠΈΡ‚ΡŒ Π΄ΠΈΠ°ΠΌΠ΅Ρ‚Ρ€ ΠΏΡ€ΠΈ ΠΏΠΎΠΌΠΎΡ‰ΠΈ Π»ΠΈΠ½Π΅ΠΉΠΊΠΈ! Π’Π°ΠΊΠΎΠΉ ΠΌΠ΅Ρ‚ΠΎΠ΄ даст Π±ΠΎΠ»ΡŒΡˆΡƒΡŽ ΠΏΠΎΠ³Ρ€Π΅ΡˆΠ½ΠΎΡΡ‚ΡŒ. Π›ΠΈΠ½Π΅ΠΉΠΊΠ° понадобится Ρ‡ΡƒΡ‚ΡŒ ΠΏΠΎΠ·ΠΆΠ΅. БСрСтся кусок ΠΏΡ€ΠΎΠ²ΠΎΠ΄Π°, ΠΏΡ€Π΅Π΄Π²Π°Ρ€ΠΈΡ‚Π΅Π»ΡŒΠ½ΠΎ Π·Π°Ρ‡ΠΈΡ‰Π΅Π½Π½Ρ‹ΠΉ ΠΎΡ‚ изоляции ΠΏΡ€ΠΈΠΌΠ΅Ρ€Π½ΠΎ Π½Π°Β 40 см, ΠΈ наматываСтся Π½Π° ΠΊΠ°Ρ€Π°Π½Π΄Π°Ρˆ. Π’Π°ΠΆΠ½ΠΎ Π½Π°ΠΌΠ°Ρ‚Ρ‹Π²Π°Ρ‚ΡŒ ΠΊΠ°ΠΊ ΠΌΠΎΠΆΠ½ΠΎ ΠΏΠ»ΠΎΡ‚Π½Π΅Π΅! Если ΠΌΠ΅ΠΆΠ΄Ρƒ Π²ΠΈΡ‚ΠΊΠ°ΠΌΠΈ останутся Π·Π°Π·ΠΎΡ€Ρ‹, измСрСния Π±ΡƒΠ΄ΡƒΡ‚ Π½Π΅Π²Π΅Ρ€Π½Ρ‹. БчитаСтся, сколько Π²ΠΈΡ‚ΠΊΠΎΠ² Π²Ρ‹ΡˆΠ»ΠΎ, Π° ΠΈΡ… Π΄Π»ΠΈΠ½Π° измСряСтся Π»ΠΈΠ½Π΅ΠΉΠΊΠΎΠΉ. НапримСр, Π½Π° ΠΊΠ°Ρ€Π°Π½Π΄Π°ΡˆΠ΅Β 21Β Π²ΠΈΡ‚ΠΎΠΊ, Π΄Π»ΠΈΠ½Π° Π²ΠΈΡ‚ΠΊΠΎΠ²Β 37 ΠΌΠΌ.Β Π”Π»ΠΈΠ½Π° Π²ΠΈΡ‚ΠΊΠΎΠ² дСлится Π½Π° ΠΈΡ… количСство ΠΈ получаСтся Π΄ΠΈΠ°ΠΌΠ΅Ρ‚Ρ€ ΠΆΠΈΠ»Ρ‹Β (37/21=1,762).


НСмного Π³Π΅ΠΎΠΌΠ΅Ρ‚Ρ€ΠΈΠΈ

Π”Π°Π»Π΅Π΅ Π½ΡƒΠΆΠ½ΠΎ Π²ΡΠΏΠΎΠΌΠ½ΠΈΡ‚ΡŒ курс школьной Π³Π΅ΠΎΠΌΠ΅Ρ‚Ρ€ΠΈΠΈ ΠΈ ΠΏΡ€ΠΈΠΌΠ΅Π½ΠΈΡ‚ΡŒ Ρ„ΠΎΡ€ΠΌΡƒΠ»Ρƒ вычислСния ΠΏΠ»ΠΎΡ‰Π°Π΄ΠΈ ΠΊΡ€ΡƒΠ³Π°:Β (S=ПИ*D2/4), для облСгчСния расчСтов ΠΌΠΎΠΆΠ½ΠΎ ΠΏΡ€Π΅ΠΎΠ±Ρ€Π°Π·ΠΎΠ²Π°Ρ‚ΡŒΒ Ρ„ΠΎΡ€ΠΌΡƒΠ»ΡƒΒ (S=0,785*D2), Π³Π΄Π΅Β DΒ — Π΄ΠΈΠ°ΠΌΠ΅Ρ‚Ρ€, Π°Β 0,785Β — Ρ‡ΠΈΡΠ»ΠΎΒ ΠŸΠ˜Β Ρ€Π°Π·Π΄Π΅Π»Π΅Π½Π½ΠΎΠ΅ Π½Π°Β 4. ΠŸΠΎΠ΄ΡΡ‚Π°Π²Π»ΡΠ΅ΠΌ наши значСния, Ρ€Π΅Π·ΡƒΠ»ΡŒΡ‚Π°Ρ‚ округляСм Π΄ΠΎ сотых:(1,762*1,762)*0,785=2,44 ΠΌΠΌ.Β Π’ΠΎΡ‡Π½ΠΎΡΡ‚ΡŒ ΠΏΠΎΠ»ΡƒΡ‡Π΅Π½Π½Ρ‹Ρ… Π΄Π°Π½Π½Ρ‹Ρ… зависит ΠΎΡ‚ плотности Π½Π°ΠΌΠΎΡ‚ΠΊΠΈ, ΠΈ количСства Π²ΠΈΡ‚ΠΊΠΎΠ² — Ρ‡Π΅ΠΌ ΠΈΡ… большС, Ρ‚Π΅ΠΌ Ρ‚ΠΎΡ‡Π½Π΅Π΅ Π±ΡƒΠ΄Π΅Ρ‚ Ρ€Π΅Π·ΡƒΠ»ΡŒΡ‚Π°Ρ‚.Β Π’ΠΎΡ‚ Ρ‚Π°ΠΊΠΈΠΌ простым ΠΌΠ΅Ρ‚ΠΎΠ΄ΠΎΠΌ ΠΌΠΎΠΆΠ½ΠΎ Π²Ρ‹Ρ‡ΠΈΡΠ»ΠΈΡ‚ΡŒ сСчСниС одноТильного кабСля.

ΠžΠΏΡ€Π΅Π΄Π΅Π»ΡΠ΅ΠΌ сСчСниС многоТильного ΠΏΡ€ΠΎΠ²ΠΎΠ΄Π°


Но Ссли кабСль ΠΌΠ½ΠΎΠ³ΠΎΠΆΠΈΠ»ΡŒΠ½Ρ‹ΠΉ? Β ΠŸΠΎΡ‚Ρ€Π΅Π±ΡƒΠ΅Ρ‚ΡΡ Π·Π½Π°Ρ‡Π΅Π½ΠΈΠ΅ сСчСния ΠΎΠ΄Π½ΠΎΠΉ ΠΈΠ· ΠΆΠΈΠ», Π° высчитываСтся ΠΎΠ½ΠΎ ΠΏΠΎ Π²Ρ‹ΡˆΠ΅ΠΏΡ€ΠΈΠ²Π΅Π΄Π΅Π½Π½ΠΎΠΉ Ρ„ΠΎΡ€ΠΌΡƒΠ»Π΅. Π”Π°Π»Π΅Π΅ умноТаСтся ΠΏΠ»ΠΎΡ‰Π°Π΄ΡŒ ΠΎΠ΄Π½ΠΎΠΉ ΠΆΠΈΠ»Ρ‹ Π½Π° ΠΈΡ… количСство. Ну Π½Π°ΠΏΡ€ΠΈΠΌΠ΅Ρ€: ΠΏΠ»ΠΎΡ‰Π°Π΄ΡŒ ΠΆΠΈΠ»Ρ‹ Ρ€Π°Π²Π½Π°Β 0,2 ΠΌΠΌ, всСго ΠΆΠΈΠ»Β 15. Π£ΠΌΠ½ΠΎΠΆΠ°Π΅ΠΌ:Β 15*0,2=3ΠΌΠΌ. Но Π±Ρ‹Π²Π°Π΅Ρ‚, Ρ‡Ρ‚ΠΎ ΠΆΠΈΠ»Ρ‹ Π² ΠΊΠ°Π±Π΅Π»Π΅ Π½Π΅ ΠΏΡ€ΠΈΠ»Π΅Π³Π°ΡŽΡ‚ Π²ΠΏΠ»ΠΎΡ‚Π½ΡƒΡŽ, ΠΈ ΠΌΠ΅ΠΆΠ΄Ρƒ Π½ΠΈΠΌΠΈ образуСтся Π·Π°Π·ΠΎΡ€, ΠΊΠΎΡ‚ΠΎΡ€Ρ‹ΠΉ Π½ΡƒΠΆΠ½ΠΎ ΡƒΡ‡Π΅ΡΡ‚ΡŒ. Если Ρ‚Π°ΠΊΠΎΠΉ Π·Π°Π·ΠΎΡ€ имССтся, Ρ€Π΅Π·ΡƒΠ»ΡŒΡ‚Π°Ρ‚ умноТаСтся Π½Π°Β 0,9. Π’ΠΎΠ·ΡŒΠΌΠ΅ΠΌ наши значСния:Β 3*0,9=2,7.

Π—Π°ΠΊΠ»ΡŽΡ‡Π΅Π½ΠΈΠ΅

ΠŸΡ€Π°Π²Π΄Π°, Ρƒ этого ΠΌΠ΅Ρ‚ΠΎΠ΄Π° опрСдСлСния Π΅ΡΡ‚ΡŒ свои минусы — с Π΅Π³ΠΎ ΠΏΠΎΠΌΠΎΡ‰ΡŒΡŽ ΠΌΠΎΠΆΠ½ΠΎ ΡƒΠ·Π½Π°Ρ‚ΡŒ сСчСния Ρ‚ΠΎΠ»ΡŒΠΊΠΎ ΠΌΠ°Π»Π΅Π½ΡŒΠΊΠΈΡ… Ρ€Π°Π·ΠΌΠ΅Ρ€ΠΎΠ².Β Π Π°Π·Π²Π΅ Ρ€Π΅Π°Π»ΡŒΠ½Ρ‹ΠΌ Π±ΡƒΠ΄Π΅Ρ‚ Π½Π°ΠΌΠΎΡ‚Π°Ρ‚ΡŒ Π½Π° ΠΊΠ°Ρ€Π°Π½Π΄Π°Ρˆ ΠΏΡ€ΠΎΠ²ΠΎΠ΄ сСчСниСм 6 ΠΌΠΌ. ΠΊΠ².? Π’ΡƒΡ‚, ΠΊΠΎΠ½Π΅Ρ‡Π½ΠΎ, Π±Π΅Π· ΡΠΏΠ΅Ρ†ΠΈΠ°Π»ΡŒΠ½ΠΎΠ³ΠΎ ΠΏΡ€ΠΈΠ±ΠΎΡ€Π° Π½ΠΈΠΊΠ°ΠΊ Π½Π΅ ΠΎΠ±ΠΎΠΉΡ‚ΠΈΡΡŒ.Β Π‘ΡƒΡ‰Π΅ΡΡ‚Π²ΡƒΡŽΡ‚ ΡΠΏΠ΅Ρ†ΠΈΠ°Π»ΡŒΠ½Ρ‹Π΅ Ρ‚Π°Π±Π»ΠΈΡ†Ρ‹ с Π½ΠΎΠΌΠΈΠ½Π°Π»ΡŒΠ½Ρ‹ΠΌ Π·Π½Π°Ρ‡Π΅Π½ΠΈΠ΅ΠΌ сСчСния ΠΊΠ°Π±Π΅Π»Π΅ΠΉ, ΠΊΠ°ΠΊ ΠΎΠ΄Π½ΠΎΠΆΠΈΠ»ΡŒΠ½Ρ‹Ρ…, Ρ‚Π°ΠΊ ΠΈ ΠΌΠ½ΠΎΠ³ΠΎΠΆΠΈΠ»ΡŒΠ½Ρ‹Ρ…, ΠΈ ΡΠΎΠΎΡ‚Π²Π΅Ρ‚ΡΡ‚Π²ΡƒΡŽΡ‰ΠΈΠ΅ значСния Π΄ΠΈΠ°ΠΌΠ΅Ρ‚Ρ€ΠΎΠ². Но для Ρ‚ΠΎΠ³ΠΎ, Ρ‡Ρ‚ΠΎΠ±Ρ‹ ΠΏΡ€ΠΎΠ»ΠΎΠΆΠΈΡ‚ΡŒ ΠΏΡ€ΠΎΠ²ΠΎΠ΄ΠΊΡƒ Ρƒ сСбя Π² ΠΊΠ²Π°Ρ€Ρ‚ΠΈΡ€Π΅, способа с ΠΊΠ°Ρ€Π°Π½Π΄Π°ΡˆΠΎΠΌ Π±ΡƒΠ΄Π΅Ρ‚ Π²ΠΏΠΎΠ»Π½Π΅ Π±ΡƒΠ΄Π΅Ρ‚ достаточно.

Ворговая ΡΠ΅Ρ‚ΡŒ «ΠŸΠ»Π°Π½Π΅Ρ‚Π° Π­Π»Π΅ΠΊΡ‚Ρ€ΠΈΠΊΠ°» ΠΈΠΌΠ΅Π΅Ρ‚ большой ассортимСнт кабСльно-ΠΏΡ€ΠΎΠ²ΠΎΠ΄Π½ΠΈΠΊΠΎΠ²ΠΎΠΉ ΠΏΡ€ΠΎΠ΄ΡƒΠΊΡ†ΠΈΠΈ, с ΠΊΠΎΡ‚ΠΎΡ€Ρ‹ΠΌΒ ΠΌΠΎΠΆΠ½ΠΎ ΠΎΠ·Π½Π°ΠΊΠΎΠΌΠΈΡ‚ΡŒΡΡΒ Π² нашСм ΠΊΠ°Ρ‚Π°Π»ΠΎΠ³Π΅.

ΠžΡΠ½ΠΎΠ²Π½Ρ‹Π΅ особСнности расчСта кабСля ΠΏΠΎ Π΅Π³ΠΎ Π΄Π»ΠΈΠ½Π΅

Одним ΠΈΠ· самых Π²Π°ΠΆΠ½Ρ‹Ρ… ΠΊΡ€ΠΈΡ‚Π΅Ρ€ΠΈΠ΅Π² Π² процСссС Π²Ρ‹Π±ΠΎΡ€Π° кабСля, ΠΊΠΎΡ‚ΠΎΡ€Ρ‹ΠΉ обСспСчиваСт элСктропитаниС, являСтся ΠΎΠΏΡ€Π΅Π΄Π΅Π»Π΅Π½Π½ΠΎΠ΅ количСство Π²Π΅Π»ΠΈΡ‡ΠΈΠ½, ΠΏΠΎΡ‚ΠΎΠΌΡƒ стоит ΠΎΠ±Ρ€Π°Ρ‰Π°Ρ‚ΡŒ Π²Π½ΠΈΠΌΠ°Π½ΠΈΠ΅ Π½Π° Ρ‚Π°ΠΊΠΎΠΉ способ, как расчСт сСчСния кабСля ΠΏΠΎ Π½Π°Π³Ρ€ΡƒΠ·ΠΊΠ΅, Π° такТС расчСт ΠΏΠΎ ΡΠ΅Ρ‡Π΅Π½ΠΈΡŽ. Для Ρ‚ΠΎΠ³ΠΎ Ρ‡Ρ‚ΠΎΠ±Ρ‹ ΠΎΠ±Π΅ΡΠΏΠ΅Ρ‡ΠΈΡ‚ΡŒ высокий ΡƒΡ€ΠΎΠ²Π΅Π½ΡŒ бСзопасности ΠΈ ΠΏΡ€Π΅Π΄Π΅Π»ΡŒΠ½ΠΎΠΉ надСТности, ΠΎΡ‡Π΅Π½ΡŒ Π²Π°ΠΆΠ½ΠΎ ΠΎΠ±Ρ€Π°Ρ‚ΠΈΡ‚ΡŒ Π²Π½ΠΈΠΌΠ°Π½ΠΈΠ΅ Π½Π° Π΄Π»ΠΈΠ½Ρƒ ΠΊΠ°ΠΆΠ΄ΠΎΠ³ΠΎ ΠΈΠ· элСмСнтов Π»ΠΈΠ½ΠΈΠΈ, ΠΊΡ€ΠΎΠΌΠ΅ Ρ‚ΠΎΠ³ΠΎ, всСй Π»ΠΈΠ½ΠΈΠΈ Π² Ρ†Π΅Π»ΠΎΠΌ. Π‘Ρ‚ΠΎΠΈΡ‚ ΠΎΡ‚ΠΌΠ΅Ρ‚ΠΈΡ‚ΡŒ, Ρ‡Ρ‚ΠΎ практичСски всС соврСмСнныС приспособлСния Π² ΠΏΠ΅Ρ€Π²ΡƒΡŽ ΠΎΡ‡Π΅Ρ€Π΅Π΄ΡŒ рассчитаны Π½Π° ΠΊΠ°ΠΊΠΈΠ΅-Ρ‚ΠΎ ΠΎΠΏΡ€Π΅Π΄Π΅Π»Π΅Π½Π½Ρ‹Π΅ ΠΌΠ°ΠΊΡΠΈΠΌΠ°Π»ΡŒΠ½Ρ‹Π΅ значСния Ρ€Π°Π±ΠΎΡ‡Π΅Π³ΠΎ напряТСния, ΠΊΠΎΡ‚ΠΎΡ€ΠΎΠ΅ ΠΌΠΎΠΆΠ΅Ρ‚ Π±Ρ‹Ρ‚ΡŒ Ρ€Π°Π²Π½ΠΎ показатСлям ΠΎΡ‚ 185 Π΄ΠΎ 240 Π’ΠΎΠ»ΡŒΡ‚. ИмСнно ΠΏΠΎ этой ΠΏΡ€ΠΈΡ‡ΠΈΠ½Π΅, Ссли ΠΏΡ€ΠΈ расчСтС Π½Π΅ ΡƒΡ‡ΠΈΡ‚Ρ‹Π²Π°Ρ‚ΡŒ ΠΏΠΎΠΊΠ°Π·Π°Ρ‚Π΅Π»ΠΈ ΠΏΠΎΡ‚Π΅Ρ€ΠΈ напряТСния, ΠΊΠΎΡ‚ΠΎΡ€Ρ‹Π΅ связаны ΠΈΠΌΠ΅Π½Π½ΠΎ с Π΄Π»ΠΈΠ½ΠΎΠΉ кабСля, появляСтся большая Π²Π΅Ρ€ΠΎΡΡ‚Π½ΠΎΡΡ‚ΡŒ Ρ‚ΠΎΠ³ΠΎ, Ρ‡Ρ‚ΠΎ напряТСниС Π½Π° ΠΊΠΎΠ½Ρ†Π΅ Π»ΠΈΠ½ΠΈΠΈ Π±ΡƒΠ΄Π΅Ρ‚ Π·Π½Π°Ρ‡ΠΈΡ‚Π΅Π»ΡŒΠ½ΠΎ мСньшС, Ρ‡Π΅ΠΌ Ρ‚ΠΎ, Ρ‡Ρ‚ΠΎ трСбуСтся для обСспСчСния Π½ΠΎΡ€ΠΌΠ°Π»ΡŒΠ½ΠΎΠΉ Ρ€Π°Π±ΠΎΡ‚Ρ‹ всСх ΠΈΠΌΠ΅ΡŽΡ‰ΠΈΡ…ΡΡ устройств. Π’ свою ΠΎΡ‡Π΅Ρ€Π΅Π΄ΡŒ это ΠΌΠΎΠΆΠ΅Ρ‚ привСсти ΠΊ нСвозмоТности ΠΈΡ… эксплуатации ΠΈΠ»ΠΈ, Ρ‡Ρ‚ΠΎ Π΅Ρ‰Π΅ Π±ΠΎΠ»Π΅Π΅ нСприятно, ΠΌΠΎΠ³ΡƒΡ‚ Π²ΠΎΠΎΠ±Ρ‰Π΅ Π²Ρ‹ΠΉΡ‚ΠΈ ΠΈΠ· строя. Π’Π°ΠΊΠΈΠΌ ΠΎΠ±Ρ€Π°Π·ΠΎΠΌ, проводя ΠΏΠΎΠ΄ΠΎΠ±Π½Ρ‹Π΅ расчСты сСчСния кабСля ΠΏΠΎ показатСлям Π΄Π»ΠΈΠ½Ρ‹, ΠΌΠΎΠΆΠ½ΠΎ ΠΎΠ±Π΅ΡΠΏΠ΅Ρ‡ΠΈΡ‚ΡŒ Π±Π΅Π·ΠΎΠΏΠ°ΡΠ½ΠΎΡΡ‚ΡŒ ΠΈ ΠΊΠ°Ρ‡Π΅ΡΡ‚Π²Π΅Π½Π½ΡƒΡŽ Ρ€Π°Π±ΠΎΡ‚Ρƒ всСй систСмы Π² Ρ†Π΅Π»ΠΎΠΌ.

РасчСт сСчСния кабСля ΠΏΠΎ Π΄Π»ΠΈΠ½Π΅ Π² Π±Ρ‹Ρ‚Ρƒ

ΠŸΡ€Π΅ΠΆΠ΄Π΅ всСго, ΠΏΠΎΠ΄ΠΎΠ±Π½Ρ‹ΠΉ ΠΌΠ΅Ρ‚ΠΎΠ΄ идСально ΠΏΠΎΠ΄ΠΎΠΉΠ΄Π΅Ρ‚ Π² Π±Ρ‹Ρ‚Ρƒ. Как ΠΏΡ€Π°Π²ΠΈΠ»ΠΎ, Ρ‚Π°ΠΊΠΎΠΉ расчСт Π² Π΄Π°Π½Π½Ρ‹Ρ… условиях Π½Π΅ΠΎΠ±Ρ…ΠΎΠ΄ΠΈΠΌ Π² процСссС изготовлСния ΡƒΠ΄Π»ΠΈΠ½ΠΈΡ‚Π΅Π»Π΅ΠΉ, ΠΊΠΎΡ‚ΠΎΡ€Ρ‹Π΅ рассчитаны Π½Π° достаточно большиС расстояния. Π§Ρ‚ΠΎ касаСтся ΠΎΡΡ‚Π°Π»ΡŒΠ½Ρ‹Ρ… случаСв, Ρ‚ΠΎ ΠΏΡ€ΠΈ ΠΏΡ€ΠΎΠΊΠ»Π°Π΄ΠΊΠ΅ кабСля Π² Π±Ρ‹Ρ‚ΠΎΠ²Ρ‹Ρ… условиях ΠΏΠΎΠ΄ΠΎΠ±Π½Ρ‹Π΅ слоТныС расчСты Π½Π΅ Ρ‚Ρ€Π΅Π±ΡƒΡŽΡ‚ΡΡ. Π­Ρ‚ΠΎ основано Π½Π° Ρ‚ΠΎΠΌ, Ρ‡Ρ‚ΠΎ Π΄Π»ΠΈΠ½Π° Π»ΠΈΠ½ΠΈΠΉ Π² Π±Ρ‹Ρ‚Ρƒ отличаСтся ΠΎΡ‚Π½ΠΎΡΠΈΡ‚Π΅Π»ΡŒΠ½ΠΎ нСбольшой Π΄Π»ΠΈΠ½ΠΎΠΉ, ΠΏΠΎΡ‚ΠΎΠΌΡƒ всС ΠΏΠΎΡ‚Π΅Ρ€ΠΈ напряТСния Π½Π°ΡΡ‚ΠΎΠ»ΡŒΠΊΠΎ ΠΌΠ°Π»Ρ‹, Ρ‡Ρ‚ΠΎ ΠΈΠΌΠΈ Π²ΠΏΠΎΠ»Π½Π΅ ΠΌΠΎΠΆΠ½ΠΎ ΠΏΡ€Π΅Π½Π΅Π±Ρ€Π΅Ρ‡ΡŒ. НСсмотря Π½Π° это, Π² процСссС ΠΏΡ€ΠΎΠΊΠ»Π°Π΄ΠΊΠΈ Π»ΠΈΠ½ΠΈΠΈ всСгда слСдуСт ΠΎΡΡ‚Π°Π²Π»ΡΡ‚ΡŒ ΠΎΠΏΡ€Π΅Π΄Π΅Π»Π΅Π½Π½Ρ‹ΠΉ запас, Ρ€Π°Π²Π½Ρ‹ΠΉ ΠΏΡ€ΠΈΠΌΠ΅Ρ€Π½ΠΎ 15 см, ΠΏΡ€ΠΈΡ‡Π΅ΠΌ ΠΎΡΡ‚Π°Π²Π»ΡΡ‚ΡŒ Π΅Π³ΠΎ трСбуСтся с ΠΊΠ°ΠΆΠ΄ΠΎΠΉ стороны Π½Π° ΠΏΡ€ΠΎΠ²Π΅Π΄Π΅Π½ΠΈΠ΅ Ρ‚Π°ΠΊΠΈΡ… процСссов, ΠΊΠ°ΠΊ коммутация ΠΏΡ€ΠΎΠ²ΠΎΠ΄ΠΎΠ², ΠΈΡ… ΠΏΠΎΠ΄ΠΊΠ»ΡŽΡ‡Π΅Π½ΠΈΠ΅, Π³Π΄Π΅ осущСствляСтся Ρ‚Π°ΠΊΠΎΠΉ процСсс, ΠΊΠ°ΠΊΒ ΠΏΠ°ΠΉΠΊΠ°, сварка ΠΈΠ»ΠΈ ΠΎΠ±ΠΆΠΈΠΌ. Π§Ρ‚ΠΎ касаСтся ΠΊΠΎΠ½Ρ†ΠΎΠ² ΠΊΠ°Π±Π΅Π»Π΅ΠΉ, Ρ‚ΠΎ Ρ‚Π΅, ΠΊΠΎΡ‚ΠΎΡ€Ρ‹Π΅ входят Π² Ρ‰ΠΈΡ‚ΠΎΠΊ, Π΄ΠΎΠ»ΠΆΠ½Ρ‹ ΠΈΠΌΠ΅Ρ‚ΡŒ Π΅Ρ‰Π΅ больший запас для ΠΏΠΎΠ΄ΠΊΠ»ΡŽΡ‡Π΅Π½ΠΈΡ Π·Π°Ρ‰ΠΈΡ‚Π½ΠΎΠΉ Π°Π²Ρ‚ΠΎΠΌΠ°Ρ‚ΠΈΠΊΠΈ ΠΈ достаточно Π°ΠΊΠΊΡƒΡ€Π°Ρ‚Π½ΠΎΠΉ ΡƒΠΊΠ»Π°Π΄ΠΊΠΈ.

Говоря ΠΈΠ½Ρ‹ΠΌΠΈ словами, Π² Π±Ρ‹Ρ‚ΠΎΠ²Ρ‹Ρ… условиях Π½Π° Ρ‚ΠΎΠΉ повСрхности, Π³Π΄Π΅ планируСтся ΠΏΡ€ΠΎΠΊΠ»Π°Π΄Ρ‹Π²Π°Ρ‚ΡŒ кабСль, ΠΏΡ€Π΅ΠΆΠ΄Π΅ всСго, стоит ΠΏΡ€ΠΎΡΡ‚Π°Π²ΠΈΡ‚ΡŒ ΠΎΠΏΡ€Π΅Π΄Π΅Π»Π΅Π½Π½Ρ‹Π΅ ΠΎΡ‚ΠΌΠ΅Ρ‚ΠΊΠΈ мСст располоТСния Ρ€ΠΎΠ·Π΅Ρ‚ΠΎΠΊ, Π²Ρ‹ΠΊΠ»ΡŽΡ‡Π°Ρ‚Π΅Π»Π΅ΠΉ, элСктропотрСбитСлСй, ΠΊΠΎΠΌΠΌΡƒΡ‚Π°Ρ†ΠΈΠΎΠ½Π½Ρ‹Ρ… ΠΊΠΎΡ€ΠΎΠ±ΠΎΠΊ ΠΈ ΠΈΠ½Ρ‹Ρ… ΠΏΠΎΠ΄ΠΎΠ±Π½Ρ‹Ρ… приспособлСний. ПослС этого Ρ€ΡƒΠ»Π΅Ρ‚ΠΊΠΎΠΉ осущСствляСтся Π·Π°ΠΌΠ΅Ρ€ расстояния ΠΈ отрСзаСтся кабСль, Π½ΠΎ с нСбольшим запасом. По ΠΎΠΊΠΎΠ½Ρ‡Π°Π½ΠΈΠΈ Π΄Π°Π½Π½Ρ‹Ρ… Ρ€Π°Π±ΠΎΡ‚ крСпится нСпосрСдствСнно сам кабСль ΠΊ повСрхности, Π½ΠΎ Π² строго соотвСтствии со всСми трСбованиями ПУЭ.

МногиС ΠΌΠΎΠ½Ρ‚Π°ΠΆΠ½ΠΈΠΊΠΈ, ΠΈΠΌΠ΅ΡŽΡ‰ΠΈΠ΅ большой ΠΎΠΏΡ‹Ρ‚ Ρ€Π°Π±ΠΎΡ‚Ρ‹ Π² Π΄Π°Π½Π½ΠΎΠΉ сфСрС Π΄Π΅ΡΡ‚Π΅Π»ΡŒΠ½ΠΎΡΡ‚ΠΈ, Π° Ρ‚Π°ΠΊΠΆΠ΅ Ρ‚Π΅, ΠΊΠΎΡ‚ΠΎΡ€Ρ‹Π΅ ΠΈΠΌΠ΅ΡŽΡ‚ Π½Π°ΠΏΠ°Ρ€Π½ΠΈΠΊΠ°, ΠΏΠΎΡΡ‚ΡƒΠΏΠ°ΡŽΡ‚ Π΅Ρ‰Π΅ Π±ΠΎΠ»Π΅Π΅ просто, Ρ‡Ρ‚ΠΎ позволяСт ΠΈΠΌ ΡΡΠΊΠΎΠ½ΠΎΠΌΠΈΡ‚ΡŒ Π½Π΅ΠΌΠ°Π»ΠΎΠ΅ количСство Π²Ρ€Π΅ΠΌΠ΅Π½ΠΈ. Π’ самом Π½Π°Ρ‡Π°Π»Π΅ производится Ρ€Π°Π·ΠΌΠ΅Ρ‚ΠΊΠ° располоТСния Ρ‚Π°ΠΊΠΈΡ… устройств, ΠΊΠ°ΠΊ ΠΊΠΎΠΌΠΌΡƒΡ‚Π°Ρ†ΠΈΠΎΠ½Π½Ρ‹Π΅ ΠΊΠΎΡ€ΠΎΠ±ΠΎΡ‡ΠΊΠΈ, Π²Ρ‹ΠΊΠ»ΡŽΡ‡Π°Ρ‚Π΅Π»ΠΈ ΠΈ Ρ€ΠΎΠ·Π΅Ρ‚ΠΊΠΈ. Π—Π°Ρ‚Π΅ΠΌ, Π±Π΅Π· ΠΏΡ€Π΅Π΄Π²Π°Ρ€ΠΈΡ‚Π΅Π»ΡŒΠ½ΠΎΠ³ΠΎ Π·Π°ΠΌΠ΅Ρ€Π° осущСствляСтся ΠΏΡ€ΠΎΠΊΠ»Π°Π΄ΠΊΠ° ΠΈ ΠΊΡ€Π΅ΠΏΠ»Π΅Π½ΠΈΠ΅ кабСля, Π½ΠΎ с запасом, послС Ρ‡Π΅Π³ΠΎ отрСзаСтся.

РасчСт сСчСния кабСля ΠΏΠΎ Π΄Π»ΠΈΠ½Π΅ Π² ΠΏΡ€ΠΎΠΌΡ‹ΡˆΠ»Π΅Π½Π½ΠΎΡΡ‚ΠΈ

Π§Ρ‚ΠΎ касаСтся области ΠΏΡ€ΠΎΠΌΡ‹ΡˆΠ»Π΅Π½Π½ΠΎΡΡ‚ΠΈ, Ρ‚ΠΎ здСсь трСбуСмый расчСт сСчСния кабСля ΠΏΠΎ длинС осущСствляСтся ΡƒΠΆΠ΅ Π½Π° этапС проСктирования элСктричСских сСтСй. ΠŸΠΎΠ΄ΠΎΠ±Π½Ρ‹Π΅ расчСты Π²Π°ΠΆΠ½ΠΎ ΡΠ΄Π΅Π»Π°Ρ‚ΡŒ Π² Ρ‚ΠΎΠΌ случаС, Ссли Π½Π° кабСль Π±ΡƒΠ΄ΡƒΡ‚ Π²ΠΎΠ·Π»ΠΎΠΆΠ΅Π½Ρ‹ Π΄ΠΎΠ»Π³ΠΎΠ²Ρ€Π΅ΠΌΠ΅Π½Π½Ρ‹Π΅ ΠΈ достаточно ΡΠ΅Ρ€ΡŒΠ΅Π·Π½Ρ‹Π΅ Π½Π°Π³Ρ€ΡƒΠ·ΠΊΠΈ.

ΠŸΡ€Π°ΠΊΡ‚ΠΈΡ‡Π΅ΡΠΊΠΈ всС ΠΏΡ€ΠΎΠ²ΠΎΠ΄Π½ΠΈΠΊΠΈ ΠΏΠΎ ΠΏΡ€ΠΈΡ‡ΠΈΠ½Π΅ своих свойств, ΠΎΠ±Π»Π°Π΄Π°ΡŽΡ‚ ΠΎΠΏΡ€Π΅Π΄Π΅Π»Π΅Π½Π½ΠΎΠΉ Π²Π΅Π»ΠΈΡ‡ΠΈΠ½ΠΎΠΉ элСктричСского сопротивлСния, ΠΊΠΎΡ‚ΠΎΡ€ΠΎΠ΅ ΠΌΠΎΠΆΠ΅Ρ‚ Π²Ρ‹Π·Π²Π°Ρ‚ΡŒ ΠΏΠΎΡ‚Π΅Ρ€ΠΈ Π² процСссС прохоТдСния ΠΏΠΎ ΠΏΡ€ΠΎΠ²ΠΎΠ΄Π°ΠΌ элСктричСского Ρ‚ΠΎΠΊΠ°. Π‘Ρ‚ΠΎΠΈΡ‚ ΠΎΡ‚ΠΌΠ΅Ρ‚ΠΈΡ‚ΡŒ Ρ‚Π°ΠΊΠΈΠ΅ Ρ„Π°ΠΊΡ‚ΠΎΡ€Ρ‹, Π²Π»ΠΈΡΡŽΡ‰ΠΈΠ΅ Π½Π° ΠΏΠ°Ρ€Π°ΠΌΠ΅Ρ‚Ρ€Ρ‹ Π²Π΅Π»ΠΈΡ‡ΠΈΠ½Ρ‹ ΠΏΠΎΡ‚Π΅Ρ€ΡŒ ΠΈ сопротивлСния, ΠΊΠ°ΠΊ ΠΌΠ°Ρ‚Π΅Ρ€ΠΈΠ°Π», ΠΈΠ· ΠΊΠΎΡ‚ΠΎΡ€ΠΎΠ³ΠΎ Π²Ρ‹ΠΏΠΎΠ»Π½Π΅Π½ ΠΏΡ€ΠΎΠ²ΠΎΠ΄Π½ΠΈΠΊ, Ρ‚ΠΎ Π΅ΡΡ‚ΡŒ алюминий ΠΈ мСдь, ΠΈΠΌΠ΅Π΅Ρ‚ Π·Π½Π°Ρ‡Π΅Π½ΠΈΠ΅ сСчСниС ΠΏΡ€ΠΎΠ²ΠΎΠ΄Π½ΠΈΠΊΠ°, ΠΊΠ°ΠΊ ΠΏΡ€Π°Π²ΠΈΠ»ΠΎ, Ρ‡Π΅ΠΌ мСньшС сСчСниС, Ρ‚Π΅ΠΌ ΠΏΠΎΡ‚Π΅Ρ€ΠΈ большС. ΠšΡ€ΠΎΠΌΠ΅ Ρ‚ΠΎΠ³ΠΎ, Π²Π°ΠΆΠ½Π° Π΄Π»ΠΈΠ½Π° ΠΏΡ€ΠΎΠ²ΠΎΠ΄Π½ΠΈΠΊΠ°, Ρ‚ΠΎ Π΅ΡΡ‚ΡŒ Ρ‡Π΅ΠΌ большС Π΄Π°Π½Π½Ρ‹ΠΉ ΠΏΠ°Ρ€Π°ΠΌΠ΅Ρ‚Ρ€, Ρ‚Π΅ΠΌ соотвСтствСнно большС ΠΈ ΠΏΠΎΡ‚Π΅Ρ€ΠΈ.

На основании всСх Π²Ρ‹ΡˆΠ΅ΠΏΠ΅Ρ€Π΅Ρ‡ΠΈΡΠ»Π΅Π½Π½Ρ‹Ρ… Ρ„Π°ΠΊΡ‚ΠΎΡ€ΠΎΠ² становится ясно, ΠΏΠΎ ΠΊΠ°ΠΊΠΎΠΉ ΠΏΡ€ΠΈΡ‡ΠΈΠ½Π΅ Π² ΠΏΡ€ΠΎΠ²ΠΎΠ΄Π½ΠΈΠΊΠ°Ρ… присутствуСт явлСниС Π½Π΅ΠΊΠΎΡ‚ΠΎΡ€ΠΎΠ³ΠΎ падСния напряТСния, ΠΊΠΎΡ‚ΠΎΡ€ΠΎΠ΅, ΠΊΠ°ΠΊ ΠΏΡ€Π°Π²ΠΈΠ»ΠΎ, Ρ€Π°Π²Π½ΠΎ Π²Π΅Π»ΠΈΡ‡ΠΈΠ½Π΅ Ρ‚ΠΎΠΊΠ°, ΡƒΠΌΠ½ΠΎΠΆΠ΅Π½Π½ΠΎΠ³ΠΎ Π½Π° ΠΏΠΎΠΊΠ°Π·Π°Ρ‚Π΅Π»ΠΈ сопротивлСния ΠΏΡ€ΠΎΠ²ΠΎΠ΄Π½ΠΈΠΊΠ°. Богласно установлСнным ΠΏΡ€Π°Π²ΠΈΠ»Π°ΠΌ, ΠΏΡ€ΠΈΠΌΠ΅Ρ€Π½ΠΎΠ΅ Π·Π½Π°Ρ‡Π΅Π½ΠΈΠ΅ падСния ΠΏΠΎΠΊΠ°Π·Π°Ρ‚Π΅Π»Π΅ΠΉ напряТСния Π΄ΠΎΠ»ΠΆΠ½ΠΎ Π±Ρ‹Ρ‚ΡŒ Ρ€Π°Π²Π½ΠΎ 5%. Если Π΄Π°Π½Π½Ρ‹ΠΉ ΠΏΠ°Ρ€Π°ΠΌΠ΅Ρ‚Ρ€ Π½Π΅ΠΌΠ½ΠΎΠ³ΠΎ Π²Ρ‹ΡˆΠ΅, ΠΏΡ€ΠΎΠ²ΠΎΠ΄Π½ΠΈΠΊ слСдуСт ΠΏΠΎΠ΄ΠΎΠ±Ρ€Π°Ρ‚ΡŒ с большим сСчСниСм.

Как осущСствляСтся расчСт сСчСния кабСля ΠΏΠΎ Π΄Π»ΠΈΠ½Π΅

Для осущСствлСния ΠΏΠΎΠ΄ΠΎΠ±Π½Ρ‹Ρ… расчСтов, ΠΊΠ°ΠΊ ΠΏΡ€Π°Π²ΠΈΠ»ΠΎ, ΠΈΡΠΏΠΎΠ»ΡŒΠ·ΡƒΠ΅Ρ‚ΡΡ ΡΠΏΠ΅Ρ†ΠΈΠ°Π»ΡŒΠ½Π°Ρ Ρ„ΠΎΡ€ΠΌΡƒΠ»Π°. Π’ Π½Π΅ΠΉ ΡΠΎΠ΄Π΅Ρ€ΠΆΠ°Ρ‚ΡŒΡΡ ΠΏΠΎΠΊΠ°Π·Π°Ρ‚Π΅Π»ΠΈ Π΄Π»ΠΈΠ½Ρ‹, ΡƒΠ΄Π΅Π»ΡŒΠ½ΠΎΠ΅ сопротивлСниС самого ΠΏΡ€ΠΎΠ²ΠΎΠ΄Π½ΠΈΠΊΠ°, ΠΏΠ»ΠΎΡ‰Π°Π΄ΡŒ сСчСния. ΠŸΡ€ΠΈ этом сопротивлСниС опрСдСляСтся ΠΏΠΎ ΡΠΏΠ΅Ρ†ΠΈΠ°Π»ΡŒΠ½Ρ‹ΠΌ справочным Ρ‚Π°Π±Π»ΠΈΡ†Π°ΠΌ, ΠΏΡ€ΠΈ этом ΠΌΠΎΠΆΠ½ΠΎ ΡƒΠ±Π΅Π΄ΠΈΡ‚ΡŒΡΡ Π² Ρ‚ΠΎΠΌ, Ρ‡Ρ‚ΠΎ ΠΌΠ½ΠΎΠ³ΠΎ здСсь зависит ΠΎΡ‚ ΠΌΠ°Ρ€ΠΊΠΈ ΠΏΡ€ΠΎΠ²ΠΎΠ΄Π° ΠΈ самого кабСля. ПослС опрСдСлСния всСх Π½Π΅ΠΎΠ±Ρ…ΠΎΠ΄ΠΈΠΌΡ‹Ρ… ΡΠΎΡΡ‚Π°Π²Π»ΡΡŽΡ‰ΠΈΡ…, ΠΎΠΏΡ€Π΅Π΄Π΅Π»ΡΡŽΡ‚ΡΡ особыС расчСтныС значСния Ρ‚ΠΎΠΊΠ°. Для этой Ρ†Π΅Π»ΠΈ суммарная ΠΌΠΎΡ‰Π½ΠΎΡΡ‚ΡŒ Π½Π°Π³Ρ€ΡƒΠ·ΠΊΠΈ раздСляСтся Π½Π° Π²Π΅Π»ΠΈΡ‡ΠΈΠ½Ρƒ ΠΏΠΎΠΊΠ°Π·Π°Ρ‚Π΅Π»Π΅ΠΉ напряТСния Π² сСти. По ΡΠΏΠ΅Ρ†ΠΈΠ°Π»ΡŒΠ½ΠΎΠΉ справочной Ρ„ΠΎΡ€ΠΌΡƒΠ»Π΅ рассчитываСтся Π²Π΅Π»ΠΈΡ‡ΠΈΠ½Π° падСния Π² сСти ΠΈΠ»ΠΈ Π² Π»ΠΈΠ½ΠΈΠΈ напряТСния. ΠžΡ†Π΅Π½ΠΊΠ° Π²Π΅Π»ΠΈΡ‡ΠΈΠ½Ρ‹ ΡΠΎΠΎΡ‚Π½ΠΎΡˆΠ΅Π½ΠΈΡ Π² ΠΏΡ€ΠΎΡ†Π΅Π½Ρ‚Π°Ρ… ΠΊ Π·Π½Π°Ρ‡Π΅Π½ΠΈΡŽ ΠΈΠ·Π½Π°Ρ‡Π°Π»ΡŒΠ½ΠΎΠ³ΠΎ напряТСния, Π° Ρ‚Π°ΠΊΠΆΠ΅ Π²Ρ‹Π±ΠΎΡ€ ΠΎΠΏΡ‚ΠΈΠΌΠ°Π»ΡŒΠ½ΠΎΠ³ΠΎ сСчСния ΠΏΡ€ΠΎΠ²ΠΎΠ΄Π½ΠΈΠΊΠ°, ΠΊΠΎΡ‚ΠΎΡ€Ρ‹ΠΉ Π΄ΠΎΠ»ΠΆΠ΅Π½ ΡƒΠΊΠ»Π°Π΄Ρ‹Π²Π°Ρ‚ΡŒΡΡ Π² пятипроцСнтный Π±Π°Ρ€ΡŒΠ΅Ρ€.

Π’Π°ΠΆΠ½ΠΎ ΠΎΠ±Ρ€Π°Ρ‚ΠΈΡ‚ΡŒ Π²Π½ΠΈΠΌΠ°Π½ΠΈΠ΅, Ρ‡Ρ‚ΠΎ для ΠΏΡ€ΠΎΠΌΡ‹ΡˆΠ»Π΅Π½Π½Ρ‹Ρ… ΠΈ ΠΈΠ½Ρ‹Ρ… прСдприятий со срСдним ΠΈ ΠΊΡ€ΡƒΠΏΠ½Ρ‹ΠΌ Ρ‚ΠΎΠ²Π°Ρ€ΠΎΠΎΠ±ΠΎΡ€ΠΎΡ‚ΠΎΠΌ, рСкомСндуСтся ΠΏΡ€ΠΎΠΈΠ·Π²ΠΎΠ΄ΠΈΡ‚ΡŒ ΡΠΏΠ΅Ρ†ΠΈΠ°Π»ΡŒΠ½Ρ‹ΠΉ комплСксный расчСт, Π² процСссС ΠΊΠΎΡ‚ΠΎΡ€ΠΎΠ³ΠΎ ΡƒΡ‡ΠΈΡ‚Ρ‹Π²Π°ΡŽΡ‚ΡΡ всС Π½Π΅ΠΎΠ±Ρ…ΠΎΠ΄ΠΈΠΌΡ‹Π΅ трСбования для Ρ‚Π΅Ρ… ΠΈΠ»ΠΈ ΠΈΠ½Ρ‹Ρ… ΠΊΠΎΠ½ΠΊΡ€Π΅Ρ‚Π½Ρ‹Ρ… условий эксплуатации. Для провСдСния ΠΏΠΎΠ΄ΠΎΠ±Π½Ρ‹Ρ… расчСтов ΠΌΠΎΠΆΠ½ΠΎ ΠΎΠ±Ρ€Π°Ρ‚ΠΈΡ‚ΡŒΡΡ Π·Π° ΠΏΠΎΠΌΠΎΡ‰ΡŒΡŽ ΠΊ спСциалистам, ΠΊΠΎΡ‚ΠΎΡ€Ρ‹Π΅ Π½Π° самом высоком ΠΏΡ€ΠΎΡ„Π΅ΡΡΠΈΠΎΠ½Π°Π»ΡŒΠ½ΠΎΠΌ ΡƒΡ€ΠΎΠ²Π½Π΅, с ΠΎΠΏΡ€Π΅Π΄Π΅Π»Π΅Π½Π½Ρ‹ΠΌΠΈ гарантиями обСспСчСния работоспособности сСти Π² процСссС Ρ€Π°Π±ΠΎΡ‡ΠΈΡ… Π½Π°Π³Ρ€ΡƒΠ·ΠΎΠΊ ΠΏΡ€ΠΎΠΈΠ·Π²Π΅Π΄ΡƒΡ‚ всС расчСты. ΠšΡ€ΠΎΠΌΠ΅ Ρ‚ΠΎΠ³ΠΎ, Π±ΡƒΠ΄ΡƒΡ‚ Π²Ρ‹ΠΏΠΎΠ»Π½Π΅Π½Ρ‹ расчСты, ΠΊΠΎΡ‚ΠΎΡ€Ρ‹Π΅ обСспСчат ΠΌΠΈΠ½ΠΈΠΌΠ°Π»ΡŒΠ½Ρ‹Π΅ Π·Π°Ρ‚Ρ€Π°Ρ‚Ρ‹, Ссли Π΅ΡΡ‚ΡŒ Π½Π΅ΠΎΠ±Ρ…ΠΎΠ΄ΠΈΠΌΠΎΡΡ‚ΡŒ произвСсти Π½Π°Ρ€Π°Ρ‰ΠΈΠ²Π°Π½ΠΈΠ΅ производствСнной мощности.

ΠŸΡ€ΠΈΠΌΠ΅Ρ€ расчСта Π±Ρ‹Ρ‚ΠΎΠ²ΠΎΠΉ сфСрС

Если послС осущСствлСния подсчСта суммарной мощности ΠΏΠΎΡ‚Ρ€Π΅Π±ΠΈΡ‚Π΅Π»Π΅ΠΉ Π±Ρ‹Π»ΠΎ ΠΏΠΎΠ»ΡƒΡ‡Π΅Π½ΠΎ 3,8 ΠΊΠ’Ρ‚, находится сила Ρ‚ΠΎΠΊΠ° ΠΏΠΎ Ρ‚Π°ΠΊΠΎΠΉ Ρ„ΠΎΡ€ΠΌΡƒΠ»Π΅ —Β I = P/UΒ·cosΟ†.Β Π—Π΄Π΅ΡΡŒΒ P – прСдставляСт собой ΡΡƒΠΌΠΌΠ°Ρ€Π½ΡƒΡŽ ΠΌΠΎΡ‰Π½ΠΎΡΡ‚ΡŒ, (Π’Ρ‚),Β IΒ — это сила Ρ‚ΠΎΠΊΠ°, (А),Β cosΟ† – коэффициСнт, ΠΊΠΎΡ‚ΠΎΡ€Ρ‹ΠΉ Ρ€Π°Π²Π΅Π½ 1, Π½ΠΎ Ρ‚ΠΎΠ»ΡŒΠΊΠΎ Ссли сСти Π±Ρ‹Ρ‚ΠΎΠ²Ρ‹Π΅, Π° Ρ‚Π°ΠΊΠΆΠ΅Β UΒ — напряТСниС Π² сСти, (Π’).

Π’ Π΄Π°Π½Π½ΠΎΠΌ случаС, Ссли 3,8 ΠΊΠ’Ρ‚ Ρ€Π°Π·Π΄Π΅Π»ΠΈΡ‚ΡŒ Π½Π° напряТСниС 220 Π’, получится число, Ρ€Π°Π²Π½ΠΎΠ΅ 17,3 А. ΠŸΡ€ΠΈΠΌΠ΅Π½ΡΡ ΡΠΏΠ΅Ρ†ΠΈΠ°Π»ΡŒΠ½Ρ‹Π΅ Ρ‚Π°Π±Π»ΠΈΡ†Ρ‹ ПУЭ ΠΏΠΎΠ΄ Π½ΠΎΠΌΠ΅Ρ€Π°ΠΌΠΈ 1.3.4 ΠΈ 1.3.5 опрСдСляСтся Π½Π΅ΠΎΠ±Ρ…ΠΎΠ΄ΠΈΠΌΠΎΠ΅ сСчСниС ΠΌΠ΅Π΄Π½ΠΎΠ³ΠΎ кабСля ΠΈΠ»ΠΈ Π²Ρ‹ΠΏΠΎΠ»Π½Π΅Π½Π½ΠΎΠ΅ ΠΈΠ· алюминия. Π§Ρ‚ΠΎ касаСтся ΠΌΠ°Ρ‚Π΅Ρ€ΠΈΠ°Π»Π°, Ρ‚ΠΎ Π² Π±Ρ‹Ρ‚Ρƒ рСкомСндуСтся ΠΈΡΠΏΠΎΠ»ΡŒΠ·ΠΎΠ²Π°Ρ‚ΡŒ ΠΈΠΌΠ΅Π½Π½ΠΎ мСдь, ΠΏΠΎΡ‚ΠΎΠΌΡƒ ΠΏΡ€ΠΈ ΠΏΠΎΠ»ΡƒΡ‡Π΅Π½Π½Ρ‹Ρ… показатСлях силы Ρ‚ΠΎΠΊΠ° потрСбуСтся кабСль ΠΈΠ· ΠΌΠ΅Π΄ΠΈ с сСчСниСм 1,5 ΠΊΠ². ΠΌΠΌ.

ПослС этого, ΠΊΠ°ΠΊ ΠΏΡ€Π°Π²ΠΈΠ»ΠΎ, рассчитываСтся ΠΏΠΎΠΊΠ°Π·Π°Ρ‚Π΅Π»ΡŒ сопротивлСния, ΠΏΠΎ Ρ„ΠΎΡ€ΠΌΡƒΠ»Π΅Β R = pΒ·L/S,Β Π³Π΄Π΅Β RΒ — это сопротивлСниС ΠΏΡ€ΠΎΠ²ΠΎΠ΄Π°, (Ом), ΡƒΠΊΠ°Π·Π°Ρ‚Π΅Π»ΡŒΒ pΒ  прСдставляСт собой Π·Π½Π°Ρ‡Π΅Π½ΠΈΠ΅ ΡƒΠ΄Π΅Π»ΡŒΠ½ΠΎΠ³ΠΎ сопротивлСния, (Ом·мм2/ΠΌ),Β L – это ΠΏΠ°Ρ€Π°ΠΌΠ΅Ρ‚Ρ€ Π΄Π»ΠΈΠ½Ρ‹ ΠΏΡ€ΠΎΠ²ΠΎΠ΄Π° ΠΈΠ»ΠΈ кабСля, (ΠΌ), Π°Β SΒ — ΠΏΠ»ΠΎΡ‰Π°Π΄ΡŒ ΠΏΠΎΠΏΠ΅Ρ€Π΅Ρ‡Π½ΠΎΠ³ΠΎ сСчСния, ΠΊΠΎΡ‚ΠΎΡ€Ρ‹ΠΉ выраТаСтся Π² ΠΌΠΌ2. Π‘Ρ‚ΠΎΠΈΡ‚ ΠΎΡ‚ΠΌΠ΅Ρ‚ΠΈΡ‚ΡŒ, Ρ‡Ρ‚ΠΎ ΡƒΠ΄Π΅Π»ΡŒΠ½ΠΎΠ΅ сопротивлСниС Π  – это постоянная Π²Π΅Π»ΠΈΡ‡ΠΈΠ½Π°, которая прямо зависит ΠΎΡ‚ ΠΌΠ°Ρ‚Π΅Ρ€ΠΈΠ°Π»Π°. Если это мСдь, Ρ‚ΠΎ ΡƒΠ΄Π΅Π»ΡŒΠ½ΠΎΠ΅ сопротивлСниС Ρ€Π°Π²Π½ΠΎ 0,0175, Ссли алюминий, Ρ‚ΠΎ ΠΎΠ½ Ρ€Π°Π²Π΅Π½ 0,0281. На основании ΠΏΡ€ΠΎΠ²Π΅Π΄Π΅Π½Π½Ρ‹Ρ… расчСтов для ΠΎΠ΄Π½ΠΎΠΉ ΠΆΠΈΠ»Ρ‹ Π² ΠΊΠ°Π±Π΅Π»Π΅, Π΄Π»ΠΈΠ½Π° ΠΊΠΎΡ‚ΠΎΡ€ΠΎΠ³ΠΎ составляСт 20 ΠΌ, получаСтся R = 0,0175Β·20/1,5 = 0,232 Ом. По Ρ‚ΠΎΠΉ ΠΏΡ€ΠΈΡ‡ΠΈΠ½Π΅, Ρ‡Ρ‚ΠΎ Ρ‚ΠΎΠΊ ΠΏΡ€ΠΎΡ…ΠΎΠ΄ΠΈΡ‚ Ρ‚ΠΎΠ»ΡŒΠΊΠΎ ΠΏΠΎ ΠΎΠ΄Π½ΠΎΠΉ ΠΆΠΈΠ»Π΅, Π° ΠΏΠΎ Π΄Ρ€ΡƒΠ³ΠΎΠΉ возвращаСтся, ΠΏΠ°Ρ€Π°ΠΌΠ΅Ρ‚Ρ€ Π΄Π»ΠΈΠ½Ρ‹ удваиваСтся, Ρ‚ΠΎ Π΅ΡΡ‚ΡŒ получаСтся RΠΎΠ±Ρ‰Β = 0,464 Ом.

ΠŸΡ€ΠΈ нСобходимости Ρ€Π°ΡΡΡ‡ΠΈΡ‚Π°Ρ‚ΡŒ ΠΏΠΎΡ‚Π΅Ρ€ΠΈ напряТСния ΠΈΡΠΏΠΎΠ»ΡŒΠ·ΡƒΠ΅Ρ‚ΡΡ Ρ„ΠΎΡ€ΠΌΡƒΠ»Π°Β dU = IΒ·R.Β Π’ Π΄Π°Π½Π½ΠΎΠΉ Ρ„ΠΎΡ€ΠΌΡƒΠ»Π΅Β IΒ — это сила Ρ‚ΠΎΠΊΠ°, (А),dU – ΠΏΠΎΡ‚Π΅Ρ€ΠΈ напряТСния, (Π’), Π°Β RΒ — ΠΏΠΎΠΊΠ°Π·Ρ‹Π²Π°Π΅Ρ‚ сопротивлСниС кабСля ΠΈΠ»ΠΈ ΠΏΡ€ΠΎΠ²ΠΎΠ΄Π° Π² Ом. ПослС провСдСния расчСтов получаСтся Ρ‚Π°ΠΊΠΎΠΉ ΠΏΡ€ΠΈΠΌΠ΅Ρ€Β dU = 17,3Β·0,464 = 4,06 Π’ = 8,02 Π’.

Π§Ρ‚ΠΎ касаСтся расчСта ΠΏΠΎΡ‚Π΅Ρ€ΡŒ Π² ΠΏΡ€ΠΎΡ†Π΅Π½Ρ‚Π½ΠΎΠΌ ΡΠΎΠΎΡ‚Π½ΠΎΡˆΠ΅Π½ΠΈΠΈ, Ρ‚ΠΎ Π΄Π°Π½Π½Ρ‹ΠΉ ΠΏΠΎΠΊΠ°Π·Π°Ρ‚Π΅Π»ΡŒ выводится Ρ‚Π°ΠΊ —Β 8,02 Π’ / 220 Π’ Ρ… 100% = 3,65%. Как Π²ΠΈΠ΄Π½ΠΎ, ΠΏΠΎΠ»ΡƒΡ‡Π΅Π½Π½Ρ‹ΠΉ ΠΏΠΎΠΊΠ°Π·Π°Ρ‚Π΅Π»ΡŒ Π½Π΅ ΠΏΡ€Π΅Π²Ρ‹ΡˆΠ°Π΅Ρ‚ 5% Ρ‚ΠΎ Π΅ΡΡ‚ΡŒ допустимоС Π·Π½Π°Ρ‡Π΅Π½ΠΈΠ΅, Π° соотвСтствСнно Π²Ρ‹Π±ΠΎΡ€ Π±Ρ‹Π» осущСствлСн Π²Π΅Ρ€Π½ΠΎ. Π’ ситуации, Ссли Ρ†ΠΈΡ„Ρ€Π° Π±ΡƒΠ΄Π΅Ρ‚ большС Π΄Π°Π½Π½ΠΎΠΉ Π²Π΅Π»ΠΈΡ‡ΠΈΠ½Ρ‹, рСкомСндуСтся ΠΏΠΎΠ΄ΠΎΠ±Ρ€Π°Ρ‚ΡŒ ΠΌΠ΅Π΄Π½Ρ‹ΠΉ кабСль с ΠΏΠ°Ρ€Π°ΠΌΠ΅Ρ‚Ρ€ΠΎΠΌ сСчСния Π½Π΅ 1,5 ΠΌΠΌ, Π° 2,5 ΠΊΠ². ΠΌΠΌ.

Как Ρ€Π°ΡΡΡ‡ΠΈΡ‚Π°Ρ‚ΡŒ сСчСниС кабСля: 3 способа

Π‘Ρ€Π΅Π΄Π°, 19/05/2021 Π² 16:17

Π­Π»Π΅ΠΊΡ‚Ρ€ΠΎΠΏΡ€ΠΎΠ²ΠΎΠ΄ΠΊΠ° – ΠΎΠ΄Π½Π° ΠΈΠ· самых слоТных ΠΈΠ½ΠΆΠ΅Π½Π΅Ρ€Π½Ρ‹Ρ… систСм Π² Π΄ΠΎΠΌΠ΅. И ΠΎΡ‡Π΅Π½ΡŒ Π²Π°ΠΆΠ½ΠΎ ΠΏΡ€Π°Π²ΠΈΠ»ΡŒΠ½ΠΎ Π²Ρ‹Π±Ρ€Π°Ρ‚ΡŒ сСчСниС элСктрокабСля.

РасчСт сСчСния кабСля ΠΌΠΎΠΆΠ΅Ρ‚ Π²Ρ‹ΠΏΠΎΠ»Π½ΡΡ‚ΡŒΡΡ ΠΎΠ΄Π½ΠΈΠΌ ΠΈΠ· ΠΌΠ΅Ρ‚ΠΎΠ΄ΠΎΠ²:

  • ΠΏΠΎ мощности приборов – прСдполагаСт вычислСниС суммарной мощности всСх элСктроприборов ΠΈ сравнСниС ΠΏΠΎΠ»ΡƒΡ‡Π΅Π½Π½ΠΎΠ³ΠΎ значСния с расчСтным, взятым ΠΈΠ· ΡΠΏΠ΅Ρ†ΠΈΠ°Π»ΡŒΠ½ΠΎΠΉ Ρ‚Π°Π±Π»ΠΈΡ†Ρ‹;
  • ΠΏΠΎ длинС – высчитываСтся Π²Π΅Π»ΠΈΡ‡ΠΈΠ½Π° ΠΏΠΎΡ‚Π΅Ρ€ΡŒ напряТСния, которая зависит ΠΎΡ‚Β Π΄Π»ΠΈΠ½Ρ‹ Π»ΠΈΠ½ΠΈΠΈ кабСля, послС Ρ‡Π΅Π³ΠΎΒ ΠΎΠ½Π° сравниваСтся с Π±Π°Π·ΠΎΠ²Ρ‹ΠΌ Π·Π½Π°Ρ‡Π΅Π½ΠΈΠ΅ΠΌ Π² 5%;
  • ΠΏΠΎ току – опрСдСляСтся сила Ρ‚ΠΎΠΊΠ° ΠΊΠ°ΠΆΠ΄ΠΎΠ³ΠΎ ΠΈΠ· ΠΏΡ€ΠΈΠ±ΠΎΡ€ΠΎΠ², суммируСтся ΠΈ соотносится с Ρ‚Π°Π±Π»ΠΈΡ‡Π½Ρ‹ΠΌ Π·Π½Π°Ρ‡Π΅Π½ΠΈΠ΅ΠΌ. По Ρ‚Π°Π±Π»ΠΈΡ†Π΅ ΠΌΠΎΠΆΠ½ΠΎ ΠΎΠΏΡ€Π΅Π΄Π΅Π»ΠΈΡ‚ΡŒ, сколько ΠΆΠΈΠ» ΠΈ ΠΊΠ°ΠΊΠΎΠ³ΠΎ сСчСния Π΄ΠΎΠ»ΠΆΠ½ΠΎ Π±Ρ‹Ρ‚ΡŒ Ρƒ кабСля.

ΠŸΡ€Π°Π²ΠΈΠ»ΡŒΠ½Ρ‹ΠΉ ΠΏΠΎΠ΄Π±ΠΎΡ€ сСчСния ΠΈΠ·Π±Π°Π²ΠΈΡ‚ вас ΠΎΡ‚ мноТСства ΠΏΡ€ΠΎΠ±Π»Π΅ΠΌ. ΠŸΠΎΠ΄Π²Π΅Ρ€Π³Π°ΡŽΡ‰ΠΈΠΉΡΡ Ρ‡Ρ€Π΅Π·ΠΌΠ΅Ρ€Π½ΠΎΠΉ Π½Π°Π³Ρ€ΡƒΠ·ΠΊΠ΅, слишком слабый ΠΏΡ€ΠΎΠ²ΠΎΠ΄ ΠΌΠΎΠΆΠ΅Ρ‚ ΡΡ‚Π°Ρ‚ΡŒ ΠΏΡ€ΠΈΡ‡ΠΈΠ½ΠΎΠΉ самовозгорания ΠΈ ΠΊΠΎΡ€ΠΎΡ‚ΠΊΠΎΠ³ΠΎ замыкания. А дорогостоящая ΠΆΠΈΠ»Π° Ρ…ΠΎΡ‚ΡŒ ΠΈ Π±ΡƒΠ΄Π΅Ρ‚ Π½Π°Π΄Π΅ΠΆΠ½ΠΎ Π²Ρ‹ΠΏΠΎΠ»Π½ΡΡ‚ΡŒ свои Ρ„ΡƒΠ½ΠΊΡ†ΠΈΠΈ, обойдСтся Π² слишком Π±ΠΎΠ»ΡŒΡˆΡƒΡŽ сумму. Π‘ΡΠΊΠΎΠ½ΠΎΠΌΠΈΡ‚ΡŒ ΠΈ ΠΏΡ€ΠΈ этом ΠΏΠΎΠ»ΡƒΡ‡ΠΈΡ‚ΡŒ ΠΊΠ°Ρ‡Π΅ΡΡ‚Π²Π΅Π½Π½ΡƒΡŽ элСктропроводку ΠΏΠΎΠΌΠΎΠΆΠ΅Ρ‚ ΠΏΡ€Π°Π²ΠΈΠ»ΡŒΠ½Ρ‹ΠΉ Π²Ρ‹Π±ΠΎΡ€ сСчСния ΠΏΡ€ΠΎΠ²ΠΎΠ΄Π°. НСсмотря Π½Π° ΠΊΠ°ΠΆΡƒΡ‰ΡƒΡŽΡΡ ΡΠ»ΠΎΠΆΠ½ΠΎΡΡ‚ΡŒ, с Π΄Π°Π½Π½ΠΎΠΉ ΠΏΡ€ΠΎΡ†Π΅Π΄ΡƒΡ€ΠΎΠΉ ΠΌΠΎΠΆΠ΅Ρ‚ Ρ€Π°Π·ΠΎΠ±Ρ€Π°Ρ‚ΡŒΡΡ ΠΈ Ρ‡Π΅Π»ΠΎΠ²Π΅ΠΊ, Π½Π΅ связанный с элСктрикой.

ΠŸΠΎΠ΄Ρ€ΠΎΠ±Π½Π΅Π΅ ΠΎ ΠΊΠ°ΠΆΠ΄ΠΎΠΌ ΠΌΠ΅Ρ‚ΠΎΠ΄Π΅ расчСта Ρ‡ΠΈΡ‚Π°ΠΉΡ‚Π΅ Π΄Π°Π»Π΅Π΅ Π² ΡΡ‚Π°Ρ‚ΡŒΠ΅.

Π’Ρ‹Π±ΠΎΡ€ сСчСния кабСля ΠΏΠΎ мощности

КабСль характСризуСтся ΠΌΠΎΡ‰Π½ΠΎΡΡ‚ΡŒΡŽ, ΠΊΠΎΡ‚ΠΎΡ€ΡƒΡŽ ΠΎΠ½ способСн Π²Ρ‹Π΄Π΅Ρ€ΠΆΠ°Ρ‚ΡŒ Π² Ρ…ΠΎΠ΄Π΅ эксплуатации ΠΏΡ€ΠΈΠ±ΠΎΡ€ΠΎΠ². Если ΠΎΠ½Π° ΠΏΡ€Π΅Π²Ρ‹ΡˆΠ°Π΅Ρ‚ расчСтноС Π·Π½Π°Ρ‡Π΅Π½ΠΈΠ΅ токопроводящСй ΠΆΠΈΠ»Ρ‹, Ρ€Π°Π½ΠΎ ΠΈΠ»ΠΈ ΠΏΠΎΠ·Π΄Π½ΠΎ случится авария.

Π§Ρ‚ΠΎΠ±Ρ‹ Ρ€Π°ΡΡΡ‡ΠΈΡ‚Π°Ρ‚ΡŒ сСчСниС кабСля ΠΏΠΎ мощности, Π½ΡƒΠΆΠ½ΠΎ Π²Ρ‹ΡΡΠ½ΠΈΡ‚ΡŒ ΡΡƒΠΌΠΌΠ°Ρ€Π½ΡƒΡŽ ΠΌΠΎΡ‰Π½ΠΎΡΡ‚ΡŒ всСх ΠΏΡ€ΠΈΠ±ΠΎΡ€ΠΎΠ² с ΡƒΡ‡Π΅Ρ‚ΠΎΠΌ ΠΏΠΎΠ½ΠΈΠΆΠ°ΡŽΡ‰Π΅Π³ΠΎ коэффициСнта 0,8. Π’ΠΎ Π΅ΡΡ‚ΡŒ, Ρ„ΠΎΡ€ΠΌΡƒΠ»Π° Π±ΡƒΠ΄Π΅Ρ‚ ΠΈΠΌΠ΅Ρ‚ΡŒ Π²ΠΈΠ΄:

PΠΎΠ±Ρ‰.=(P1+P2+…+Pn)*0,8

ΠŸΠΎΠ½ΠΈΠΆΠ°ΡŽΡ‰Π΅ΠΉ коэффициСнт ΠΏΡ€Π΅Π΄ΠΏΠΎΠ»Π°Π³Π°Π΅Ρ‚, Ρ‡Ρ‚ΠΎ Π½Π΅ вся Ρ‚Π΅Ρ…Π½ΠΈΠΊΠ° Π² Π΄ΠΎΠΌΠ΅ Π±ΡƒΠ΄Π΅Ρ‚ ΠΎΠ΄Π½ΠΎΠΌΠΎΠΌΠ΅Π½Ρ‚Π½ΠΎ ΠΏΠΎΡ‚Ρ€Π΅Π±Π»ΡΡ‚ΡŒ ΡΠ»Π΅ΠΊΡ‚Ρ€ΠΎΡΠ½Π΅Ρ€Π³ΠΈΡŽ. ΠŸΠΎΠ»ΡƒΡ‡ΠΈΠ²ΡˆΠΈΠΉΡΡ расчСт сСчСния кабСля ΠΏΠΎ мощности сравниваСтся с Π΄Π°Π½Π½Ρ‹ΠΌΠΈ Π² Ρ‚Π°Π±Π»ΠΈΡ†Π΅ – это ΠΈ Π±ΡƒΠ΄Π΅Ρ‚ подходящСС сСчСниС.

Π’Π°Π±Π»ΠΈΡ†Π° сСчСния кабСля

Β 

НапримСр, общая ΠΌΠΎΡ‰Π½ΠΎΡΡ‚ΡŒ элСктроприборов Π² ΠΊΠ²Π°Ρ€Ρ‚ΠΈΡ€Π΅ равняСтся 15 ΠΊΠ’Ρ‚. Π£ΠΌΠ½ΠΎΠΆΠ°Π΅ΠΌ Π΅Π΅ Π½Π° 0,8 ΠΈ ΠΏΠΎΠ»ΡƒΡ‡Π°Π΅ΠΌ 12 ΠΊΠ’Ρ‚ Π½Π°Π³Ρ€ΡƒΠ·ΠΊΠΈ. Π’ Ρ‚Π°Π±Π»ΠΈΡ†Π΅ Π½ΡƒΠΆΠ½ΠΎ Π½Π°ΠΉΡ‚ΠΈ Π½Π°ΠΈΠ±ΠΎΠ»Π΅Π΅ подходящСС Π·Π½Π°Ρ‡Π΅Π½ΠΈΠ΅. Π’Π°ΠΊΠΈΠΌ ΠΎΠ±Ρ€Π°Π·ΠΎΠΌ, Π½Π΅ΠΎΠ±Ρ…ΠΎΠ΄ΠΈΠΌΠΎ Π²Ρ‹Π±Ρ€Π°Ρ‚ΡŒ ΠΌΠ΅Π΄Π½Ρ‹ΠΉ кабСль с сСчСниСм 10 ΠΌΠΌ для ΠΎΠ΄Π½ΠΎΡ„Π°Π·Π½ΠΎΠΉ сСти ΠΈ 6 ΠΌΠΌ для Ρ‚Ρ€Π΅Ρ…Ρ„Π°Π·Π½ΠΎΠΉ.

Π’Ρ‹Π±ΠΎΡ€ сСчСния кабСля ΠΏΠΎ Π΄Π»ΠΈΠ½Π΅

Π£ ΠΊΠ°ΠΆΠ΄ΠΎΠ³ΠΎ ΠΏΡ€ΠΎΠ²ΠΎΠ΄Π½ΠΈΠΊΠ° Π΅ΡΡ‚ΡŒ собствСнноС сопротивлСниС. Π‘ ΡƒΠ²Π΅Π»ΠΈΡ‡Π΅Π½ΠΈΠ΅ΠΌ Π΄Π»ΠΈΠ½Ρ‹ Π»ΠΈΠ½ΠΈΠΈ Π½Π°Π±Π»ΡŽΠ΄Π°Π΅Ρ‚ΡΡ потСря напряТСния, ΠΈ Ρ‡Π΅ΠΌ большС расстояниС, Ρ‚Π΅ΠΌ Π²Ρ‹ΡˆΠ΅ ΠΏΠΎΡ‚Π΅Ρ€ΠΈ. Если расчСтная Π²Π΅Π»ΠΈΡ‡ΠΈΠ½Π° ΠΏΠΎΡ‚Π΅Ρ€ΡŒ становится большС 5%, трСбуСтся Π²Ρ‹Π±Ρ€Π°Ρ‚ΡŒ ΠΏΡ€ΠΎΠ²ΠΎΠ΄ с Π±ΠΎΠ»Π΅Π΅ ΠΊΡ€ΡƒΠΏΠ½Ρ‹ΠΌΠΈ ΠΆΠΈΠ»Π°ΠΌΠΈ.

РасчСт ΠΏΠΎ Π΄Π»ΠΈΠ½Π΅ состоит ΠΈΠ· Π΄Π²ΡƒΡ… этапов ΠΈ ΠΏΠΎΠ΄Ρ€Π°Π·ΡƒΠΌΠ΅Π²Π°Π΅Ρ‚, Ρ‡Ρ‚ΠΎ Π·Π°Ρ€Π°Π½Π΅Π΅ извСстно, сколько ΠΌΠ΅Ρ‚Ρ€ΠΎΠ² ΠΏΡ€ΠΎΠ²ΠΎΠ΄Π° потрСбуСтся для ΠΌΠΎΠ½Ρ‚Π°ΠΆΠ° элСктропроводки.

  1. Π’Π½Π°Ρ‡Π°Π»Π΅ слСдуСт ΠΎΠΏΡ€Π΅Π΄Π΅Π»ΠΈΡ‚ΡŒ Π½ΠΎΠΌΠΈΠ½Π°Π»ΡŒΠ½ΡƒΡŽ силу Ρ‚ΠΎΠΊΠ°. Π”Π»ΠΈΠ½Π° ΠΏΡ€ΠΎΠ²ΠΎΠ΄ΠΊΠΈ пСрСводится Π² ΠΌΠΈΠ»Π»ΠΈΠΌΠ΅Ρ‚Ρ€Ρ‹ ΠΈ умноТаСтся Π½Π° 2, ΠΏΠΎΡ‚ΠΎΠΌΡƒ Ρ‡Ρ‚ΠΎ Ρ‚ΠΎΠΊ ΡƒΡ…ΠΎΠ΄ΠΈΡ‚ ΠΏΠΎ ΠΎΠ΄Π½ΠΎΠΉ ΠΆΠΈΠ»Π΅, Π° возвращаСтся ΠΏΠΎ Π΄Ρ€ΡƒΠ³ΠΎΠΉ.

I = (P · Кс) / (UΒ Β·Β cosΒ Ο•) = 8000/220 = 36 А,

Π³Π΄Π΅ P – ΠΌΠΎΡ‰Π½ΠΎΡΡ‚ΡŒ Π² Π’Ρ‚ (суммируСтся вся Ρ‚Π΅Ρ…Π½ΠΈΠΊΠ° Π² Π΄ΠΎΠΌΠ΅), U – 220 Π’, ΠšΡΒ β€“ коэффициСнт ΠΎΠ΄Π½ΠΎΠ²Ρ€Π΅ΠΌΠ΅Π½Π½ΠΎΠ³ΠΎ Π²ΠΊΠ»ΡŽΡ‡Π΅Π½ΠΈΡ (0,75), cos Ο† – 1 для Π±Ρ‹Ρ‚ΠΎΠ²Ρ‹Ρ… ΠΏΡ€ΠΈΠ±ΠΎΡ€ΠΎΠ².

  1. НСобходимо ΡƒΠ·Π½Π°Ρ‚ΡŒ сСчСниС ΠΏΡ€ΠΎΠ²ΠΎΠ΄Π½ΠΈΠΊΠ°. ΠŸΠΎΠΌΠΎΠΆΠ΅Ρ‚ Ρ„ΠΎΡ€ΠΌΡƒΠ»Π°: RΒ = ρ Β·Β L/S. Зная, Ρ‡Ρ‚ΠΎ ΠΏΠΎΡ‚Π΅Ρ€ΠΈ напряТСния Π΄ΠΎΠ»ΠΆΠ½Ρ‹ ΡΠΎΡΡ‚Π°Π²Π»ΡΡ‚ΡŒ максимум 5%, рассчитываСм:

dU = 0,05Β Β· 220 Π’Β = 11 Π’.

Π”Π°Π»Π΅Π΅ выясняСм ΠΏΠΎΡ‚Π΅Ρ€ΡŽ напряТСния ΠΏΠΎ Ρ„ΠΎΡ€ΠΌΡƒΠ»Π΅:

dU = I · R. R = dU/I = 11/36 = 0,31 Ом.

Π’Π°ΠΊΠΈΠΌ ΠΎΠ±Ρ€Π°Π·ΠΎΠΌ, искомоС сСчСниС ΠΏΡ€ΠΎΠ²ΠΎΠ΄Π½ΠΈΠΊΠ°:

S = ρ · L/R = 0,0175 · 40/0,31 = 2,25 мм2.

Π’ случаС с Ρ‚Ρ€Π΅Ρ…ΠΆΠΈΠ»ΡŒΠ½Ρ‹ΠΌ ΠΊΠ°Π±Π΅Π»Π΅ΠΌ, ΠΏΠ»ΠΎΡ‰Π°Π΄ΡŒ Π΅Π³ΠΎ ΠΏΠΎΠΏΠ΅Ρ€Π΅Ρ‡Π½ΠΎΠ³ΠΎ сСчСния (ΠΎΠ΄Π½Π° ΠΆΠΈΠ»Π°) Π΄ΠΎΠ»ΠΆΠ½Π° ΡΠΎΡΡ‚Π°Π²Π»ΡΡ‚ΡŒ 0,75 ΠΌΠΌ2. Π’Π°ΠΊΠΈΠΌ ΠΎΠ±Ρ€Π°Π·ΠΎΠΌ, Π΄ΠΈΠ°ΠΌΠ΅Ρ‚Ρ€ ΠΆΠΈΠ»Ρ‹ Π΄ΠΎΠ»ΠΆΠ΅Π½ Π±Ρ‹Ρ‚ΡŒ ΠΌΠΈΠ½ΠΈΠΌΡƒΠΌ (√S/Β Ο€)Β Β·Β 2 = 0,98 ΠΌΠΌ.Β Π­Ρ‚ΠΎΠΌΡƒ ΡƒΡΠ»ΠΎΠ²ΠΈΡŽ удовлСтворяСт кабСль BBΠ“Π½Π³ 3×1,5Β ΠΌΠΌ.

Π’Ρ‹Π±ΠΎΡ€ сСчСния кабСля ΠΏΠΎ Ρ‚ΠΎΠΊΡƒ

Π”Π°Π½Π½Ρ‹ΠΉ ΠΌΠ΅Ρ‚ΠΎΠ΄, Ρ‚Π°ΠΊΠΆΠ΅ извСстный ΠΊΠ°ΠΊ расчСт сСчСния ΠΏΡ€ΠΎΠ²ΠΎΠ΄Π° ΠΏΠΎ Π½Π°Π³Ρ€ΡƒΠ·ΠΊΠ΅, считаСтся самым Ρ‚ΠΎΡ‡Π½Ρ‹ΠΌ. Π’Π½Π°Ρ‡Π°Π»Π΅ Π½Π΅ΠΎΠ±Ρ…ΠΎΠ΄ΠΈΠΌΠΎ Π½Π°ΠΉΡ‚ΠΈ силу Ρ‚ΠΎΠΊΠ° ΠΊΠ°ΠΆΠ΄ΠΎΠ³ΠΎ ΠΏΡ€ΠΈΠ±ΠΎΡ€Π°.

Π’ случаС ΠΎΠ΄Π½ΠΎΡ„Π°Π·Π½ΠΎΠΉ сСти для расчСта Π½Π΅ΠΎΠ±Ρ…ΠΎΠ΄ΠΈΠΌΠΎ Π²ΠΎΡΠΏΠΎΠ»ΡŒΠ·ΠΎΠ²Π°Ρ‚ΡŒΡΡ ΡΠ»Π΅Π΄ΡƒΡŽΡ‰Π΅ΠΉ Ρ„ΠΎΡ€ΠΌΡƒΠ»ΠΎΠΉ

I = (P Β· Кс) / (U Β· cos Ο•).

Π‘ΡƒΠΌΠΌΡƒ высчитанных Ρ‚ΠΎΠΊΠΎΠ² Π½Π΅ΠΎΠ±Ρ…ΠΎΠ΄ΠΈΠΌΠΎ соотнСсти с Ρ‚Π°Π±Π»ΠΈΡ‡Π½Ρ‹ΠΌΠΈ значСниями.

Π’ ΠΏΡ€ΠΈΠΌΠ΅Ρ€Π΅ с ΠΎΠ΄Π½ΠΎΡ„Π°Π·Π½ΠΎΠΉ Π·Π°ΠΊΡ€Ρ‹Ρ‚ΠΎΠΉ ΡΠ΅Ρ‚ΡŒΡŽ ΠΈ ΠΌΠΎΡ‰Π½ΠΎΡΡ‚ΡŒΡŽ ΠΏΡ€ΠΈΠ±ΠΎΡ€ΠΎΠ² 5 ΠΊΠ’Ρ‚:

I = (P Β· Кс) / (U Β· cos Ο•) = (5000 Β· 0,75) / (220 Β· 1) = 17,05 А, ΠΏΡ€ΠΈ ΠΎΠΊΡ€ΡƒΠ³Π»Π΅Π½ΠΈΠΈ 18 А.

BBΠ“Π½Π³ 3×1,5 – это ΠΌΠ΅Π΄Π½Ρ‹ΠΉ Ρ‚Ρ€Π΅Ρ…ΠΆΠΈΠ»ΡŒΠ½Ρ‹ΠΉ кабСль. Π’ Ρ‚Π°Π±Π»ΠΈΡ†Π΅ блиТайшСС ΠΊ силС Ρ‚ΠΎΠΊΠ° 18Β  A Π·Π½Π°Ρ‡Π΅Π½ΠΈΠ΅ – 19 А (Π² случаС ΠΏΡ€ΠΎΠΊΠ»Π°Π΄ΠΊΠΈ Π² Π²ΠΎΠ·Π΄ΡƒΡ…Π΅). Π’Π°ΠΊΠΈΠΌ ΠΎΠ±Ρ€Π°Π·ΠΎΠΌ, сСчСниС Π΅Π³ΠΎ ΠΆΠΈΠ»Ρ‹ Π΄ΠΎΠ»ΠΆΠ½ΠΎ ΡΠΎΡΡ‚Π°Π²Π»ΡΡ‚ΡŒ ΠΌΠΈΠ½ΠΈΠΌΡƒΠΌ 1,5 ΠΌΠΌ2. Π‘Π΅Ρ‡Π΅Π½ΠΈΠ΅ ΠΆΠΈΠ»Ρ‹ BBΠ“Π½Π³ 3×1,5 Ρ€Π°Π²Π½ΠΎ S = Ο€ Β· r2 = 3,14 Β· (1,5/2)2 = 1,8 ΠΌΠΌ2, соотвСтствСнно, ΠΎΠ½ΠΎ удовлСтворяСт ΡƒΠΊΠ°Π·Π°Π½Π½ΠΎΠΌΡƒ Ρ‚Ρ€Π΅Π±ΠΎΠ²Π°Π½ΠΈΡŽ.

Π’Ρ‹Π±Ρ€Π°Ρ‚ΡŒ Π½Π΅ΠΎΠ±Ρ…ΠΎΠ΄ΠΈΠΌΡ‹ΠΉ Π²Π°ΠΌ кабСль Π²Ρ‹ ΠΌΠΎΠΆΠ΅Ρ‚Π΅ Π² нашСм ΠΊΠ°Ρ‚Π°Π»ΠΎΠ³Π΅. ΠœΡ‹ Ρ€Π΅Π°Π»ΠΈΠ·ΡƒΠ΅ΠΌ ΠΏΡ€ΠΎΠ΄ΡƒΠΊΡ†ΠΈΡŽ физичСским ΠΈ ΡŽΡ€ΠΈΠ΄ΠΈΡ‡Π΅ΡΠΊΠΈΠΌ Π»ΠΈΡ†Π°ΠΌ с Π²ΠΎΠ·ΠΌΠΎΠΆΠ½ΠΎΡΡ‚ΡŒΡŽ доставки ΠΏΠΎ адрСсу ΠΈΠ»ΠΈ Π² ΡƒΠ΄ΠΎΠ±Π½Ρ‹ΠΉ ΠΏΡƒΠ½ΠΊΡ‚ самовывоза.

Π’ΠΈΠ΄Π΅ΠΎ с вопросом: РасчСт ΠΏΠΎΠΏΠ΅Ρ€Π΅Ρ‡Π½ΠΎΠ³ΠΎ сСчСния ΠΏΡ€ΠΎΠ²ΠΎΠ΄Π° с использованиСм сопротивлСния ΠΈ ΡƒΠ΄Π΅Π»ΡŒΠ½ΠΎΠ³ΠΎ сопротивлСния

Π‘Ρ‚Π΅Π½ΠΎΠ³Ρ€Π°ΠΌΠΌΠ° видСозаписи

ΠœΠ΅Π΄Π½Ρ‹ΠΉ ΠΏΡ€ΠΎΠ²ΠΎΠ΄ с сопротивлСниСм 22 ΠΌΠΈΠ»Π»ΠΈΠΎΠΌ ΠΈΠΌΠ΅Π΅Ρ‚ Π΄Π»ΠΈΠ½Ρƒ 6,2 ΠΌΠ΅Ρ‚Ρ€Π°. НайдитС ΠΏΠ»ΠΎΡ‰Π°Π΄ΡŒ ΠΏΠΎΠΏΠ΅Ρ€Π΅Ρ‡Π½ΠΎΠ³ΠΎ сСчСния. Π˜ΡΠΏΠΎΠ»ΡŒΠ·ΡƒΠΉΡ‚Π΅ 1,7 ΡƒΠΌΠ½ΠΎΠΆΠΈΡ‚ΡŒ Π½Π° 10 Π΄ΠΎ ΠΎΡ‚Ρ€ΠΈΡ†Π°Ρ‚Π΅Π»ΡŒΠ½Ρ‹Ρ… восьми Ом для ΡƒΠ΄Π΅Π»ΡŒΠ½ΠΎΠ³ΠΎ сопротивлСния ΠΌΠ΅Π΄ΠΈ. ΠžΡ‚Π²Π΅Ρ‚ Π΄Π°ΠΉΡ‚Π΅ Π² Π½Π°ΡƒΡ‡Π½ΠΎΠΌ прСдставлСнии с Ρ‚ΠΎΡ‡Π½ΠΎΡΡ‚ΡŒΡŽ Π΄ΠΎ ΠΎΠ΄Π½ΠΎΠ³ΠΎ дСсятичного Π·Π½Π°ΠΊΠ°.

Допустим, это наш ΠΌΠ΅Π΄Π½Ρ‹ΠΉ ΠΏΡ€ΠΎΠ²ΠΎΠ΄.Π•Π³ΠΎ Π΄Π»ΠΈΠ½Π°, Π½Π°Π·ΠΎΠ²Π΅ΠΌ Π΅Π³ΠΎ 𝐿, составляСт 6,2 ΠΌΠ΅Ρ‚Ρ€Π°. А ΠΎΠ±Ρ‰Π΅Π΅ сопротивлСниС ΠΏΠΎΡ‚ΠΎΠΊΡƒ заряда ΠΎΡ‚ ΠΎΠ΄Π½ΠΎΠ³ΠΎ ΠΊΠΎΠ½Ρ†Π° ΠΏΡ€ΠΎΠ²ΠΎΠ΄Π° ΠΊ Π΄Ρ€ΡƒΠ³ΠΎΠΌΡƒ составляСт 22 ΠΌΠΈΠ»Π»ΠΈΠΎΠΌ; Π½Π°Π·ΠΎΠ²Π΅ΠΌ это 𝑅. Зная Ρ‚Π°ΠΊΠΆΠ΅ ΡƒΠ΄Π΅Π»ΡŒΠ½ΠΎΠ΅ сопротивлСниС — Π½Π°Π·ΠΎΠ²Π΅ΠΌ Π΅Π³ΠΎ 𝜌 ΠΏΡ€ΠΎΠ²ΠΎΠ΄Π° — ΠΌΡ‹ Ρ…ΠΎΡ‚ΠΈΠΌ Π½Π°ΠΉΡ‚ΠΈ ΠΏΠ»ΠΎΡ‰Π°Π΄ΡŒ ΠΏΠΎΠΏΠ΅Ρ€Π΅Ρ‡Π½ΠΎΠ³ΠΎ сСчСния ΠΏΡ€ΠΎΠ²ΠΎΠ΄Π°. Π§Ρ‚ΠΎΠ±Ρ‹ ΠΏΠΎΠΌΠΎΡ‡ΡŒ Π½Π°ΠΌ Π² этом, ΠΌΡ‹ ΠΌΠΎΠΆΠ΅ΠΌ Π²ΡΠΏΠΎΠΌΠ½ΠΈΡ‚ΡŒ взаимосвязь ΠΌΠ΅ΠΆΠ΄Ρƒ сопротивлСниСм ΠΈ ΡƒΠ΄Π΅Π»ΡŒΠ½Ρ‹ΠΌ сопротивлСниСм. Π‘ΠΎΠΏΡ€ΠΎΡ‚ΠΈΠ²Π»Π΅Π½ΠΈΠ΅ 𝑅 Ρ€Π°Π²Π½ΠΎ ΡƒΠ΄Π΅Π»ΡŒΠ½ΠΎΠΌΡƒ ΡΠΎΠΏΡ€ΠΎΡ‚ΠΈΠ²Π»Π΅Π½ΠΈΡŽ 𝜌, ΡƒΠΌΠ½ΠΎΠΆΠ΅Π½Π½ΠΎΠΌΡƒ Π½Π° Π΄Π»ΠΈΠ½Ρƒ рассматриваСмого ΠΌΠ°Ρ‚Π΅Ρ€ΠΈΠ°Π»Π° 𝐿, Π΄Π΅Π»Π΅Π½Π½ΡƒΡŽ Π½Π° Π΅Π³ΠΎ ΠΏΠ»ΠΎΡ‰Π°Π΄ΡŒ ΠΏΠΎΠΏΠ΅Ρ€Π΅Ρ‡Π½ΠΎΠ³ΠΎ сСчСния.

ΠžΠ±Ρ€Π°Ρ‚ΠΈΡ‚Π΅ Π²Π½ΠΈΠΌΠ°Π½ΠΈΠ΅, Ρ‡Ρ‚ΠΎ Ссли ΠΌΡ‹ ΡƒΠΌΠ½ΠΎΠΆΠΈΠΌ ΠΎΠ±Π΅ части этого уравнСния Π½Π° 𝐴, Ρ€Π°Π·Π΄Π΅Π»Π΅Π½Π½ΠΎΠ΅ Π½Π°, Π² Π»Π΅Π²ΠΎΠΉ части сопротивлСниС сократится, Π° ΠΏΠ»ΠΎΡ‰Π°Π΄ΡŒ ΠΏΠΎΠΏΠ΅Ρ€Π΅Ρ‡Π½ΠΎΠ³ΠΎ сСчСния справа ΡƒΠΌΠ΅Π½ΡŒΡˆΠΈΡ‚ΡΡ. ΠœΡ‹ ΠΎΠ±Π½Π°Ρ€ΡƒΠΆΠΈΠ»ΠΈ, Ρ‡Ρ‚ΠΎ ΠΏΠ»ΠΎΡ‰Π°Π΄ΡŒ ΠΏΠΎΠΏΠ΅Ρ€Π΅Ρ‡Π½ΠΎΠ³ΠΎ сСчСния Ρ€Π°Π²Π½Π° ΡƒΠ΄Π΅Π»ΡŒΠ½ΠΎΠΌΡƒ ΡΠΎΠΏΡ€ΠΎΡ‚ΠΈΠ²Π»Π΅Π½ΠΈΡŽ, ΡƒΠΌΠ½ΠΎΠΆΠ΅Π½Π½ΠΎΠΌΡƒ Π½Π° Π΄Π»ΠΈΠ½Ρƒ ΠΌΠ°Ρ‚Π΅Ρ€ΠΈΠ°Π»Π° 𝐿, Π΄Π΅Π»Π΅Π½Π½ΠΎΠΌΡƒ Π½Π° сопротивлСниС ΠΌΠ°Ρ‚Π΅Ρ€ΠΈΠ°Π»Π°. Если ΠΌΡ‹ подставим Π² это ΡƒΡ€Π°Π²Π½Π΅Π½ΠΈΠ΅ наши Π·Π°Π΄Π°Π½Π½Ρ‹Π΅ значСния ΡƒΠ΄Π΅Π»ΡŒΠ½ΠΎΠ³ΠΎ сопротивлСния, Π΄Π»ΠΈΠ½Ρ‹ ΠΈ сопротивлСния, ΠΌΡ‹ смоТСм Π±ΠΎΠ»Π΅Π΅ Ρ‡Π΅Ρ‚ΠΊΠΎ ΡƒΠ²ΠΈΠ΄Π΅Ρ‚ΡŒ, ΠΊΠ°ΠΊΠΈΠΌΠΈ Π±ΡƒΠ΄ΡƒΡ‚ Π΅Π΄ΠΈΠ½ΠΈΡ†Ρ‹ Π² этом Π²Ρ‹Ρ€Π°ΠΆΠ΅Π½ΠΈΠΈ.

Π’ числитСлС Ρƒ нас Π΅ΡΡ‚ΡŒ Ом, ΡƒΠΌΠ½ΠΎΠΆΠ΅Π½Π½ΠΎΠ΅ Π½Π° ΠΌΠ΅Ρ‚Ρ€, ΡƒΠΌΠ½ΠΎΠΆΠ΅Π½Π½ΠΎΠ΅ Π½Π° ΠΌΠ΅Ρ‚Ρ€.ΠŸΠΎΡΠΊΠΎΠ»ΡŒΠΊΡƒ ΠΌΡ‹ вычисляСм ΠΏΠ»ΠΎΡ‰Π°Π΄ΡŒ, Π½Π°ΠΌ Π½ΡƒΠΆΠ½ΠΎ, Ρ‡Ρ‚ΠΎΠ±Ρ‹ наши ΠΊΠΎΠ½Π΅Ρ‡Π½Ρ‹Π΅ Π΅Π΄ΠΈΠ½ΠΈΡ†Ρ‹ Π±Ρ‹Π»ΠΈ Π² ΠΌΠ΅Ρ‚Ρ€Π°Ρ… Π² ΠΊΠ²Π°Π΄Ρ€Π°Ρ‚Π΅. Π­Ρ‚ΠΎ ΠΎΠ·Π½Π°Ρ‡Π°Π΅Ρ‚, Ρ‡Ρ‚ΠΎ Π΅Π΄ΠΈΠ½ΠΈΡ†Ρ‹ Π² ΠΎΠΌΠ°Ρ… Π² числитСлС Π΄ΠΎΠ»ΠΆΠ½Ρ‹ ΡΠΎΠΊΡ€Π°Ρ‰Π°Ρ‚ΡŒΡΡ с использованиСм ΠΎΠΌ Π² Π·Π½Π°ΠΌΠ΅Π½Π°Ρ‚Π΅Π»Π΅. Π§Ρ‚ΠΎΠ±Ρ‹ это ΠΏΡ€ΠΎΠΈΠ·ΠΎΡˆΠ»ΠΎ, Π΄Π°Π²Π°ΠΉΡ‚Π΅ ΠΏΡ€Π΅ΠΎΠ±Ρ€Π°Π·ΡƒΠ΅ΠΌ нашС сопротивлСниС, ΠΊΠΎΡ‚ΠΎΡ€ΠΎΠ΅ Π² настоящСС врСмя выраТаСтся Π² ΠΌΠΈΠ»Π»ΠΈΠΎΠΌΠ°Ρ…, Π² число Π² ΠΎΠΌΠ°Ρ…. ΠŸΡ€Π΅ΠΎΠ±Ρ€Π°Π·ΠΎΠ²Π°Π½ΠΈΠ΅ ΠΌΠ΅ΠΆΠ΄Ρƒ этими Π΅Π΄ΠΈΠ½ΠΈΡ†Π°ΠΌΠΈ Ρ‚Π°ΠΊΠΎΠ²ΠΎ, Ρ‡Ρ‚ΠΎ 1000 ΠΌΠΈΠ»Π»ΠΈΠΎΠΌ Ρ€Π°Π²Π΅Π½ ΠΎΠ΄Π½ΠΎΠΌΡƒ ΠΎΠΌΡƒ. Π˜Ρ‚Π°ΠΊ, Ρ‡Ρ‚ΠΎΠ±Ρ‹ ΠΏΡ€Π΅ΠΎΠ±Ρ€Π°Π·ΠΎΠ²Π°Ρ‚ΡŒ ΠΌΠΈΠ»Π»ΠΈΠΎΠΌ Π² ΠΎΠΌ, ΠΌΡ‹ возьмСм дСсятичный разряд ΠΈ сдвинСм Π΅Π³ΠΎ Π½Π° ΠΎΠ΄Π½Ρƒ, Π΄Π²Π΅, Ρ‚Ρ€ΠΈ ΠΏΠΎΠ·ΠΈΡ†ΠΈΠΈ Π²Π»Π΅Π²ΠΎ. Π­Ρ‚ΠΎ эквивалСнтно дСлСнию Π½Π° 1000.Π’ΠΎΠ³Π΄Π° 22 ΠΌΠΈΠ»Π»ΠΈΠΎΠΌ Ρ€Π°Π²Π½ΠΎ 0,022 Ом. ΠžΠ±Ρ€Π°Ρ‚ΠΈΡ‚Π΅ Π²Π½ΠΈΠΌΠ°Π½ΠΈΠ΅, Ρ‡Ρ‚ΠΎ Ρ‚Π΅ΠΏΠ΅Ρ€ΡŒ ΠΌΡ‹ Π²Ρ‹Ρ‡Ρ‚Π΅ΠΌ Π΅Π΄ΠΈΠ½ΠΈΡ†Ρ‹ Ом ΠΈΠ· числитСля ΠΈ знамСнатСля.

Когда ΠΌΡ‹ вычисляСм 𝐴 ΠΈ округляСм Π΅Π³ΠΎ Π΄ΠΎ ΠΎΠ΄Π½ΠΎΠ³ΠΎ дСсятичного Π·Π½Π°ΠΊΠ°, Π² ΡΠΊΡΠΏΠΎΠ½Π΅Π½Ρ†ΠΈΠ°Π»ΡŒΠ½ΠΎΠΌ прСдставлСнии получаСтся 4,8 ΡƒΠΌΠ½ΠΎΠΆΠ΅Π½Π½ΠΎΠ΅ Π½Π° 10 Π΄ΠΎ ΠΎΡ‚Ρ€ΠΈΡ†Π°Ρ‚Π΅Π»ΡŒΠ½Ρ‹Ρ… ΡˆΠ΅ΡΡ‚ΠΈ ΠΌΠ΅Ρ‚Ρ€ΠΎΠ² Π² ΠΊΠ²Π°Π΄Ρ€Π°Ρ‚Π΅. Π­Ρ‚ΠΎ ΠΏΠ»ΠΎΡ‰Π°Π΄ΡŒ ΠΏΠΎΠΏΠ΅Ρ€Π΅Ρ‡Π½ΠΎΠ³ΠΎ сСчСния нашСго ΠΌΠ΅Π΄Π½ΠΎΠ³ΠΎ ΠΏΡ€ΠΎΠ²ΠΎΠ΄Π°.

Litz Calculator & Design from YDK Litz Wire & Cable

РасчСт шага ΠΈ Π½Π°ΠΏΡ€Π°Π²Π»Π΅Π½ΠΈΠ΅ ΠΏΡ€ΠΎΠΊΠ»Π°Π΄ΠΊΠΈ Π»ΠΈΡ‚Ρ†-ΠΏΡ€ΠΎΠ²ΠΎΠ΄Π°
ΠŸΡ€ΠΎΠ΅ΠΊΡ‚ΠΈΡ€ΠΎΠ²Π°Π½ΠΈΠ΅ конструкции ΠΈ расчСт Π»ΠΈΡ‚Ρ†-ΠΏΡ€ΠΎΠ²ΠΎΠ΄Π°
Для расчСта Β«ΠΎΠ΄ΠΈΠ½Π°ΠΊΠΎΠ²ΠΎΠΉ ΠΏΠ»ΠΎΡ‰Π°Π΄ΠΈ ΠΏΠΎΠΏΠ΅Ρ€Π΅Ρ‡Π½ΠΎΠ³ΠΎ сСчСния ΠΈΠ»ΠΈ ΠΎΠ΄ΠΈΠ½Π°ΠΊΠΎΠ²ΠΎΠΉ ΠΏΠ»ΠΎΡ‰Π°Π΄ΠΈ повСрхности» Π»ΠΈΡ‚Ρ†-ΠΏΡ€ΠΎΠ²ΠΎΠ΄Π° ΠΏΡ€ΠΎΠ²ΠΎΠ΄
ΠœΠ΅Ρ‚ΠΎΠ΄ увСличСния коэффициСнта добротности ΠΈ значСния индуктивности
РасчСт ΠΊΠ²Π°Π΄Ρ€Π°Ρ‚Π½ΠΎΠ³ΠΎ ΠΌΠ΅Ρ‚Ρ€Π° для Ρ‚Ρ€Π΅Π±ΠΎΠ²Π°Π½ΠΈΠΉ ΠΊ ΡˆΠ΅Π»ΠΊΡƒ ΠΈ Π½Π΅ΠΉΠ»ΠΎΠ½Ρƒ Π½Π° повСрхности Π»ΠΈΡ‚Ρ†-ΠΏΡ€ΠΎΠ²ΠΎΠ»ΠΎΠΊΠΈ

1.РасчСт шага

Π¨Π°Π³ ΠΏΡ€ΠΎΠ²ΠΎΠ»ΠΎΠΊΠΈ Litz

Π”Π»ΠΈΠ½Π° свивки ΠΏΠΎΠΊΠ°Π·Ρ‹Π²Π°Π΅Ρ‚ ΠΈΠ½Ρ‚Π΅Ρ€Π²Π°Π», Π½Π΅ΠΎΠ±Ρ…ΠΎΠ΄ΠΈΠΌΡ‹ΠΉ ΠΎΠ΄ΠΈΠ½ΠΎΡ‡Π½ΠΎΠΌΡƒ ΠΏΡ€ΠΎΠ²ΠΎΠ΄Ρƒ для ΠΎΠ΄Π½ΠΎΠ³ΠΎ ΠΏΠΎΠ»Π½ΠΎΠ³ΠΎ Π²ΠΈΡ‚ΠΊΠ° (= ΠΎΠ±ΠΎΡ€ΠΎΡ‚) ΠΏΠΎ ΠΏΠ΅Ρ€ΠΈΠΌΠ΅Ρ‚Ρ€Ρƒ Π³ΠΈΠ±ΠΊΠΎΠ³ΠΎ ΠΏΡ€ΠΎΠ²ΠΎΠ΄Π° (360 градусов).

Π’Π΅Ρ€ΠΌΠΈΠ½ относится ΠΊ Ρ€Π°ΡΡΡ‚ΠΎΡΠ½ΠΈΡŽ, Ρ‚Ρ€Π΅Π±ΡƒΠ΅ΠΌΠΎΠΌΡƒ для Β«Π΄Π»ΠΈΠ½Ρ‹ ΡƒΠΊΠ»Π°Π΄ΠΊΠΈ (= шаг)Β» (см. Рисунок Π²Ρ‹ΡˆΠ΅), ΠΊΠΎΡ‚ΠΎΡ€Ρ‹ΠΉ ΠΌΠΎΠΆΠ΅Ρ‚ Π±Ρ‹Ρ‚ΡŒ ΠΏΠΎΠ²Π΅Ρ€Π½ΡƒΡ‚ Π½Π° 360 Β° Π½Π° ΠΎΠ΄Π½Ρƒ линию. Π‘Ρ‚Π°Π½Π΄Π°Ρ€Ρ‚ EN 60317-11 Ρ€Π΅ΠΊΠΎΠΌΠ΅Π½Π΄ΡƒΠ΅Ρ‚ ΠΎΠ±Π΅ΡΠΏΠ΅Ρ‡ΠΈΠ²Π°Ρ‚ΡŒ 60 ΠΌΠΌ, ΠΌΠ°ΠΊΡΠΈΠΌΠ°Π»ΡŒΠ½ΡƒΡŽ Π΄Π»ΠΈΠ½Ρƒ свивки обслуТиваСмой Π»ΠΈΡ‚Ρ†ΠΎΠ²ΠΎΠΉ ΠΏΡ€ΠΎΠ²ΠΎΠ»ΠΎΠΊΠΈ. Однако Π½Π° самом Π΄Π΅Π»Π΅ Π΄Π»ΠΈΠ½Π° свивки ΠΎΡ‚ 0.ΠžΡ‚ 80 ΠΌΠΌ Π΄ΠΎ 60 ΠΌΠΌ. (0,4 Π²ΠΈΡ‚ΠΊΠ° Π½Π° дюйм / 5 Π²ΠΈΡ‚ΠΊΠΎΠ² Π½Π° Ρ„ΡƒΡ‚, 32 Π²ΠΈΡ‚ΠΊΠ° Π½Π° дюйм)

ex 1) Π§Ρ‚ΠΎ касаСтся 4 Π½ΠΈΡ‚Π΅ΠΉ x 0,63 ΠΌΠΌ,
OD (= Π’Π½Π΅ΡˆΠ½ΠΈΠΉ Π΄ΠΈΠ°ΠΌΠ΅Ρ‚Ρ€) для 4 x 0,63 ΠΌΠΌ составляСт ΠΏΡ€ΠΈΠ±Π»ΠΈΠ·ΠΈΡ‚Π΅Π»ΡŒΠ½ΠΎ 1,45 ΠΌΠΌ. (ΠΈΡΠΏΠΎΠ»ΡŒΠ·ΡƒΡ Π²Π°ΡˆΡƒ Ρ„ΠΎΡ€ΠΌΡƒΠ»Ρƒ). Π­Ρ‚ΠΎ Π΄Π°Π΅Ρ‚ Π΄Π»ΠΈΠ½Ρƒ ΡƒΠΊΠ»Π°Π΄ΠΊΠΈ 36,25 ΠΌΠΌ, Ρ‡Ρ‚ΠΎ соотвСтствуСт 28 Π²ΠΈΡ‚ΠΊΠ°ΠΌ Π½Π° ΠΌΠ΅Ρ‚Ρ€, ΠΎΠ΄Π½Π°ΠΊΠΎ для Ρ…ΠΎΡ€ΠΎΡˆΠ΅ΠΉ Ρ€Π°Π±ΠΎΡ‚Ρ‹ Π² этой ΠΊΠΎΠ½Ρ„ΠΈΠ³ΡƒΡ€Π°Ρ†ΠΈΠΈ слСдуСт ΠΈΡΠΏΠΎΠ»ΡŒΠ·ΠΎΠ²Π°Ρ‚ΡŒ 54 Π²ΠΈΡ‚ΠΊΠ° Π½Π° ΠΌΠ΅Ρ‚Ρ€.

-> 4 Ρ… 0,63.
√4 x 1,154 x (0,63 + Ρ‚ΠΎΠ»Ρ‰ΠΈΠ½Π° эмалСвого покрытия) od = ΠΏΡ€ΠΈΠ±Π».1,45 ΠΌΠΌ
закладная Π΄Π»ΠΈΠ½Π° = 25 x OD 1,45 = 36,25 ΠΌΠΌ 1000 / 36,25 = ΠΎΠΊΠΎΠ»ΠΎ 28 x 2 = 54, Π½Π΅ΠΎΠ±Ρ…ΠΎΠ΄ΠΈΠΌΡ‹Ρ… для фактичСского примСнСния, Ρ‡Ρ‚ΠΎΠ±Ρ‹ ΠΎΠ±Π΅ΡΠΏΠ΅Ρ‡ΠΈΡ‚ΡŒ

ex 2) Π§Ρ‚ΠΎ касаСтся шага,
РСкомСндуСмая Π΄Π»ΠΈΠ½Π° свивки Π΄ΠΎΠ»ΠΆΠ½Π° Π±Ρ‹Ρ‚ΡŒ 25 x OD (= внСшний Π΄ΠΈΠ°ΠΌΠ΅Ρ‚Ρ€)
НапримСр, для 150 ΠΆΠΈΠ» x 0,100 ΠΌΠΌ
OD для 150 x 0,118 ΠΌΠΌ (Π½Π°Ρ€ΡƒΠΆΠ½Ρ‹ΠΉ Π΄ΠΈΠ°ΠΌΠ΅Ρ‚Ρ€, Π²ΠΊΠ»ΡŽΡ‡Π°Ρ ΠΈΠ·ΠΎΠ»ΡΡ†ΠΈΡŽ, gr1 ΠΌΠ΅Π΄Π½ΠΎΠ³ΠΎ ΠΏΡ€ΠΎΠ²ΠΎΠ΄Π° 0,100 ΠΌΠΌ)
приблиТаСтся ΠΊ 1,67 ΠΌΠΌ (√150 x 1,154 x 0,118)
Π­Ρ‚ΠΎ Π΄Π°Π΅Ρ‚ Π΄Π»ΠΈΠ½Ρƒ ΡƒΠΊΠ»Π°Π΄ΠΊΠΈ 41,75 ΠΌΠΌ, Ρ‡Ρ‚ΠΎ равняСтся 23.95 (β‰’ 24) Π²ΠΈΡ‚ΠΊΠΎΠ² Π½Π° ΠΌΠ΅Ρ‚Ρ€, ΠΎΠ΄Π½Π°ΠΊΠΎ Π½Π° самом Π΄Π΅Π»Π΅ Π·Π°ΠΊΠ°Π·Ρ‡ΠΈΠΊΠΈ заявили 48 Π²ΠΈΡ‚ΠΊΠΎΠ² Π½Π° ΠΌΠ΅Ρ‚Ρ€. ΠŸΠΎΡ‚ΠΎΠΌΡƒ Ρ‡Ρ‚ΠΎ ΠΌΠ°Ρ‚Π΅Ρ€ΠΈΠ°Π» Ρ…ΠΎΡ€ΠΎΡˆΠΎ Ρ€Π°Π±ΠΎΡ‚Π°Π΅Ρ‚ ΠΏΡ€ΠΈ 48 скрутках. Π˜Ρ‚Π°ΠΊ, Ρ‚ΠΎ Π΅ΡΡ‚ΡŒ 25 x OD x 2 Ρ€Π°Π·Π°.

2. НаправлСниС ΡƒΠΊΠ»Π°Π΄ΠΊΠΈ (= шаг)

НаТмитС для увСличСния! — Π½Π°ΠΏΡ€Π°Π²Π»Π΅Π½ΠΈΠ΅ ΠΏΡ€ΠΎΠΊΠ»Π°Π΄ΠΊΠΈ Π»ΠΈΡ†Π΅Π²ΠΎΠΉ ΠΏΡ€ΠΎΠ²ΠΎΠ»ΠΎΠΊΠΈ

Π”ΠΎΠ»ΠΆΠ½Ρ‹ ΡΡƒΡ‰Π΅ΡΡ‚Π²ΠΎΠ²Π°Ρ‚ΡŒ ΠΊΠΎΠ½ΠΊΡ€Π΅Ρ‚Π½Ρ‹Π΅ ΠΏΠ°Ρ€Π°ΠΌΠ΅Ρ‚Ρ€Ρ‹ Π»ΠΈΡ‚Ρ†-ΠΏΡ€ΠΎΠ²ΠΎΠ»ΠΎΠΊΠΈ, Π° Ρ‚Π°ΠΊΠΆΠ΅ Π΄Π»ΠΈΠ½Π° свивки (= шаг) ΠΏΠΎ Π½Π°ΠΏΡ€Π°Π²Π»Π΅Π½ΠΈΡŽ Β«SΒ» ΠΈΠ»ΠΈ Β«ZΒ». НаправлСниС ΠΏΡ€ΠΎΠΊΠ»Π°Π΄ΠΊΠΈ ΠΎΠ±Ρ‹Ρ‡Π½ΠΎ ΡƒΠΊΠ°Π·Ρ‹Π²Π°Π΅Ρ‚ Π½Π°ΠΏΡ€Π°Π²Π»Π΅Π½ΠΈΠ΅ скручСнной ΠΈ ΡƒΠ»ΠΎΠΆΠ΅Π½Π½ΠΎΠΉ Π³ΠΈΠ±ΠΊΠΎΠΉ ΠΏΡ€ΠΎΠ²ΠΎΠ»ΠΎΠΊΠΈ Π² Π΄Π²ΡƒΡ… Ρ€Π°Π·Π½Ρ‹Ρ… направлСниях, Ρ‚Π°ΠΊΠΈΡ… ΠΊΠ°ΠΊ Π»Π΅Π²ΠΎΠ΅ Π½Π°ΠΏΡ€Π°Π²Π»Π΅Π½ΠΈΠ΅ Β«SΒ» ΠΈΠ»ΠΈ ΠΏΡ€Π°Π²ΠΎΠ΅ Π½Π°ΠΏΡ€Π°Π²Π»Π΅Π½ΠΈΠ΅ Β«ZΒ».

исх. Участок Π½Π΅Ρ‚. Π΄Π»ΠΈΠ½Ρ‹ Π»ΠΈΡ‚Ρ†-ΠΏΡ€ΠΎΠ²ΠΎΠ»ΠΎΠΊΠΈ ΠΌΠΎΠΆΠ½ΠΎ ΡƒΠΌΠ΅Π½ΡŒΡˆΠΈΡ‚ΡŒ, Ρ‡Ρ‚ΠΎΠ±Ρ‹ ΡƒΠΌΠ΅Π½ΡŒΡˆΠΈΡ‚ΡŒ влияниС Π½Π°Π³Ρ€Π΅Π²Π°.

РасчСт вСса Π½Π΅Ρ‚Ρ‚ΠΎ для Π³ΠΈΠ±ΠΊΠΎΠ³ΠΎ ΠΏΡ€ΠΎΠ²ΠΎΠ΄Π°
● Π³Ρ€Π°ΠΌΠΌ / ΠΌΠ΅Ρ‚Ρ€ = od2 x количСство ΠΆΠΈΠ» x 7
od2: чистый Π΄ΠΈΠ°ΠΌΠ΅Ρ‚Ρ€ + изоляция
7: постоянная ΠΌΠ΅Π΄Π½ΠΎΠ³ΠΎ ΠΏΡ€ΠΎΠ²ΠΎΠ΄Π° (= ΡƒΠ΄Π΅Π»ΡŒΠ½Ρ‹ΠΉ вСс)
FYR, фактичСский ΡƒΠ΄Π΅Π»ΡŒΠ½Ρ‹ΠΉ вСс Cu = 8,92 / Al = 2,71 / Fe = 7,85

РасчСт внСшнСго Π΄ΠΈΠ°ΠΌΠ΅Ρ‚Ρ€Π° Π³ΠΈΠ±ΠΊΠΎΠ³ΠΎ ΠΏΡ€ΠΎΠ²ΠΎΠ΄Π°
● OD (ΠΌΠΌ) = √N x 1,154 x d (ΠΌΠΌ)
N: количСство ΠΏΡ€ΠΎΠ²ΠΎΠ΄Π½ΠΈΠΊΠΎΠ² (Π²ΠΊΠ»ΡŽΡ‡Π°Ρ Ρ‚ΠΎΠ»Ρ‰ΠΈΠ½Ρƒ покрытия, 0080 ΠΌΠΌ -> 0.087 ΠΌΠΌ ΠΈ Ρ‚. Π”.)
d: Π”ΠΈΠ°ΠΌΠ΅Ρ‚Ρ€ ΠΆΠΈΠ»Ρ‹
OD: НаруТный Π΄ΠΈΠ°ΠΌΠ΅Ρ‚Ρ€ Π»ΠΈΡ‚Ρ†ΠΎΠ²ΠΎΠΉ ΠΏΡ€ΠΎΠ²ΠΎΠ»ΠΎΠΊΠΈ

НаруТный Π΄ΠΈΠ°ΠΌΠ΅Ρ‚Ρ€ послС покрытия: ΠΎΠ΄ΠΈΠ½Π°Ρ€Π½Ρ‹ΠΉ (SSC, USTC)
● OD + (0,02–0,04 ΠΌΠΌ) x 2

РасчСт проводимости для прСобразования Cu (мСдь) Π² Al (алюминий)
● ex. Если Π²Π°ΠΌ Π½ΡƒΠΆΠ½ΠΎ Π·Π°ΠΌΠ΅Π½ΠΈΡ‚ΡŒ Cu (0,25 ΠΌΠΌ) Π½Π° Al, Ρ‚ΠΎ ΠΊΠ°ΠΊΠΎΠ² OD Al.
(Cu (0,25 ΠΌΠΌ) Γ· 2) Β² x Ο€ = ΠΏΡ€ΠΈΠ±Π». 0,049㎟
0,049㎟ x 1,61 (ΡƒΠ²Π΅Π»ΠΈΡ‡Π΅Π½ΠΈΠ΅ Π½Π° 61%) = ΠΏΡ€ΠΈΠ±Π». 0,79㎟
Π’Π½Π΅ΡˆΠ½ΠΈΠΉ Π΄ΠΈΠ°ΠΌΠ΅Ρ‚Ρ€ алюминиСвой ΠΏΡ€ΠΎΠ²ΠΎΠ»ΠΎΠΊΠΈ 0.079㎟ (ΠΏΠΎΠΏΠ΅Ρ€Π΅Ρ‡Π½ΠΎΠ΅ сСчСниС) составляСт 0,32 ΠΌΠΌ.

Π’Ρ‹Π±ΠΎΡ€ ΠΊΠ°Π»ΠΈΠ±Ρ€Π° ΠΏΡ€ΠΎΠ²ΠΎΠ»ΠΎΠΊΠΈ Π² качСствС частоты (Π’Π°Π±Π»ΠΈΡ†Π° 2)

Частота Π Π΅ΠΊΠΎΠΌΠ΅Π½Π΄ΡƒΠ΅ΠΌΠΎΠ΅ сСчСниС ΠΏΡ€ΠΎΠ²ΠΎΠ΄Π° OD (ΠΌΠΌ) Π‘ΠΎΠΏΡ€ΠΎΡ‚ΠΈΠ²Π»Π΅Π½ΠΈΠ΅ постоянного Ρ‚ΠΎΠΊΠ° Ом / М ’(макс.) ΠžΠ΄Π½ΠΎΠ½ΠΈΡ‚Π΅Π²ΠΎΠΉ RAC / RDC Β«SΒ»
ΠΎΡ‚ 60 Π“Ρ† Π΄ΠΎ 1 ΠΊΠ“Ρ† 28 AWG 0,32 66,37 1,0000
ΠΎΡ‚ 1 ΠΊΠ“Ρ† Π΄ΠΎ 10 ΠΊΠ“Ρ† 30 AWG 0.25 105,82 1,0000
ΠΎΡ‚ 10 ΠΊΠ“Ρ† Π΄ΠΎ 20 ΠΊΠ“Ρ† 33 AWG 0,18 211,70 1,0000
ΠΎΡ‚ 20 Π΄ΠΎ 50 ΠΊΠ“Ρ† 36 AWG 0,12 431,90 1,0000
ΠΎΡ‚ 50 Π΄ΠΎ 100 ΠΊΠ“Ρ† 38 AWG 0,10 681,90 1,0000
ΠΎΡ‚ 100 Π΄ΠΎ 200 ΠΊΠ“Ρ† 40 AWG 0.08 1152,3 1,0000
ΠΎΡ‚ 200 Π΄ΠΎ 350 ΠΊΠ“Ρ† 42 AWG 0,06 1801,0 1,0000
ΠΎΡ‚ 350 Π΄ΠΎ 850 ΠΊΠ“Ρ† 44 AWG 0,05 2873,0 1,0000
ΠΎΡ‚ 850 ΠΊΠ“Ρ† Π΄ΠΎ 1,4 ΠœΠ“Ρ† 46 AWG 0,04 4544,0 1.0003
ΠΎΡ‚ 1,4 Π΄ΠΎ 2,8 ΠœΠ“Ρ† 48 AWG 0.03 7285,0 1.0003

Π€ΠΎΡ€ΠΌΡƒΠ»Π° ΠΏΠΎΡ‚Π΅Ρ€ΡŒ Π½Π° гистСрСзис (Ph)

НаТмитС, Ρ‡Ρ‚ΠΎΠ±Ρ‹ ΡƒΠ²Π΅Π»ΠΈΡ‡ΠΈΡ‚ΡŒ!
— расчСт ΠΏΠΎΡ‚Π΅Ρ€ΡŒ Π½Π° гистСрСзис

f = частота (Π“Ρ†)
v = объСм сСрдСчника [γŽ₯]
h (постоянная) = коэффициСнт гистСрСзиса
Bm1 * 6 = пСрСмСнная ΠΏΠ»ΠΎΡ‚Π½ΠΎΡΡ‚ΡŒ ΠΌΠ°Π³Π½ΠΈΡ‚Π½ΠΎΠ³ΠΎ ΠΏΠΎΡ‚ΠΎΠΊΠ°

Π€ΠΎΡ€ΠΌΡƒΠ»Π° расчСта ΠΏΠΎΡ‚Π΅Ρ€ΡŒ Π½Π° Π²ΠΈΡ…Ρ€Π΅Π²Ρ‹Π΅ Ρ‚ΠΎΠΊΠΈ (Pe)

НаТмитС, Ρ‡Ρ‚ΠΎΠ±Ρ‹ ΡƒΠ²Π΅Π»ΠΈΡ‡ΠΈΡ‚ΡŒ!
— расчСт ΠΏΠΎΡ‚Π΅Ρ€ΡŒ Π½Π° Π²ΠΈΡ…Ρ€Π΅Π²Ρ‹Π΅ Ρ‚ΠΎΠΊΠΈ

f = частота
k = ΠΏΡ€ΠΎΠ²ΠΎΠ΄ΠΈΠΌΠΎΡΡ‚ΡŒ
t = Ρ‚ΠΎΠ»Ρ‰ΠΈΠ½Π° сСрдцСвины (ΠΎΠ±Ρ‹Ρ‡Π½ΠΎ 0.3 ~ 05 ΠΌΠΌ)
Bm = пСрСмСнная ΠΏΠ»ΠΎΡ‚Π½ΠΎΡΡ‚ΡŒ ΠΌΠ°Π³Π½ΠΈΡ‚Π½ΠΎΠ³ΠΎ ΠΏΠΎΡ‚ΠΎΠΊΠ° (Π½Π°ΠΏΡ€ΠΈΠΌΠ΅Ρ€, 1,6 ~ 2 -> Bm1,6 ~ 2)

НаТмитС, Ρ‡Ρ‚ΠΎΠ±Ρ‹ ΡƒΠ²Π΅Π»ΠΈΡ‡ΠΈΡ‚ΡŒ!
— расчСт ΠΏΠΎΡ‚Π΅Ρ€ΡŒ ΠΆΠ΅Π»Π΅Π·Π°

ΠŸΠΎΡ‚Π΅Ρ€ΠΈ ΠΆΠ΅Π»Π΅Π·Π° = потСря ΠΈΠ·-Π·Π° гистСрСзиса (Ph) + потСря Π½Π° Π²ΠΈΡ…Ρ€Π΅Π²Ρ‹Π΅ Ρ‚ΠΎΠΊΠΈ (Pe)

ΠŸΡ€ΠΎΠ΅ΠΊΡ‚ΠΈΡ€ΠΎΠ²Π°Π½ΠΈΠ΅ ΠΈ расчСт Π»ΠΈΡ†Π΅Π²Ρ‹Ρ… ΠΏΡ€ΠΎΠ²ΠΎΠ΄ΠΎΠ².
Π˜Π½ΠΆΠ΅Π½Π΅Ρ€Ρ‹-ΠΏΡ€ΠΎΠ΅ΠΊΡ‚ΠΈΡ€ΠΎΠ²Ρ‰ΠΈΠΊΠΈ, ΠΈΡΠΏΠΎΠ»ΡŒΠ·ΡƒΡŽΡ‰ΠΈΠ΅ Π»ΠΈΡ‚Ρ†, ΠΎΠ±Ρ‹Ρ‡Π½ΠΎ Π·Π½Π°ΡŽΡ‚ свою Ρ€Π°Π±ΠΎΡ‡ΡƒΡŽ частоту, Ρ‚Ρ€Π΅Π±ΡƒΠ΅ΠΌΡƒΡŽ ΠΏΡ€ΠΈΠ»ΠΎΠΆΠ΅Π½ΠΈΠ΅ΠΌ, ΠΈ срСднСквадратичный Ρ‚ΠΎΠΊ. ОсновноС прСимущСство ΡƒΠΌΠ΅Π½ΡŒΡˆΠ΅Π½ΠΈΡ ΠΏΠΎΡ‚Π΅Ρ€ΡŒ ΠΏΠ΅Ρ€Π΅ΠΌΠ΅Π½Π½ΠΎΠ³ΠΎ Ρ‚ΠΎΠΊΠ° Π² Π»ΠΈΡ†Π΅Π²Ρ‹Ρ… ΠΏΡ€ΠΎΠ²ΠΎΠ΄Π½ΠΈΠΊΠ°Ρ…, ΠΏΠΎΡ‚ΠΎΠΌΡƒ Ρ‡Ρ‚ΠΎ пСрвая ΠΌΡ‹ΡΠ»ΡŒ любого Π»ΠΈΡ†Π΅Π²ΠΎΠ³ΠΎ Π΄ΠΈΠ·Π°ΠΉΠ½Π° — это частота срабатывания.Π­Ρ‚ΠΎ Ρ‚Π°ΠΊΠΆΠ΅, ΠΊΠ°ΠΊ ΠΈ частота срабатывания, влияСт Π½Π° строСниС ΠΏΠΎΠ»Π½ΠΎΡ†Π΅Π½Π½ΠΎΠ³ΠΎ Π»ΠΈΡ†Π° ΠΈΠ½Π΄ΠΈΠ²ΠΈΠ΄ΡƒΡƒΠΌΠ°, опрСдСляСтся ΠΊΠ°Π»ΠΈΠ±Ρ€ΠΎΠΌ ΠΏΡ€ΠΎΠ²ΠΎΠ΄ΠΎΠ². Π‘ΠΎΠΎΡ‚Π½ΠΎΡˆΠ΅Π½ΠΈΠ΅ сопротивлСний постоянному Ρ‚ΠΎΠΊΡƒ Π·Π½Π°Ρ‡Π΅Π½ΠΈΠΉ сопротивлСния ΠΏΠΎΡ‚ΠΎΠΊΡƒ (X) для ΠΈΠ·ΠΎΠ»ΠΈΡ€ΠΎΠ²Π°Π½Π½ΠΎΠ³ΠΎ сплошного ΠΊΡ€ΡƒΠ³Π»ΠΎΠ³ΠΎ ΠΏΡ€ΠΎΠ²ΠΎΠ΄Π° (S) ΠΏΠΎΠΊΠ°Π·Π°Π½ΠΎ Π² Ρ‚Π°Π±Π»ΠΈΡ†Π΅ 1.

Π‘Ρ‚ΠΎΠ»1

X 0 0,5 0,6 0,7 0,8 0,9 1,0
S 1.0000 1.0003 1.0007 1,0012 1,0021 1,0034 1,005

ΠœΠ΅Π΄Π½Ρ‹ΠΉ ΠΏΡ€ΠΎΠ²ΠΎΠ΄, Π·Π½Π°Ρ‡Π΅Π½ΠΈΠ΅ X опрСдСляСтся ΡƒΡ€Π°Π²Π½Π΅Π½ΠΈΠ΅ΠΌ 1.
Π›ΠΈΡ‚Ρ†-структура Π±ΠΎΠ»ΡŒΡˆΠΈΠ½ΡΡ‚Π²Π° Π΄Ρ€ΡƒΠ³ΠΈΡ… Ρ€Π΅Π°Π»ΡŒΠ½Ρ‹Ρ… Π΄Π°Π½Π½Ρ‹Ρ… Π² Ρ‚Π°Π±Π»ΠΈΡ†Π΅ 1 для Ρ€Π΅ΠΊΠΎΠΌΠ΅Π½Π΄ΠΎΠ²Π°Π½Π½ΠΎΠ³ΠΎ ΠΊΠ°Π»ΠΈΠ±Ρ€Π° ΠΏΡ€ΠΎΠ²ΠΎΠ΄Π° для частот Π² ΡΠ»Π΅Π΄ΡƒΡŽΡ‰Π΅ΠΉ Ρ‚Π°Π±Π»ΠΈΡ†Π΅.

НаТмитС, Ρ‡Ρ‚ΠΎΠ±Ρ‹ ΡƒΠ²Π΅Π»ΠΈΡ‡ΠΈΡ‚ΡŒ!

Если Π°Ρ€Ρ…ΠΈΡ‚Π΅ΠΊΡ‚ΡƒΡ€Π½Ρ‹ΠΉ ΠΏΡ€ΠΎΠ΅ΠΊΡ‚ Π±Ρ‹Π» Π±Ρ‹ принят, Ρ‚ΠΎ Π±Ρ‹Π»Π° ΠΎΠΏΡ€Π΅Π΄Π΅Π»Π΅Π½Π° шовная конструкция ΠΈ ΠΈΠ½Π΄ΠΈΠ²ΠΈΠ΄ΡƒΠ°Π»ΡŒΠ½Ρ‹ΠΉ ΠΊΠ°Π»ΠΈΠ±Ρ€ ΠΏΡ€ΠΎΠ²ΠΎΠ»ΠΎΠΊΠΈ.КаТдая ΠΏΡ€ΡΠ΄ΡŒ ΠΈΠΌΠ΅Π΅Ρ‚ Ρ‚Π΅Π½Π΄Π΅Π½Ρ†ΠΈΡŽ Π±Ρ‹Ρ‚ΡŒ засСлСнными ΠΏΠΎΡ‡Ρ‚ΠΈ Π² ΠΎΠ΄ΠΈΠ½Π°ΠΊΠΎΠ²ΠΎΠΉ стСпСни Π²ΠΎ всСх Π²ΠΎΠ·ΠΌΠΎΠΆΠ½Ρ‹Ρ… полоТСниях кабСля. Дистанционный Π»ΠΈΡ‚Ρ†-ΠΏΡ€ΠΎΠ²ΠΎΠ΄Π½ΠΈΠΊ D / C ΠΎΡ‚Π½ΠΎΡˆΠ΅Π½ΠΈΠ΅ сопротивлСния ΠΏΠ΅Ρ€Π΅ΠΌΠ΅Π½Π½ΠΎΠ³ΠΎ Ρ‚ΠΎΠΊΠ° ΠΌΠΎΠΆΠ΅Ρ‚ Π±Ρ‹Ρ‚ΡŒ ΠΎΠΏΡ€Π΅Π΄Π΅Π»Π΅Π½ ΠΏΠΎ ΡΠ»Π΅Π΄ΡƒΡŽΡ‰Π΅ΠΌΡƒ ΡƒΡ€Π°Π²Π½Π΅Π½ΠΈΡŽ.

НаТмитС, Ρ‡Ρ‚ΠΎΠ±Ρ‹ ΡƒΠ²Π΅Π»ΠΈΡ‡ΠΈΡ‚ΡŒ!

Π“Π΄Π΅: S = коэффициСнт сопротивлСния ΠΎΡ‚Π΄Π΅Π»ΡŒΠ½Ρ‹Ρ… ΠΆΠΈΠ» Π² ΠΈΠ·ΠΎΠ»ΠΈΡ€ΠΎΠ²Π°Π½Π½ΠΎΠΌ состоянии (взято ΠΈΠ· Ρ‚Π°Π±Π»ΠΈΡ† 1 ΠΈΠ»ΠΈ 2)
G = базисный коэффициСнт Π²ΠΈΡ…Ρ€Π΅Π²Ρ‹Ρ… Ρ‚ΠΎΠΊΠΎΠ² =

НаТмитС, Ρ‡Ρ‚ΠΎΠ±Ρ‹ ΡƒΠ²Π΅Π»ΠΈΡ‡ΠΈΡ‚ΡŒ!

F = рабочая частота Π² Π“Ρ†
N = количСство ΠΆΠΈΠ» Π² ΠΊΠ°Π±Π΅Π»Π΅
D1 = Π΄ΠΈΠ°ΠΌΠ΅Ρ‚Ρ€ ΠΎΡ‚Π΄Π΅Π»ΡŒΠ½Ρ‹Ρ… ΠΆΠΈΠ» ΠΏΠΎ ΠΌΠ΅Π΄ΠΈ Π² Π΄ΡŽΠΉΠΌΠ°Ρ…
DO = Π΄ΠΈΠ°ΠΌΠ΅Ρ‚Ρ€ Π³ΠΎΡ‚ΠΎΠ²ΠΎΠ³ΠΎ кабСля ΠΏΠΎ ΠΆΠΈΠ»Π°ΠΌ Π² Π΄ΡŽΠΉΠΌΠ°Ρ…
K = постоянный Π² зависимости ΠΎΡ‚ N, ΠΏΡ€ΠΈΠ²Π΅Π΄Π΅Π½ΠΎ Π² ΡΠ»Π΅Π΄ΡƒΡŽΡ‰Π΅ΠΉ Ρ‚Π°Π±Π»ΠΈΡ†Π΅:

N 3 9 27 Π±Π΅ΡΠΊΠΎΠ½Π΅Ρ‡Π½ΠΎΡΡ‚ΡŒ
К 1.55 1,84 1,92 2

Π‘ΠΎΠΏΡ€ΠΎΡ‚ΠΈΠ²Π»Π΅Π½ΠΈΠ΅ Π»ΠΈΡ‚Ρ†-ΠΏΡ€ΠΎΠ²ΠΎΠ΄Π½ΠΈΠΊΠ° постоянному Ρ‚ΠΎΠΊΡƒ связано со ΡΠ»Π΅Π΄ΡƒΡŽΡ‰ΠΈΠΌΠΈ ΠΏΠ°Ρ€Π°ΠΌΠ΅Ρ‚Ρ€Π°ΠΌΠΈ:
1) ΠžΡ‚Π΄Π΅Π»ΡŒΠ½Ρ‹Π΅ ΠΆΠΈΠ»Ρ‹ AWG.
2) ΠšΠΎΠ»ΠΈΡ‡Π΅ΡΡ‚Π²ΠΎ ΠΆΠΈΠ» кабСля.
3) Π€Π°ΠΊΡ‚ΠΎΡ€Ρ‹, связанныС с ΡƒΠ²Π΅Π»ΠΈΡ‡Π΅Π½ΠΈΠ΅ΠΌ Π΄Π»ΠΈΠ½Ρ‹ Π½Π° ΠΊΠ°ΠΆΠ΄ΡƒΡŽ ΠΏΡ€ΡΠ΄ΡŒ (ΠΏΠ΅Ρ€Π΅Π΄Π½ΡŽΡŽ) Π΅Π΄ΠΈΠ½ΠΈΡ†Ρƒ Π΄Π»ΠΈΠ½Ρ‹ кабСля. ΠŸΡ€ΠΈΠΌΠ΅Ρ€Π½ΠΎ Π½Π° 2,5% ΡƒΠ²Π΅Π»ΠΈΡ‡Π΅Π½ΠΎ сопротивлСниС постоянному Ρ‚ΠΎΠΊΡƒ, сопротивлСниС постоянному Ρ‚ΠΎΠΊΡƒ для всСх Π·Π°Π΄Π°Ρ‡ связки для стандартных структур Ritz ΠΈ ΡƒΠ²Π΅Π»ΠΈΡ‡Π΅Π½ΠΎ Π½Π° 1,5% для всСх ΠΊΠ°Π±Π΅Π»Π΅ΠΉ, Ρ‡Ρ‚ΠΎΠ±Ρ‹ ΡƒΠ±Π΅Π΄ΠΈΡ‚ΡŒΡΡ, Ρ‡Ρ‚ΠΎ ΠΎΠ½ΠΈ ΠΏΡ€Π°Π²ΠΈΠ»ΡŒΠ½Ρ‹Π΅.Π‘ΠΎΠΏΡ€ΠΎΡ‚ΠΈΠ²Π»Π΅Π½ΠΈΠ΅ постоянному Ρ‚ΠΎΠΊΡƒ конструкции ΠΏΠΎ любой Π»ΠΈΡ‚Ρ†-Ρ„ΠΎΡ€ΠΌΡƒΠ»Π΅ получаСтся ΠΈΠ· ΠΏΠ°Ρ€Π°ΠΌΠ΅Ρ‚Ρ€ΠΎΠ²:

НаТмитС, Ρ‡Ρ‚ΠΎΠ±Ρ‹ ΡƒΠ²Π΅Π»ΠΈΡ‡ΠΈΡ‚ΡŒ!

НиТС ΠΏΡ€ΠΈΠ²Π΅Π΄Π΅Π½ ΠΏΡ€ΠΈΠΌΠ΅Ρ€ расчСтов, Π½Π΅ΠΎΠ±Ρ…ΠΎΠ΄ΠΈΠΌΡ‹Ρ… для ΠΎΡ†Π΅Π½ΠΊΠΈ конструкции ΠΎΠ΄Π½ΠΎΠΏΠ»Π΅Π½ΠΎΡ‡Π½ΠΎΠ³ΠΎ ΠΏΠΎΠ»ΠΈΡƒΡ€Π΅Ρ‚Π°Π½ΠΎΠ²ΠΎΠ³ΠΎ покрытия ΠΈΠ· ΠΆΠΈΠ»Ρ‹ ΠΏΡ€ΠΎΠ²ΠΎΠ»ΠΎΠΊΠΈ 38 AWG Litz, состоящСй ΠΈΠ· 400 ΠΏΡ€ΠΎΠ²ΠΎΠ΄ΠΎΠ², Ρ€Π°Π±ΠΎΡ‚Π°ΡŽΡ‰ΠΈΡ… Π½Π° частотС 500 ΠΊΠ“Ρ†. Π—Π°ΠΏΠΈΡˆΠ΅ΠΌ эту ΠΊΠΎΠ½ΡΡ‚Ρ€ΡƒΠΊΡ†ΠΈΡŽ, ΠΆΠ³ΡƒΡ‚ ΠΏΡ€ΠΎΠ²ΠΎΠ΄ΠΎΠ² Π΄Π²Π° 5 Γ— 5/40 16 AWG

1) РассчитанноС ΠΏΠΎ Ρ„ΠΎΡ€ΠΌΡƒΠ»Π΅ 3 сопротивлСниС конструкции Π»ΠΈΡ‚Ρ† постоянному Ρ‚ΠΎΠΊΡƒ.

НаТмитС, Ρ‡Ρ‚ΠΎΠ±Ρ‹ ΡƒΠ²Π΅Π»ΠΈΡ‡ΠΈΡ‚ΡŒ!

2) Π‘ΠΎΠΎΡ‚Π½ΠΎΡˆΠ΅Π½ΠΈΠ΅ сопротивлСний постоянному ΠΈ ΠΏΠ΅Ρ€Π΅ΠΌΠ΅Π½Π½ΠΎΠΌΡƒ Ρ‚ΠΎΠΊΡƒ рассчитываСтся ΠΏΠΎ Ρ„ΠΎΡ€ΠΌΡƒΠ»Π΅ 2.

НаТмитС, Ρ‡Ρ‚ΠΎΠ±Ρ‹ ΡƒΠ²Π΅Π»ΠΈΡ‡ΠΈΡ‚ΡŒ!

3) Π‘ΠΎΠΏΡ€ΠΎΡ‚ΠΈΠ²Π»Π΅Π½ΠΈΠ΅ ΠΏΠ΅Ρ€Π΅ΠΌΠ΅Π½Π½ΠΎΠΌΡƒ Ρ‚ΠΎΠΊΡƒ, ΡΠ»Π΅Π΄ΠΎΠ²Π°Ρ‚Π΅Π»ΡŒΠ½ΠΎ, составляСт 1,2068 ΠΈΠ»ΠΈ 1,80 Ом / 1000 Ρ„ΡƒΡ‚ΠΎΠ² (= 304,8 ΠΌ).

ΠŸΡ€ΠΎΠΈΠ·Π²ΠΎΠ΄ΠΈΡ‚Π΅Π»ΡŽ ΠΏΡ€ΠΎΠ²ΠΎΠ»ΠΎΠΊΠΈ

Litz Π±Ρ‹Π»ΠΎ ΠΏΡ€Π΅Π΄Π»ΠΎΠΆΠ΅Π½ΠΎ ΠΏΡ€Π΅Π΄ΠΎΡΡ‚Π°Π²ΠΈΡ‚ΡŒ Ρ€Π°Π·ΠΌΠ΅Ρ€ AWG Ρ‚ΠΎΠ»ΡŒΠΊΠΎ Π² ΠΌΠΈΠ»Π»ΠΈΠΌΠ΅Ρ‚Ρ€ΠΎΠ²ΠΎΠΉ (ΠΌΠΈΠ»Π»ΠΈΠΌΠ΅Ρ‚Ρ€ΠΎΠ²ΠΎΠΉ) шкалС. НапримСр, AWG36 = 0,127 ΠΌΠΌ (Π²ΠΊΠ»ΡŽΡ‡Π°Ρ Π½Π°Ρ€ΡƒΠΆΠ½Ρ‹ΠΉ Π΄ΠΈΠ°ΠΌΠ΅Ρ‚Ρ€, ΠΏΠΎΠΊΡ€Ρ‹Ρ‚Ρ‹ΠΉ эмалью), Ρ‚ΠΎ Π΅ΡΡ‚ΡŒ ΠΏΠ»ΠΎΡ‰Π°Π΄ΡŒ ΠΎΠ΄Π½ΠΎΠΏΡ€ΠΎΠ²ΠΎΠ»ΠΎΡ‡Π½ΠΎΠ³ΠΎ сСчСния (= 0,0126677 ΠΌΠΌ ΠΊΠ².), И Ссли ΠΌΡ‹ ΠΈΡΠΏΠΎΠ»ΡŒΠ·ΡƒΠ΅ΠΌ Π°Π½Π°Π»ΠΎΠ³ΠΈΡ‡Π½ΡƒΡŽ ΠΏΠ»ΠΎΡ‰Π°Π΄ΡŒ одноТильного сСчСния 0,120 ΠΌΠΌ (= 0,0113097 ΠΊΠ². Мм) AWG36. Когда Π·Π°ΠΊΠ°Π·Ρ‡ΠΈΠΊ Π·Π°ΠΏΡ€Π°ΡˆΠΈΠ²Π°Π΅Ρ‚ ΠΆΠΈΠ»Ρ‹ AWG36 x 1000, общая ΠΏΠ»ΠΎΡ‰Π°Π΄ΡŒ сСчСния Π΄Π°Π΅Ρ‚ 0.0126677 ΠΊΠ². Мм x 1000 Π½ΠΈΡ‚Π΅ΠΉ = 12,6677 ΠΊΠ². Мм

Если ΠΌΡ‹ ΠΈΡΠΏΠΎΠ»ΡŒΠ·ΡƒΠ΅ΠΌ ΠΏΡ€ΠΎΠ²ΠΎΠ»ΠΎΠΊΡƒ 0,120 ΠΌΠΌ, ΠΊΠ²Π°Π΄Ρ€Π°Ρ‚ 12,6677 ΠΌΠΌ / 0,0113097 ΠΌΠΌ = 1120 прядСй.
Π‘Π»Π΅Π΄ΠΎΠ²Π°Ρ‚Π΅Π»ΡŒΠ½ΠΎ, ΠΌΡ‹ ΠΌΠΎΠΆΠ΅ΠΌ ΠΈΡΠΏΠΎΠ»ΡŒΠ·ΠΎΠ²Π°Ρ‚ΡŒ Π»ΠΈΠ±ΠΎ AWG36 x 1000 ΠΆΠΈΠ», Π»ΠΈΠ±ΠΎ 0,120 ΠΌΠΌ x 1120 ΠΆΠΈΠ» для ΠΏΡ€ΠΎΠ²ΠΎΠ΄Π°, Ρ‡Ρ‚ΠΎΠ±Ρ‹ ΠΈΠΌΠ΅Ρ‚ΡŒ Ρ‚Ρƒ ΠΆΠ΅ ΠΏΡ€ΠΎΠΏΡƒΡΠΊΠ½ΡƒΡŽ ΡΠΏΠΎΡΠΎΠ±Π½ΠΎΡΡ‚ΡŒ ΠΏΠΎ Ρ‚ΠΎΠΊΡƒ, Π½ΠΎ 0,120 ΠΌΠΌ x 1120 ΠΆΠΈΠ» ΠΏΠΎΠΊΠ°ΠΆΡƒΡ‚ Π±ΠΎΠ»Π΅Π΅ Π»ΡƒΡ‡ΡˆΠΈΠ΅ характСристики скин-эффСкта, Ρ‡Π΅ΠΌ AWG36 x 1000 ΠΆΠΈΠ», ΠΏΠΎΡ‚ΠΎΠΌΡƒ Ρ‡Ρ‚ΠΎ ΠΎΠ΄ΠΈΠ½ΠΎΡ‡Π½Ρ‹Π΅ 0,120 ΠΌΠΌ ΠΈΠΌΠ΅Π΅Ρ‚ Π»ΡƒΡ‡ΡˆΠΈΠΉ скин-эффСкт, Ρ‡Π΅ΠΌ 0,127 ΠΌΠΌ (AWG36).

1000 Π½ΠΈΡ‚Π΅ΠΉ x AWG36 Π΄Π°Π΅Ρ‚ 1120 Π½ΠΈΡ‚Π΅ΠΉ ΠΈΠ· 0.120 ΠΌΠΌ (ΠΈΠ»ΠΈ ΠΎΠΊΠΎΠ»ΠΎ 1125 Π½ΠΈΡ‚Π΅ΠΉ = 5 Ρ… 5 Ρ… 45 Π½ΠΈΡ‚Π΅ΠΉ).

UL Π³ΠΎΠ²ΠΎΡ€ΠΈΡ‚, Ρ‡Ρ‚ΠΎ Β«ΠΏΠΎΠ²Ρ‹ΡˆΠ΅Π½ΠΈΠ΅ Ρ‚Π΅ΠΌΠΏΠ΅Ρ€Π°Ρ‚ΡƒΡ€Ρ‹Β» Π΄ΠΎΠ»ΠΆΠ½ΠΎ ΡΠΎΡΡ‚Π°Π²Π»ΡΡ‚ΡŒ 75 градусов. C мСньшС Π½Π° классС A ΠΈ 95 Π³Ρ€Π°Π΄. C мСньшС Π² классС B, ΠΈ ΠΌΡ‹ часто сталкиваСмся с Β«ΠΏΡ€ΠΎΠ±Π»Π΅ΠΌΠΎΠΉ ΠΏΠΎΠ²Ρ‹ΡˆΠ΅Π½ΠΈΡ Ρ‚Π΅ΠΌΠΏΠ΅Ρ€Π°Ρ‚ΡƒΡ€Ρ‹Β», ΠΊΠΎΠ³Π΄Π° Ρ€Π°Π·Ρ€Π°Π±Π°Ρ‚Ρ‹Π²Π°Π΅ΠΌ трансформатор Π½Π° этапС исслСдований ΠΈ Ρ€Π°Π·Ρ€Π°Π±ΠΎΡ‚ΠΎΠΊ. Π§Ρ‚ΠΎΠ±Ρ‹ Ρ€Π΅ΡˆΠΈΡ‚ΡŒ Β«ΠΏΠΎΠ²Ρ‹ΡˆΠ΅Π½ΠΈΠ΅ Ρ‚Π΅ΠΌΠΏΠ΅Ρ€Π°Ρ‚ΡƒΡ€Ρ‹ (= Π΄Π΅Π»ΡŒΡ‚Π° T, Ρ‡Ρ‚ΠΎ ΠΎΠ·Π½Π°Ρ‡Π°Π΅Ρ‚ ΠΈΡΠΊΠ»ΡŽΡ‡Π΅Π½Π½ΡƒΡŽ Ρ‚Π΅ΠΌΠΏΠ΅Ρ€Π°Ρ‚ΡƒΡ€Ρƒ ΠΎΠΊΡ€ΡƒΠΆΠ°ΡŽΡ‰Π΅ΠΉ срСды)Β», ΠΌΡ‹ ΠΌΠΎΠΆΠ΅ΠΌ ΡƒΠΌΠ΅Π½ΡŒΡˆΠΈΡ‚ΡŒ ΠΏΡ€Π΅Π²Ρ‹ΡˆΠ΅Π½ΠΈΠ΅ ΠΏΡ€Π΅Π²Ρ‹ΡˆΠ΅Π½ΠΈΡ высокой Ρ‚Π΅ΠΌΠΏΠ΅Ρ€Π°Ρ‚ΡƒΡ€Ρ‹ Π² соотвСтствии со стандартом UL, Ρ‚Π°ΠΊ ΠΆΠ΅ ΠΊΠ°ΠΊ ΠΈ ΠΏΡ€ΠΈ ΡΡ‚Ρ€ΠΎΠΈΡ‚Π΅Π»ΡŒΡΡ‚Π²Π΅ большого количСства ΠΆΠΈΠ».

1. ΠœΠ΅Ρ‚ΠΎΠ΄ ΠΏΠΎΠ²Ρ‹ΡˆΠ΅Π½ΠΈΡ «добротности»
Π’ Ρ„ΠΈΠ·ΠΈΠΊΠ΅ ΠΈ Ρ‚Π΅Ρ…Π½ΠΈΠΊΠ΅ Π΄ΠΎΠ±Ρ€ΠΎΡ‚Π½ΠΎΡΡ‚ΡŒ ΠΈΠ»ΠΈ Π΄ΠΎΠ±Ρ€ΠΎΡ‚Π½ΠΎΡΡ‚ΡŒ являСтся Π±Π΅Π·Ρ€Π°Π·ΠΌΠ΅Ρ€Π½Ρ‹ΠΌ ΠΏΠ°Ρ€Π°ΠΌΠ΅Ρ‚Ρ€ΠΎΠΌ.Π­Ρ‚ΠΎ Ρ€Π΅ΡˆΠ°Π΅Ρ‚ΡΡ ΠΏΡƒΡ‚Π΅ΠΌ дСмонстрации Ρ‚ΠΎΠ³ΠΎ, слСдуСт Π»ΠΈ ΡΠΎΠΎΠ±Ρ‰Π°Ρ‚ΡŒ Π½Π°ΠΌ Π΅Π³ΠΎ статус, ΡΠ»Π°Π±ΠΎΠ·Π°Ρ‚ΡƒΡ…Π°ΡŽΡ‰ΡƒΡŽ Π²ΠΈΠ±Ρ€Π°Ρ†ΠΈΡŽ ΠΈΠ»ΠΈ Ρ€Π΅Π·ΠΎΠ½Π°Ρ‚ΠΎΡ€. Π’Π°ΠΊΠΆΠ΅ опрСдСляСтся ΡˆΠΈΡ€ΠΈΠ½Π° полосы Ρ€Π΅Π·ΠΎΠ½Π°Ρ‚ΠΎΡ€Π° ΠΏΠΎ Ρ†Π΅Π½Ρ‚Ρ€Π°Π»ΡŒΠ½ΠΎΠΉ частотС. Высокая Π΄ΠΎΠ±Ρ€ΠΎΡ‚Π½ΠΎΡΡ‚ΡŒ ΠΎΠ·Π½Π°Ρ‡Π°Π΅Ρ‚ мСньшиС ΠΏΠΎΡ‚Π΅Ρ€ΠΈ энСргии, Ρ‡Π΅ΠΌ энСргия, запасСнная Π² осцилляторС, Π° Ρ‚Π°ΠΊΠΆΠ΅ постСпСнно ΡƒΠΌΠ΅Π½ΡŒΡˆΠ°Π΅Ρ‚ΡΡ вибрация, ΠΊΠ°ΠΊ Π² этом случаС. Π’ΠΈΠ±Ρ€ΠΈΡ€ΡƒΡŽΡ‰ΠΈΠΉ маятник Π² Π²ΠΎΠ·Π΄ΡƒΡ…Π΅, Π½Π° ΠΊΠΎΡ‚ΠΎΡ€ΠΎΠΌ висит высококачСствСнный подшипник, ΠΈΠΌΠ΅Π΅Ρ‚ Π²Ρ‹ΡΠΎΠΊΡƒΡŽ Π΄ΠΎΠ±Ρ€ΠΎΡ‚Π½ΠΎΡΡ‚ΡŒ. Π‘ Π΄Ρ€ΡƒΠ³ΠΎΠΉ стороны, Π²ΠΈΠ±Ρ€ΠΈΡ€ΡƒΡŽΡ‰ΠΈΠΉ маятник, ΠΏΠΎΠ³Ρ€ΡƒΠΆΠ΅Π½Π½Ρ‹ΠΉ Π² масло, ΠΈΠΌΠ΅Π΅Ρ‚ Π½ΠΈΠ·ΠΊΡƒΡŽ Π΄ΠΎΠ±Ρ€ΠΎΡ‚Π½ΠΎΡΡ‚ΡŒ. Π“Π΅Π½Π΅Ρ€Π°Ρ‚ΠΎΡ€ с высокой Π΄ΠΎΠ±Ρ€ΠΎΡ‚Π½ΠΎΡΡ‚ΡŒΡŽ ΠΈΠΌΠ΅Π΅Ρ‚ Π±ΠΎΠ»Π΅Π΅ Π½ΠΈΠ·ΠΊΠΎΠ΅ Ρ‚ΠΎΡ€ΠΌΠΎΠΆΠ΅Π½ΠΈΠ΅ ΠΈ Π±ΠΎΠ»Π΅Π΅ Π΄Π»ΠΈΡ‚Π΅Π»ΡŒΠ½ΡƒΡŽ Π²ΠΈΠ±Ρ€Π°Ρ†ΠΈΡŽ.

Для увСличСния значСния добротности Π³ΠΈΠ±ΠΊΠΎΠ³ΠΎ ΠΏΡ€ΠΎΠ²ΠΎΠ΄Π° ΠΈ кабСля, Π²Ρ‹ Π΄ΠΎΠ»ΠΆΠ½Ρ‹ ΠΏΠ»ΠΎΡ‚Π½ΠΎ Π½Π°ΠΌΠΎΡ‚Π°Ρ‚ΡŒ Π»ΠΈΡ‚Ρ†Π΅Π²Ρ‹ΠΉ ΠΏΡ€ΠΎΠ²ΠΎΠ΄, склССнный вмСстС, Ρ‡Ρ‚ΠΎΠ±Ρ‹ ΠΎΠ±Π΅ΡΠΏΠ΅Ρ‡ΠΈΡ‚ΡŒ максимальноС сцСплСниС ΠΌΠ΅ΠΆΠ΄Ρƒ ΠΌΠ΅Π΄Π½Ρ‹ΠΌ ΠΏΡ€ΠΎΠ²ΠΎΠ΄ΠΎΠΌ ΠΈ ΠΌΠ΅Π΄Π½Ρ‹ΠΌ ΠΏΡ€ΠΎΠ²ΠΎΠ΄ΠΎΠΌ, ΠΊΠ°ΠΊ ΠΏΠΎΠΊΠ°Π·Π°Π½ΠΎ Π½Π° ΡΠ»Π΅Π΄ΡƒΡŽΡ‰Π΅ΠΌ рисункС, исх. 1.

ΠΌΠ΅Ρ‚ΠΎΠ΄ ΠΏΠΎΠ²Ρ‹ΡˆΠ΅Π½ΠΈΡ добротности

Когда ΠΌΡ‹ Ρ€Π°Π·Ρ€Π°Π±Π°Ρ‚Ρ‹Π²Π°Π΅ΠΌ элСктронныС ΠΏΡ€ΠΎΠ΄ΡƒΠΊΡ‚Ρ‹, Q-Ρ„Π°ΠΊΡ‚ΠΎΡ€ являСтся Π²Π°ΠΆΠ½ΠΎΠΉ ΠΏΠ΅Ρ€Π΅ΠΌΠ΅Π½Π½ΠΎΠΉ. Он опрСдСляСт ΡˆΠΈΡ€ΠΈΠ½Ρƒ полосы Ρ€Π΅Π·ΠΎΠ½Π°Ρ‚ΠΎΡ€Π° Π² соотвСтствии с Ρ†Π΅Π½Ρ‚Ρ€Π°Π»ΡŒΠ½ΠΎΠΉ частотой, Π° Ρ‚Π°ΠΊΠΆΠ΅ Π²Ρ‹ΡΠΎΠΊΡƒΡŽ Π΄ΠΎΠ±Ρ€ΠΎΡ‚Π½ΠΎΡΡ‚ΡŒ, которая Π΄ΠΎΠ»ΠΆΠ½Π° Π±Ρ‹Ρ‚ΡŒ спроСктирована для сниТСния ΠΏΠΎΡ‚Π΅Ρ€ΡŒ энСргии ΠΏΠΎ ΡΡ€Π°Π²Π½Π΅Π½ΠΈΡŽ с энСргиСй, хранящСйся Π² Π²ΠΈΠ±Ρ€Π°Ρ‚ΠΎΡ€Π΅.Π”Ρ€ΡƒΠ³ΠΈΠΌΠΈ словами, сниТСниС Π²ΠΈΠ±Ρ€Π°Ρ†ΠΈΠΈ Π΄ΠΎΠ»ΠΆΠ½ΠΎ Π±Ρ‹Ρ‚ΡŒ ΠΌΠ΅Π΄Π»Π΅Π½Π½Ρ‹ΠΌ.

Q = X / R = øL / R = 2∏fL / R
— Q: Π΄ΠΎΠ±Ρ€ΠΎΡ‚Π½ΠΎΡΡ‚ΡŒ
— X: Ρ€Π΅Π°ΠΊΡ‚ΠΈΠ²Π½ΠΎΠ΅ сопротивлСниС опрСдСляСтся Π·Π½Π°Ρ‡Π΅Π½ΠΈΠ΅ΠΌ сопротивлСния индуктивности ΠΊΠ°Ρ‚ΡƒΡˆΠΊΠΈ
— R: сопротивлСниС опрСдСляСтся Π·Π½Π°Ρ‡Π΅Π½ΠΈΠ΅ΠΌ сопротивлСния ΠΊΠ°Ρ‚ΡƒΡˆΠΊΠΈ.
— f: рСзонансная частота.
— ∏: круговая постоянная (ΠΏΠΈ), 3,14…

2. ΠœΠ΅Ρ‚ΠΎΠ΄ увСличСния «значСния индуктивности» ΡΠ»Π΅Π΄ΡƒΡŽΡ‰ΠΈΠΉ.
Π˜Π·Π³ΠΎΡ‚ΠΎΠ²Π»Π΅Π½ΠΈΠ΅ ΠΏΡ€ΠΎΠ²ΠΎΠ»ΠΎΠΊΠΈ ΠΊΠ°ΠΊ ΠΌΠΎΠΆΠ½ΠΎ Π±ΠΎΠ»Π΅Π΅ ΠΏΠ»ΠΎΡ‚Π½ΠΎ ΠΏΡ€ΠΈΠ»Π΅Π³Π°ΡŽΡ‰Π΅ΠΉ.ΠŸΠ»ΠΎΡ‚Π½ΠΎΡΡ‚ΡŒ ΠΌΠ°Π³Π½ΠΈΡ‚Π½ΠΎΠ³ΠΎ ΠΏΠΎΡ‚ΠΎΠΊΠ° удваиваСтся Π·Π° счСт размСщСния Ρ„Π΅Ρ€Ρ€ΠΎΠΌΠ°Π³Π½ΠΈΡ‚Π½ΠΎΠ³ΠΎ сСрдСчника Π²ΠΎ Π²Π½ΡƒΡ‚Ρ€Π΅Π½Π½Π΅ΠΉ ΠΊΠ°Ρ‚ΡƒΡˆΠΊΠ΅. Π£Π²Π΅Π»ΠΈΡ‡Π΅Π½ΠΈΠ΅ плотности ΠΌΠ°Π³Π½ΠΈΡ‚Π½ΠΎΠ³ΠΎ ΠΏΠΎΡ‚ΠΎΠΊΠ° Ρ‚Π°ΠΊΠΆΠ΅ ΠΏΡ€ΠΈΠ²ΠΎΠ΄ΠΈΡ‚ ΠΊ ΡƒΠ²Π΅Π»ΠΈΡ‡Π΅Π½ΠΈΡŽ индуктивности. Π‘Π»Π΅Π΄ΠΎΠ²Π°Ρ‚Π΅Π»ΡŒΠ½ΠΎ, Π·Π½Π°Ρ‡Π΅Π½ΠΈΠ΅ индуктивности Ρ„Π΅Ρ€Ρ€ΠΎΠΌΠ°Π³Π½ΠΈΡ‚Π½ΠΎΠ³ΠΎ сСрдСчника Π² нСсколько Ρ€Π°Π· большС, Ρ‡Π΅ΠΌ Ρƒ Π²ΠΎΠ·Π΄ΡƒΡˆΠ½ΠΎΠΉ ΠΊΠ°Ρ‚ΡƒΡˆΠΊΠΈ ΠΈΠ»ΠΈ Π½Π΅ΠΌΠ°Π³Π½ΠΈΡ‚Π½ΠΎΠ³ΠΎ сСрдСчника, Ρ‚Π°ΠΊΠΎΠ³ΠΎ ΠΊΠ°ΠΊ пластик, Π΄Π΅Ρ€Π΅Π²ΠΎ ΠΈ Ρ‚. Π”. Π’Π΅Π»ΠΈΡ‡ΠΈΠ½Π° индуктивности зависит ΠΎΡ‚ количСства ΠΎΠ±ΠΎΡ€ΠΎΡ‚ΠΎΠ² ΠΎΠ±ΠΌΠΎΡ‚ΠΊΠΈ ΠΊΠ°Ρ‚ΡƒΡˆΠΊΠΈ, Π΄ΠΈΠ°ΠΌΠ΅Ρ‚Ρ€Π° ΠΊΠ°Ρ‚ΡƒΡˆΠΊΠΈ ΠΈ ΠΎΠ±Ρ‰Π΅Π³ΠΎ Ρ„ΠΎΡ€ΠΌΠ° ΠΊΠ°Ρ‚ΡƒΡˆΠΊΠΈ. Π˜Π½Π΄ΡƒΠΊΡ‚ΠΈΠ²Π½ΠΎΡΡ‚ΡŒ ΠΊΠ°Ρ‚ΡƒΡˆΠΊΠΈ прямо ΠΏΡ€ΠΎΠΏΠΎΡ€Ρ†ΠΈΠΎΠ½Π°Π»ΡŒΠ½Π° скорости вращСния (числу Π²ΠΈΡ‚ΠΊΠΎΠ²) Π½Π°ΠΌΠΎΡ‚Π°Π½Π½ΠΎΠ³ΠΎ ΠΏΡ€ΠΎΠ²ΠΎΠ΄Π° ΠΈ прямо ΠΏΡ€ΠΎΠΏΠΎΡ€Ρ†ΠΈΠΎΠ½Π°Π»ΡŒΠ½Π° индуктивности Π΄ΠΈΠ°ΠΌΠ΅Ρ‚Ρ€Π° ΠΊΠ°Ρ‚ΡƒΡˆΠΊΠΈ.Π’ΠΎΡ‡Π½Π΅Π΅, ΠΈΠ½Π΄ΡƒΠΊΡ‚ΠΈΠ²Π½ΠΎΡΡ‚ΡŒ ΠΊΠ°Ρ‚ΡƒΡˆΠΊΠΈ солСноида Π½Π° Π΅Π΄ΠΈΠ½ΠΈΡ†Ρƒ Π΄Π»ΠΈΠ½Ρ‹ прямо ΠΏΡ€ΠΎΠΏΠΎΡ€Ρ†ΠΈΠΎΠ½Π°Π»ΡŒΠ½Π° ΠΏΠ»ΠΎΡ‰Π°Π΄ΠΈ ΠΏΠΎΠΏΠ΅Ρ€Π΅Ρ‡Π½ΠΎΠ³ΠΎ сСчСния ΠΈ прямо ΠΏΡ€ΠΎΠΏΠΎΡ€Ρ†ΠΈΠΎΠ½Π°Π»ΡŒΠ½Π° ΠΊΠ²Π°Π΄Ρ€Π°Ρ‚Ρƒ ΠΎΠ±ΠΎΡ€ΠΎΡ‚ΠΎΠ² Π½Π°ΠΌΠΎΡ‚Π°Π½Π½Ρ‹Ρ… ΠΏΡ€ΠΎΠ²ΠΎΠ΄ΠΎΠ² Π½Π° Π΅Π΄ΠΈΠ½ΠΈΡ†Ρƒ Π΄Π»ΠΈΠ½Ρ‹. Он влияСт Π½Π° Π·Π½Π°Ρ‡Π΅Π½ΠΈΠ΅ индуктивности Π² случаС постоянного обСспСчСния ΠΎΠ±ΠΎΡ€ΠΎΡ‚ΠΎΠ² ΠΈ Π΄ΠΈΠ°ΠΌΠ΅Ρ‚Ρ€Π° ΠΊΠ°Ρ‚ΡƒΡˆΠΊΠΈ, Π° Ρ‚Π°ΠΊΠΆΠ΅ Π΄Π»ΠΈΠ½Ρ‹ ΠΊΠ°Ρ‚ΡƒΡˆΠΊΠΈ. Если Π²Ρ‹Ρ‚Π°Ρ‰ΠΈΡ‚ΡŒ ΠΊΠ°Ρ‚ΡƒΡˆΠΊΡƒ с постоянным числом ΠΎΠ±ΠΎΡ€ΠΎΡ‚ΠΎΠ² ΠΈ Π΄ΠΈΠ°ΠΌΠ΅Ρ‚Ρ€ΠΎΠΌ Π·Π° счСт увСличСния Π΄Π»ΠΈΠ½Ρ‹, Π·Π½Π°Ρ‡Π΅Π½ΠΈΠ΅ индуктивности ΠΊΠ°Ρ‚ΡƒΡˆΠΊΠΈ ΡƒΠΌΠ΅Π½ΡŒΡˆΠ°Π΅Ρ‚ΡΡ. Напротив, Ссли ΠΊΠ°Ρ‚ΡƒΡˆΠΊΠ° Π±ΡƒΠ΄Π΅Ρ‚ сТата, Ρ‡Ρ‚ΠΎΠ±Ρ‹ ΡΠ΄Π΅Π»Π°Ρ‚ΡŒ Π΅Π΅ плоской, Π·Π½Π°Ρ‡Π΅Π½ΠΈΠ΅ индуктивности ΠΊΠ°Ρ‚ΡƒΡˆΠΊΠΈ Π±ΡƒΠ΄Π΅Ρ‚ ΡƒΠ²Π΅Π»ΠΈΡ‡Π΅Π½ΠΎ.Π’ случаС Π³ΠΈΠ±ΠΊΠΎΠ³ΠΎ ΠΏΡ€ΠΎΠ²ΠΎΠ΄Π°, Ссли частота увСличиваСтся, Π·Π½Π°Ρ‡Π΅Π½ΠΈΠ΅ индуктивности увСличиваСтся.

Для ΠΏΠΎΠ²Ρ‹ΡˆΠ΅Π½ΠΈΡ добротности ΠΈ индуктивности Π² случаС Π³ΠΈΠ±ΠΊΠΎΠ³ΠΎ ΠΏΡ€ΠΎΠ²ΠΎΠ΄Π° ΠΊΠ°ΠΆΠ΄Ρ‹ΠΉ ΠΏΡ€ΠΎΠ²ΠΎΠ΄ ΠΌΠΎΠΆΠ΅Ρ‚ Π±Ρ‹Ρ‚ΡŒ ΠΎΡ‡Π΅Π½ΡŒ ΠΏΠ»ΠΎΡ‚Π½ΠΎ склССн ΠΏΡƒΡ‚Π΅ΠΌ самоклССния ΠΈ управлСния расчСтом шага. Π‘Π»Π΅Π΄ΠΎΠ²Π°Ρ‚Π΅Π»ΡŒΠ½ΠΎ, Π»ΠΈΡ‚Ρ†-ΠΏΡ€ΠΎΠ²ΠΎΠ΄ Π΄ΠΎΠ»ΠΆΠ΅Π½ ΠΏΡ€ΠΎΡ…ΠΎΠ΄ΠΈΡ‚ΡŒ Ρ‡Π΅Ρ€Π΅Π· ΠΏΡ€ΠΎΠ²ΠΎΠ΄Π°, Π²ΠΎΠ·ΠΌΠΎΠΆΠ½ΠΎ, ΠΌΠ½ΠΎΠ³ΠΎΠΊΡ€Π°Ρ‚Π½ΠΎ, ΠΈ ΠΏΠΎΠ²Ρ‹ΡˆΠ°Ρ‚ΡŒ Ρ‚Π΅ΠΌΠΏΠ΅Ρ€Π°Ρ‚ΡƒΡ€Ρƒ Π½Π° Π·Π°ΠΊΠ»ΡŽΡ‡ΠΈΡ‚Π΅Π»ΡŒΠ½ΠΎΠΌ этапС ΠΏΠΎΠ΄Π°Ρ‡ΠΈ, ΠΈ Ρ‚ΠΎΠ³Π΄Π° Π±ΡƒΠ΄Π΅Ρ‚ ΠΏΠΎΠ»ΡƒΡ‡Π΅Π½ΠΎ Π±ΠΎΠ»Π΅Π΅ высокоС Π·Π½Π°Ρ‡Π΅Π½ΠΈΠ΅ добротности.

3. Π’ зависимости ΠΎΡ‚ направлСния индуктивности, ΠΊΠ°ΠΊ ΠΏΠΎΠΊΠ°Π·Π°Π½ΠΎ Π½ΠΈΠΆΠ΅.
Π˜Π½Π΄ΡƒΠΊΡ‚ΠΈΠ²Π½ΠΎΡΡ‚ΡŒ увСличиваСтся ΠΏΡ€ΠΈ Π΄Π²ΠΈΠΆΠ΅Π½ΠΈΠΈ Π² Ρ‚ΠΎΠΌ ΠΆΠ΅ Π½Π°ΠΏΡ€Π°Π²Π»Π΅Π½ΠΈΠΈ, Ρ‡Ρ‚ΠΎ ΠΈ Π½Π° рисункС β„– 1. L (ΠΈΠ½Π΄ΡƒΠΊΡ‚ΠΈΠ²Π½ΠΎΡΡ‚ΡŒ) = L1 + L2
. Π˜Π½Π΄ΡƒΠΊΡ‚ΠΈΠ²Π½ΠΎΡΡ‚ΡŒ Ρ‚Π΅Ρ‡Π΅Ρ‚ Π² ΠΏΡ€ΠΎΡ‚ΠΈΠ²ΠΎΠΏΠΎΠ»ΠΎΠΆΠ½ΠΎΠΌ Π½Π°ΠΏΡ€Π°Π²Π»Π΅Π½ΠΈΠΈ, ΠΊΠ°ΠΊ ΠΏΠΎΠΊΠ°Π·Π°Π½ΠΎ Π½Π° рисункС β„– 2. L (ΠΈΠ½Π΄ΡƒΠΊΡ‚ΠΈΠ²Π½ΠΎΡΡ‚ΡŒ) = L1-L2
Π‘Π»Π΅Π΄ΠΎΠ²Π°Ρ‚Π΅Π»ΡŒΠ½ΠΎ, β„– 1 ΠΎΠ±Ρ‹Ρ‡Π½ΠΎ ΠΈΡΠΏΠΎΠ»ΡŒΠ·ΡƒΠ΅Ρ‚ΡΡ для увСличСния значСния L.

взаимная ΠΈΠ½Π΄ΡƒΠΊΡ‚ΠΈΠ²Π½ΠΎΡΡ‚ΡŒ

ΠœΡ‹ Ρ…ΠΎΡ‚Π΅Π»ΠΈ Π±Ρ‹ ΠΎΠ±ΡŠΡΡΠ½ΠΈΡ‚ΡŒ ΠΊΠ»ΠΈΠ΅Π½Ρ‚Π°ΠΌ «РасчСт Π½Π΅ΠΎΠ±Ρ…ΠΎΠ΄ΠΈΠΌΠΎΠ³ΠΎ ΡΡ‹Ρ€ΡŒΡΒ» для шСлка ΠΈΠ»ΠΈ Π½Π΅ΠΉΠ»ΠΎΠ½Π° Β»ΡΠ»Π΅Π΄ΡƒΡŽΡ‰ΠΈΠΌ ΠΎΠ±Ρ€Π°Π·ΠΎΠΌ. РасчСт ΠΊΠ²Π°Π΄Ρ€Π°Ρ‚Π½ΠΎΠ³ΠΎ ΠΌΠ΅Ρ‚Ρ€Π° являСтся большим ΠΏΠΎΠ΄ΡΠΏΠΎΡ€ΡŒΠ΅ΠΌ Π½Π° основной стадии производства, Ρ‚Π°ΠΊ ΠΊΠ°ΠΊ позволяСт ΡΠΏΡ€ΠΎΠ³Π½ΠΎΠ·ΠΈΡ€ΠΎΠ²Π°Ρ‚ΡŒ ΠΏΠΎΡ‚Ρ€Π΅Π±Π½ΠΎΡΡ‚ΡŒ Π² расходах Π½Π° ΡΡ‹Ρ€ΡŒΠ΅.

1) НапримСр, 0,05 ΠΌΠΌ x 1000 ΠΆΠΈΠ» с Π΄Π²ΠΎΠΉΠ½ΠΎΠΉ ΠΏΠΎΠ΄Π°Ρ‡Π΅ΠΉ,
— OD = √1,000 x 1,154 x 0,062 ΠΌΠΌ (Ρ‚ΠΎΠ»Ρ‰ΠΈΠ½Π°, Π²ΠΊΠ»ΡŽΡ‡Π°Ρ эмалСвоС ΠΏΠΎΠΊΡ€Ρ‹Ρ‚ΠΈΠ΅) = ΠΏΡ€ΠΈΠ±Π». Ξ¦2,5 ΠΌΠΌ (Π²ΠΊΠ»ΡŽΡ‡Π°Ρ Ρ‚ΠΎΠ»Ρ‰ΠΈΠ½Ρƒ шСлка ΠΈΠ»ΠΈ Π½Π΅ΠΉΠ»ΠΎΠ½Π°)
— S (ΠΊΠ²Π°Π΄Ρ€Π°Ρ‚Π½Ρ‹ΠΉ ΠΌΠ΅Ρ‚Ρ€) = Ξ¦2,5 ΠΌΠΌ x Ο€ = ΠΏΡ€ΠΈΠ±Π». 7,6㎟

расчСт ΠΊΠ²Π°Π΄Ρ€Π°Ρ‚Π½ΠΎΠ³ΠΎ ΠΌΠ΅Ρ‚Ρ€Π° для шСлка ΠΈ Π½Π΅ΠΉΠ»ΠΎΠ½Π° Π½Π° Π»ΠΈΡ‚Ρ†ΠΎΠ²ΠΎΠΉ ΠΏΡ€ΠΎΠ²ΠΎΠ»ΠΎΠΊΠ΅

2) Π’ случаС ΠΎΠ΄ΠΈΠ½Π°Ρ€Π½Ρ‹Ρ… Π½ΠΈΡ‚Π΅ΠΉ 0,05 ΠΌΠΌ x 1,680, вСс Π½Π΅ΠΉΠ»ΠΎΠ½Π° составляСт ΠΏΡ€ΠΈΠ±Π». 32 Π³ / ΠΌ (фактичСскоС ΠΈΠ·ΠΌΠ΅Ρ€Π΅Π½ΠΈΠ΅).
— Π’Ρ€Π΅Π±ΡƒΠ΅ΠΌΡ‹ΠΉ объСм Β«0,05 ΠΌΠΌ x 1000 прядСй» рассчитан Π½Π° ΠΏΡ€ΠΈΠ±Π».19,2 Π³ / ΠΌ ΠΏΠΎ ΡƒΡ€Π°Π²Π½Π΅Π½ΠΈΡŽ.
— 19,2 Π³ / ΠΌ x 2 (Π² случаС Π΄Π²ΠΎΠΉΠ½ΠΎΠΉ ΠΏΠΎΠ΄Π°Ρ‡ΠΈ) = 38,4 Π³ / ΠΌ
— Если Π²Ρ‹ Ρ…ΠΎΡ‚ΠΈΡ‚Π΅ Π·Π½Π°Ρ‚ΡŒ () ΠΌ Π½Π° ΠΊΠ³, X = 1000 strnads x 1 ΠΌ / 19,2 = 52,08 ΠΌ / ΠΊΠ³ -> 100 ΠΊΠ³ = 5 208 ΠΌ

3) Π’ случаС ΠΆΠΈΠ»Ρ‹ 0,05 ΠΌΠΌ x 1000 с Π΄Π²ΠΎΠΉΠ½ΠΎΠΉ ΠΏΠΎΠ΄Π°Ρ‡Π΅ΠΉ Π½Π΅ΠΎΠ±Ρ…ΠΎΠ΄ΠΈΠΌΠΎ Ρ€Π°ΡΡΡ‡ΠΈΡ‚Π°Ρ‚ΡŒ ΠΊΠ²Π°Π΄Ρ€Π°Ρ‚Π½Ρ‹ΠΉ ΠΌΠ΅Ρ‚Ρ€ ΡΠ»Π΅Π΄ΡƒΡŽΡ‰ΠΈΠΌ ΠΎΠ±Ρ€Π°Π·ΠΎΠΌ.
— 0,0076 ΠΌ (1 ΠΌ ΠΊΠ²Π°Π΄Ρ€Π°Ρ‚Π½Ρ‹ΠΉ ΠΌΠ΅Ρ‚Ρ€ 0,05 ΠΌΠΌ x 1000 Π½ΠΈΡ‚Π΅ΠΉ) x 5,208 ΠΌ (количСство шСлка ΠΈΠ»ΠΈ Π½Π΅ΠΉΠ»ΠΎΠ½Π°, Π½Π°ΠΌΠΎΡ‚Π°Π½Π½ΠΎΠ³ΠΎ Π½Π° 100 ΠΊΠ³ Π»ΠΈΡ‚Ρ†ΠΎΠ²ΠΎΠΉ ΠΏΡ€ΠΎΠ²ΠΎΠ»ΠΎΠΊΠΈ) = S = 39,58㎑ (ΠΏΠ»ΠΎΡ‰Π°Π΄ΡŒ повСрхности 100 ΠΊΠ³ Π»ΠΈΡ‚Ρ†-ΠΏΡ€ΠΎΠ²ΠΎΠ»ΠΎΠΊΠΈ)

РассчитайтС ΠΏΠ»ΠΎΡ‰Π°Π΄ΡŒ ΠΏΠΎΠΏΠ΅Ρ€Π΅Ρ‡Π½ΠΎΠ³ΠΎ сСчСния ΠΏΡ€ΠΎΠ²ΠΎΠ΄Π° класса 10 ΠΏΠΎ Ρ„ΠΈΠ·ΠΈΠΊΠ΅ CBSE

Подсказка: это Π·Π°Π΄Π°Ρ‡ΠΈ Π±Π°Π·ΠΎΠ²ΠΎΠ³ΠΎ уровня, ΠΊΠΎΡ‚ΠΎΡ€Ρ‹Π΅ ΠΌΠΎΠΆΠ½ΠΎ Π»Π΅Π³ΠΊΠΎ Ρ€Π΅ΡˆΠΈΡ‚ΡŒ, ΠΎΠΏΡ€Π΅Π΄Π΅Π»ΠΈΠ² значСния, Ρ‡Ρ‚ΠΎΠ±Ρ‹ ΠΈΡΠΏΠΎΠ»ΡŒΠ·ΠΎΠ²Π°Ρ‚ΡŒ Ρ„ΠΎΡ€ΠΌΡƒΠ»Ρƒ ΠΏΠ»ΠΎΡ‰Π°Π΄ΠΈ ΠΏΡ€ΠΎΠ²ΠΎΠ΄Π°, ΠΈΠΌΠ΅ΡŽΡ‰Π΅Π³ΠΎ сопротивлСниС ΠΈ ΡƒΠ΄Π΅Π»ΡŒΠ½ΠΎΠ΅ сопротивлСниС i.Π΅. $ area = \ dfrac {{\ left ({resistivity \ times length} \ right)}} {{resis \ tan ce}} $, Ρ‡Ρ‚ΠΎΠ±Ρ‹ Π½Π°ΠΉΡ‚ΠΈ ΠΏΠ»ΠΎΡ‰Π°Π΄ΡŒ ΠΏΠΎΠΏΠ΅Ρ€Π΅Ρ‡Π½ΠΎΠ³ΠΎ сСчСния Π΄Π°Π½Π½ΠΎΠ³ΠΎ ΠΏΡ€ΠΎΠ²ΠΎΠ΄Π°.

ПолноС пошаговоС Ρ€Π΅ΡˆΠ΅Π½ΠΈΠ΅:
Π’ этом вопросС Π΅ΡΡ‚ΡŒ 2 Ρ‚Π΅Ρ€ΠΌΠΈΠ½Π° «сопротивлСниС» ΠΈ Β«ΡƒΠ΄Π΅Π»ΡŒΠ½ΠΎΠ΅ сопротивлСниС», ΠΊΠΎΡ‚ΠΎΡ€Ρ‹Π΅ ΠΌΠΎΠΆΠ½ΠΎ ΠΎΠ±ΡŠΡΡΠ½ΠΈΡ‚ΡŒ ΡΠ»Π΅Π΄ΡƒΡŽΡ‰ΠΈΠΌ ΠΎΠ±Ρ€Π°Π·ΠΎΠΌ: Π‘ΠΎΠΏΡ€ΠΎΡ‚ΠΈΠ²Π»Π΅Π½ΠΈΠ΅ — это Π²Π΅Π»ΠΈΡ‡ΠΈΠ½Π° сопротивлСния Ρ‚ΠΎΠΊΡƒ, ΠΏΡ€ΠΎΡ‚Π΅ΠΊΠ°ΡŽΡ‰Π΅ΠΌΡƒ Π² ΠΏΡ€ΠΎΠ²ΠΎΠ΄Π΅, ΠΈ измСряСтся Π² ΠΎΠΌΠ°Ρ…. А ΡƒΠ΄Π΅Π»ΡŒΠ½ΠΎΠ΅ сопротивлСниС — это Ρ…Π°Ρ€Π°ΠΊΡ‚Π΅Ρ€Π½ΠΎΠ΅ свойство ΠΌΠ°Ρ‚Π΅Ρ€ΠΈΠ°Π»Π° ΠΏΡ€ΠΎΠ²ΠΎΠ΄Π°: ΡƒΠ΄Π΅Π»ΡŒΠ½ΠΎΠ΅ сопротивлСниС ΠΏΠΎΠΌΠΎΠ³Π°Π΅Ρ‚ ΡΡ€Π°Π²Π½ΠΈΠ²Π°Ρ‚ΡŒ ΠΌΠ°Ρ‚Π΅Ρ€ΠΈΠ°Π»Ρ‹ Π½Π° основС ΠΈΡ… способности ΠΏΡ€ΠΎΠ²ΠΎΠ΄ΠΈΡ‚ΡŒ элСктричСскиС Ρ‚ΠΎΠΊΠΈ.2} $.

ΠŸΡ€ΠΈΠΌΠ΅Ρ‡Π°Π½ΠΈΠ΅. Π—Π½Π°Π΅Ρ‚Π΅ Π»ΠΈ Π²Ρ‹, ΠΊΠ°ΠΊ сопротивлСниС ΠΏΡ€ΠΎΠ²ΠΎΠ΄Π° зависит ΠΎΡ‚ ΠΏΠ»ΠΎΡ‰Π°Π΄ΠΈ ΠΏΠΎΠΏΠ΅Ρ€Π΅Ρ‡Π½ΠΎΠ³ΠΎ сСчСния этого ΠΏΡ€ΠΎΠ²ΠΎΠ΄Π°, Π΄Π°Π²Π°ΠΉΡ‚Π΅ выясним, ΠΊΠ°ΠΊ это ΡΠ΄Π΅Π»Π°Ρ‚ΡŒ; Π’ΠΎΠ»Ρ‰ΠΈΠ½Π° ΠΏΡ€ΠΎΠ²ΠΎΠ΄ΠΎΠ² влияСт Π½Π° сумму сопротивлСний. ΠŸΡ€ΠΎΠ²ΠΎΠ΄Π° большСго Ρ€Π°Π·ΠΌΠ΅Ρ€Π° ΠΈΠΌΠ΅ΡŽΡ‚ большСС ΠΏΠΎΠΏΠ΅Ρ€Π΅Ρ‡Π½ΠΎΠ΅ сСчСниС. Π–ΠΈΠ΄ΠΊΠΎΡΡ‚ΡŒ ΠΏΡ€ΠΎΡ…ΠΎΠ΄ΠΈΡ‚ Π² Π±ΠΎΠ»ΡŒΡˆΡƒΡŽ Ρ‚Ρ€ΡƒΠ±Ρƒ с большСй ΡΠΊΠΎΡ€ΠΎΡΡ‚ΡŒΡŽ, Ρ‡Π΅ΠΌ Ρ‡Π΅Ρ€Π΅Π· ΠΌΠ°Π»Π΅Π½ΡŒΠΊΡƒΡŽ Π²ΠΎΡ€ΠΎΠ½ΠΊΡƒ. Π­Ρ‚ΠΎ ΠΌΠΎΠΆΠ΅Ρ‚ Π±Ρ‹Ρ‚ΡŒ связано с ΠΏΠΎΠ½ΠΈΠΆΠ΅Π½Π½Ρ‹ΠΌ сопротивлСниСм Π² Ρ‚Ρ€ΡƒΠ±Π΅ большСго Ρ€Π°Π·ΠΌΠ΅Ρ€Π°. Π’ΠΎΡ‡Π½ΠΎ Ρ‚Π°ΠΊ ΠΆΠ΅, Ρ‡Π΅ΠΌ Π΄Π»ΠΈΠ½Π½Π΅Π΅ ΠΏΡ€ΠΎΠ²ΠΎΠ΄, Ρ‚Π΅ΠΌ мСньшСС сопротивлСниС ΠΎΠ½ ΠΎΠΊΠ°Π·Ρ‹Π²Π°Π΅Ρ‚ ΠΏΠΎΡ‚ΠΎΠΊΡƒ элСктричСского заряда.Если всС ΠΎΡΡ‚Π°Π»ΡŒΠ½Ρ‹Π΅ Ρ„Π°ΠΊΡ‚ΠΎΡ€Ρ‹ ΠΎΠ΄ΠΈΠ½Π°ΠΊΠΎΠ²Ρ‹, заряд ΠΌΠΎΠΆΠ΅Ρ‚ ΠΏΡ€ΠΎΡ…ΠΎΠ΄ΠΈΡ‚ΡŒ Ρ‡Π΅Ρ€Π΅Π· Π±ΠΎΠ»Π΅Π΅ толстыС ΠΏΡ€ΠΎΠ²ΠΎΠ΄Π° с большСй ΠΏΠ»ΠΎΡ‰Π°Π΄ΡŒΡŽ ΠΏΠΎΠΏΠ΅Ρ€Π΅Ρ‡Π½ΠΎΠ³ΠΎ сСчСния, Ρ‡Π΅ΠΌ Ρ‡Π΅Ρ€Π΅Π· Π±ΠΎΠ»Π΅Π΅ Ρ‚ΠΎΠ½ΠΊΠΈΠ΅ ΠΏΡ€ΠΎΠ²ΠΎΠ΄Π° с большСй ΡΠΊΠΎΡ€ΠΎΡΡ‚ΡŒΡŽ.

Π Π°Π·ΠΌΠ΅Ρ€ ΠΏΡ€ΠΎΠ²ΠΎΠ΄Π½ΠΈΠΊΠ°

Π Π°Π·ΠΌΠ΅Ρ€ ΠΏΡ€ΠΎΠ²ΠΎΠ΄Π½ΠΈΠΊΠ°

Π‘Π»Π΅Π΄ΡƒΡŽΡ‰ΠΈΠ΅ Π½ΠΈΠΆΠ΅ тСксты ΡΠ²Π»ΡΡŽΡ‚ΡΡ ΡΠΎΠ±ΡΡ‚Π²Π΅Π½Π½ΠΎΡΡ‚ΡŒΡŽ ΠΈΡ… Π°Π²Ρ‚ΠΎΡ€ΠΎΠ², ΠΈ ΠΌΡ‹ Π±Π»Π°Π³ΠΎΠ΄Π°Ρ€ΠΈΠΌ ΠΈΡ… Π·Π° ΠΏΡ€Π΅Π΄ΠΎΡΡ‚Π°Π²Π»Π΅Π½Π½ΡƒΡŽ Π½Π°ΠΌ Π²ΠΎΠ·ΠΌΠΎΠΆΠ½ΠΎΡΡ‚ΡŒ бСсплатно Π΄Π΅Π»ΠΈΡ‚ΡŒΡΡ срСди студСнтов, ΠΏΡ€Π΅ΠΏΠΎΠ΄Π°Π²Π°Ρ‚Π΅Π»Π΅ΠΉ ΠΈ ΠΏΠΎΠ»ΡŒΠ·ΠΎΠ²Π°Ρ‚Π΅Π»Π΅ΠΉ Π˜Π½Ρ‚Π΅Ρ€Π½Π΅Ρ‚Π° ΠΈΡ… тСксты, ΠΊΠΎΡ‚ΠΎΡ€Ρ‹Π΅ Π±ΡƒΠ΄ΡƒΡ‚ ΠΈΡΠΏΠΎΠ»ΡŒΠ·ΠΎΠ²Π°Ρ‚ΡŒΡΡ Ρ‚ΠΎΠ»ΡŒΠΊΠΎ Π² ΠΈΠ»Π»ΡŽΡΡ‚Ρ€Π°Ρ‚ΠΈΠ²Π½Ρ‹Ρ… ΠΎΠ±Ρ€Π°Π·ΠΎΠ²Π°Ρ‚Π΅Π»ΡŒΠ½Ρ‹Ρ… ΠΈ Π½Π°ΡƒΡ‡Π½Ρ‹Ρ… цСлях.

Вся информация Π½Π° нашСм сайтС ΠΏΡ€Π΅Π΄Π½Π°Π·Π½Π°Ρ‡Π΅Π½Π° для нСкоммСрчСских ΠΎΠ±Ρ€Π°Π·ΠΎΠ²Π°Ρ‚Π΅Π»ΡŒΠ½Ρ‹Ρ… Ρ†Π΅Π»Π΅ΠΉ

Π˜Π½Ρ„ΠΎΡ€ΠΌΠ°Ρ†ΠΈΡ ΠΎ ΠΌΠ΅Π΄ΠΈΡ†ΠΈΠ½Π΅ ΠΈ Π·Π΄ΠΎΡ€ΠΎΠ²ΡŒΠ΅, содСрТащаяся Π½Π° сайтС, носит ΠΎΠ±Ρ‰ΠΈΠΉ Ρ…Π°Ρ€Π°ΠΊΡ‚Π΅Ρ€ ΠΈ носит ΠΈΡΠΊΠ»ΡŽΡ‡ΠΈΡ‚Π΅Π»ΡŒΠ½ΠΎ ΠΈΠ½Ρ„ΠΎΡ€ΠΌΠ°Ρ‚ΠΈΠ²Π½Ρ‹ΠΉ Ρ…Π°Ρ€Π°ΠΊΡ‚Π΅Ρ€ ΠΈ ΠΏΠΎ этой ΠΏΡ€ΠΈΡ‡ΠΈΠ½Π΅ Π½ΠΈ Π² ΠΊΠΎΠ΅ΠΌ случаС Π½Π΅ ΠΌΠΎΠΆΠ΅Ρ‚ Π·Π°ΠΌΠ΅Π½ΠΈΡ‚ΡŒ совСт Π²Ρ€Π°Ρ‡Π° ΠΈΠ»ΠΈ ΠΊΠ²Π°Π»ΠΈΡ„ΠΈΡ†ΠΈΡ€ΠΎΠ²Π°Π½Π½ΠΎΠ³ΠΎ ΡŽΡ€ΠΈΠ΄ΠΈΡ‡Π΅ΡΠΊΠΎΠ³ΠΎ Π»ΠΈΡ†Π°, ΠΈΠΌΠ΅ΡŽΡ‰Π΅Π³ΠΎ Π·Π°ΠΊΠΎΠ½Π½ΡƒΡŽ ΠΏΡ€ΠΎΡ„Π΅ΡΡΠΈΡŽ.

Π­Ρ‚ΠΎ Π΄ΠΎΠ»ΠΆΠ½ΠΎ Π±Ρ‹Ρ‚ΡŒ Π·Π΄Ρ€Π°Π²Ρ‹ΠΌ смыслом, Ρ‡Ρ‚ΠΎ ΠΆΠΈΠ΄ΠΊΠΎΡΡ‚ΡŒ Ρ‚Π΅Ρ‡Π΅Ρ‚ ΠΏΠΎ Ρ‚Ρ€ΡƒΠ±Π°ΠΌ большого Π΄ΠΈΠ°ΠΌΠ΅Ρ‚Ρ€Π° Π»Π΅Π³Ρ‡Π΅, Ρ‡Π΅ΠΌ ΠΏΠΎ Ρ‚Ρ€ΡƒΠ±Π°ΠΌ ΠΌΠ°Π»ΠΎΠ³ΠΎ Π΄ΠΈΠ°ΠΌΠ΅Ρ‚Ρ€Π° (Ссли Π²Π°ΠΌ Π½ΡƒΠΆΠ½Π° практичСская ΠΈΠ»Π»ΡŽΡΡ‚Ρ€Π°Ρ†ΠΈΡ, ΠΏΠΎΠΏΡ€ΠΎΠ±ΡƒΠΉΡ‚Π΅ ΠΏΠΈΡ‚ΡŒ ΠΆΠΈΠ΄ΠΊΠΎΡΡ‚ΡŒ Ρ‡Π΅Ρ€Π΅Π· соломинку Ρ€Π°Π·Π½ΠΎΠ³ΠΎ Π΄ΠΈΠ°ΠΌΠ΅Ρ‚Ρ€Π°).Π’ΠΎΡ‚ ΠΆΠ΅ ΠΎΠ±Ρ‰ΠΈΠΉ ΠΏΡ€ΠΈΠ½Ρ†ΠΈΠΏ дСйствуСт для ΠΏΠΎΡ‚ΠΎΠΊΠ° элСктронов Ρ‡Π΅Ρ€Π΅Π· ΠΏΡ€ΠΎΠ²ΠΎΠ΄Π½ΠΈΠΊΠΈ: Ρ‡Π΅ΠΌ ΡˆΠΈΡ€Π΅ ΠΏΠ»ΠΎΡ‰Π°Π΄ΡŒ ΠΏΠΎΠΏΠ΅Ρ€Π΅Ρ‡Π½ΠΎΠ³ΠΎ сСчСния (Ρ‚ΠΎΠ»Ρ‰ΠΈΠ½Π°) ΠΏΡ€ΠΎΠ²ΠΎΠ΄Π½ΠΈΠΊΠ°, Ρ‚Π΅ΠΌ большС мСста для протСкания элСктронов ΠΈ, ΡΠ»Π΅Π΄ΠΎΠ²Π°Ρ‚Π΅Π»ΡŒΠ½ΠΎ, Ρ‚Π΅ΠΌ Π»Π΅Π³Ρ‡Π΅ Π²ΠΎΠ·Π½ΠΈΠΊΠ°Π΅Ρ‚ ΠΏΠΎΡ‚ΠΎΠΊ (мСньшСС сопротивлСниС). .
ЭлСктричСский ΠΏΡ€ΠΎΠ²ΠΎΠ΄ ΠΎΠ±Ρ‹Ρ‡Π½ΠΎ ΠΈΠΌΠ΅Π΅Ρ‚ ΠΊΡ€ΡƒΠ³Π»ΠΎΠ΅ ΠΏΠΎΠΏΠ΅Ρ€Π΅Ρ‡Π½ΠΎΠ΅ сСчСниС (хотя Π΅ΡΡ‚ΡŒ Π½Π΅ΠΊΠΎΡ‚ΠΎΡ€Ρ‹Π΅ ΡƒΠ½ΠΈΠΊΠ°Π»ΡŒΠ½Ρ‹Π΅ ΠΈΡΠΊΠ»ΡŽΡ‡Π΅Π½ΠΈΡ ΠΈΠ· этого ΠΏΡ€Π°Π²ΠΈΠ»Π°) ΠΈ Π±Ρ‹Π²Π°Π΅Ρ‚ Π΄Π²ΡƒΡ… основных разновидностСй: ΠΎΠ΄Π½ΠΎΠΆΠΈΠ»ΡŒΠ½Ρ‹ΠΉ ΠΈ ΠΌΠ½ΠΎΠ³ΠΎΠΆΠΈΠ»ΡŒΠ½Ρ‹ΠΉ. Бплошная мСдная ΠΏΡ€ΠΎΠ²ΠΎΠ»ΠΎΠΊΠ° — это Ρ‚Π°ΠΊ, ΠΊΠ°ΠΊ Π·Π²ΡƒΡ‡ΠΈΡ‚: ΠΎΠ΄Π½Π° сплошная мСдная ΠΆΠΈΠ»Π° ΠΏΠΎ всСй Π΄Π»ΠΈΠ½Π΅ ΠΏΡ€ΠΎΠ²ΠΎΠ΄Π°.ΠœΠ½ΠΎΠ³ΠΎΠΆΠΈΠ»ΡŒΠ½Ρ‹ΠΉ ΠΏΡ€ΠΎΠ²ΠΎΠ΄ состоит ΠΈΠ· Π±ΠΎΠ»Π΅Π΅ ΠΌΠ΅Π»ΠΊΠΈΡ… ΠΆΠΈΠ» сплошного ΠΌΠ΅Π΄Π½ΠΎΠ³ΠΎ ΠΏΡ€ΠΎΠ²ΠΎΠ΄Π°, скручСнных вмСстС, Ρ‡Ρ‚ΠΎΠ±Ρ‹ ΡΡ„ΠΎΡ€ΠΌΠΈΡ€ΠΎΠ²Π°Ρ‚ΡŒ Π΅Π΄ΠΈΠ½Ρ‹ΠΉ ΠΏΡ€ΠΎΠ²ΠΎΠ΄ большСго Ρ€Π°Π·ΠΌΠ΅Ρ€Π°. Π‘Π°ΠΌΡ‹ΠΌ большим прСимущСством многоТильного ΠΏΡ€ΠΎΠ²ΠΎΠ΄Π° являСтся Π΅Π³ΠΎ мСханичСская Π³ΠΈΠ±ΠΊΠΎΡΡ‚ΡŒ, ΡΠΏΠΎΡΠΎΠ±Π½ΠΎΡΡ‚ΡŒ Π²Ρ‹Π΄Π΅Ρ€ΠΆΠΈΠ²Π°Ρ‚ΡŒ ΠΌΠ½ΠΎΠ³ΠΎΠΊΡ€Π°Ρ‚Π½Ρ‹Π΅ ΠΈΠ·Π³ΠΈΠ±Ρ‹ ΠΈ скручивания Π½Π°ΠΌΠ½ΠΎΠ³ΠΎ Π»ΡƒΡ‡ΡˆΠ΅, Ρ‡Π΅ΠΌ сплошная мСдь (которая со Π²Ρ€Π΅ΠΌΠ΅Π½Π΅ΠΌ склонна ΠΊ усталости ΠΈ ломаСтся).
Π Π°Π·ΠΌΠ΅Ρ€ ΠΏΡ€ΠΎΠ²ΠΎΠ΄Π° ΠΌΠΎΠΆΠ½ΠΎ ΠΈΠ·ΠΌΠ΅Ρ€ΠΈΡ‚ΡŒ нСсколькими способами. ΠœΡ‹ ΠΌΠΎΠ³Π»ΠΈ Π±Ρ‹ Π³ΠΎΠ²ΠΎΡ€ΠΈΡ‚ΡŒ ΠΎ Π΄ΠΈΠ°ΠΌΠ΅Ρ‚Ρ€Π΅ ΠΏΡ€ΠΎΠ²ΠΎΠ΄Π°, Π½ΠΎ ΠΏΠΎΡΠΊΠΎΠ»ΡŒΠΊΡƒ Π½Π° самом Π΄Π΅Π»Π΅ наибольшСС Π·Π½Π°Ρ‡Π΅Π½ΠΈΠ΅ для ΠΏΠΎΡ‚ΠΎΠΊΠ° элСктронов ΠΈΠΌΠ΅Π΅Ρ‚ ΠΏΠ»ΠΎΡ‰Π°Π΄ΡŒ ΠΏΠΎΠΏΠ΅Ρ€Π΅Ρ‡Π½ΠΎΠ³ΠΎ сСчСния , Π½Π°ΠΌ Π»ΡƒΡ‡ΡˆΠ΅ ΠΎΠΏΡ€Π΅Π΄Π΅Π»ΡΡ‚ΡŒ Ρ€Π°Π·ΠΌΠ΅Ρ€ ΠΏΡ€ΠΎΠ²ΠΎΠ΄Π° Π² Ρ‚Π΅Ρ€ΠΌΠΈΠ½Π°Ρ… ΠΏΠ»ΠΎΡ‰Π°Π΄ΠΈ.

Π˜Π·ΠΎΠ±Ρ€Π°ΠΆΠ΅Π½ΠΈΠ΅ сСчСния ΠΏΡ€ΠΎΠ²ΠΎΠ΄Π°, ΠΏΠΎΠΊΠ°Π·Π°Π½Π½ΠΎΠ΅ Π²Ρ‹ΡˆΠ΅, ΠΊΠΎΠ½Π΅Ρ‡Π½ΠΎ, Π½Π΅ Π² ΠΌΠ°ΡΡˆΡ‚Π°Π±Π΅. Π”ΠΈΠ°ΠΌΠ΅Ρ‚Ρ€ ΠΏΠΎΠΊΠ°Π·Π°Π½ ΠΊΠ°ΠΊ 0,1019 дюйма. Вычисляя ΠΏΠ»ΠΎΡ‰Π°Π΄ΡŒ ΠΏΠΎΠΏΠ΅Ρ€Π΅Ρ‡Π½ΠΎΠ³ΠΎ сСчСния ΠΏΠΎ Ρ„ΠΎΡ€ΠΌΡƒΠ»Π΅ Area = Ο€r2, ΠΌΡ‹ ΠΏΠΎΠ»ΡƒΡ‡Π°Π΅ΠΌ ΠΏΠ»ΠΎΡ‰Π°Π΄ΡŒ 0,008155 ΠΊΠ²Π°Π΄Ρ€Π°Ρ‚Π½Ρ‹Ρ… дюймов:

Π­Ρ‚ΠΎ довольно нСбольшиС числа, с ΠΊΠΎΡ‚ΠΎΡ€Ρ‹ΠΌΠΈ ΠΌΠΎΠΆΠ½ΠΎ Ρ€Π°Π±ΠΎΡ‚Π°Ρ‚ΡŒ, поэтому Ρ€Π°Π·ΠΌΠ΅Ρ€Ρ‹ ΠΏΡ€ΠΎΠ²ΠΎΠ΄ΠΎΠ² часто Π²Ρ‹Ρ€Π°ΠΆΠ°ΡŽΡ‚ΡΡ Π² Π΅Π΄ΠΈΠ½ΠΈΡ†Π°Ρ… тысячных Π΄ΠΎΠ»Π΅ΠΉ Π΄ΠΈΠ°ΠΌΠ΅Ρ‚Ρ€Π°. дюйма ΠΈΠ»ΠΈ ΠΌΠΈΠ» . Для ΠΏΡ€ΠΎΠΈΠ»Π»ΡŽΡΡ‚Ρ€ΠΈΡ€ΠΎΠ²Π°Π½Π½ΠΎΠ³ΠΎ ΠΏΡ€ΠΈΠΌΠ΅Ρ€Π° ΠΌΡ‹ Π±Ρ‹ сказали, Ρ‡Ρ‚ΠΎ Π΄ΠΈΠ°ΠΌΠ΅Ρ‚Ρ€ ΠΏΡ€ΠΎΠ²ΠΎΠ»ΠΎΠΊΠΈ Π±Ρ‹Π» 101.9 ΠΌΠΈΠ» (0,1019 дюйма, ΡƒΠΌΠ½ΠΎΠΆΠ΅Π½Π½ΠΎΠ΅ Π½Π° 1000). ΠœΡ‹ Ρ‚Π°ΠΊΠΆΠ΅ ΠΌΠΎΠ³Π»ΠΈ Π±Ρ‹, Ссли Π±Ρ‹ Π·Π°Ρ…ΠΎΡ‚Π΅Π»ΠΈ, Π²Ρ‹Ρ€Π°Π·ΠΈΡ‚ΡŒ ΠΏΠ»ΠΎΡ‰Π°Π΄ΡŒ ΠΏΡ€ΠΎΠ²ΠΎΠ΄Π° Π² ΠΊΠ²Π°Π΄Ρ€Π°Ρ‚Π½Ρ‹Ρ… милях, вычислив это Π·Π½Π°Ρ‡Π΅Π½ΠΈΠ΅ ΠΏΠΎ Ρ‚ΠΎΠΉ ΠΆΠ΅ Ρ„ΠΎΡ€ΠΌΡƒΠ»Π΅ ΠΊΡ€ΡƒΠ³Π° ΠΈ ΠΏΠ»ΠΎΡ‰Π°Π΄ΠΈ, ΠŸΠ»ΠΎΡ‰Π°Π΄ΡŒ = Ο€r2:


Однако элСктрики ΠΈ Π΄Ρ€ΡƒΠ³ΠΈΠ΅ Π»ΠΈΡ†Π°, часто ΠΎΠ·Π°Π±ΠΎΡ‡Π΅Π½Π½Ρ‹Π΅ Ρ€Π°Π·ΠΌΠ΅Ρ€ΠΎΠΌ ΠΏΡ€ΠΎΠ²ΠΎΠ΄Π°, ΠΈΡΠΏΠΎΠ»ΡŒΠ·ΡƒΡŽΡ‚ Π΄Ρ€ΡƒΠ³ΡƒΡŽ Π΅Π΄ΠΈΠ½ΠΈΡ†Ρƒ измСрСния ΠΏΠ»ΠΎΡ‰Π°Π΄ΠΈ, ΡΠΏΠ΅Ρ†ΠΈΠ°Π»ΡŒΠ½ΠΎ Ρ€Π°Π·Ρ€Π°Π±ΠΎΡ‚Π°Π½Π½ΡƒΡŽ для ΠΊΡ€ΡƒΠ³Π»ΠΎΠ³ΠΎ сСчСния ΠΏΡ€ΠΎΠ²ΠΎΠ΄Π°. Π­Ρ‚ΠΎ ΡΠΏΠ΅Ρ†ΠΈΠ°Π»ΡŒΠ½ΠΎΠ΅ устройство называСтся ΠΊΡ€ΡƒΠ³Π»Ρ‹ΠΌ ΠΌΠΈΠ» (ΠΈΠ½ΠΎΠ³Π΄Π° сокращСнно см ). ЕдинствСнная Ρ†Π΅Π»ΡŒ наличия этой ΡΠΏΠ΅Ρ†ΠΈΠ°Π»ΡŒΠ½ΠΎΠΉ Π΅Π΄ΠΈΠ½ΠΈΡ†Ρ‹ измСрСния состоит Π² Ρ‚ΠΎΠΌ, Ρ‡Ρ‚ΠΎΠ±Ρ‹ ΠΈΡΠΊΠ»ΡŽΡ‡ΠΈΡ‚ΡŒ Π½Π΅ΠΎΠ±Ρ…ΠΎΠ΄ΠΈΠΌΠΎΡΡ‚ΡŒ использования мноТитСля Ο€ (3.1415927. . .) Π² Ρ„ΠΎΡ€ΠΌΡƒΠ»Π΅ для расчСта ΠΏΠ»ΠΎΡ‰Π°Π΄ΠΈ плюс Π½Π΅ΠΎΠ±Ρ…ΠΎΠ΄ΠΈΠΌΠΎΡΡ‚ΡŒ ΠΎΠ±ΠΎΠ·Π½Π°Ρ‡ΠΈΡ‚ΡŒ радиус ΠΏΡ€ΠΎΠ²ΠΎΠ΄Π° радиус , ΠΊΠΎΠ³Π΄Π° Π²Π°ΠΌ Π΄Π°Π»ΠΈ Π΄ΠΈΠ°ΠΌΠ΅Ρ‚Ρ€ . Π€ΠΎΡ€ΠΌΡƒΠ»Π° для расчСта ΠΏΠ»ΠΎΡ‰Π°Π΄ΠΈ Π² ΠΌΠΈΠ»Π°Ρ… ΠΊΡ€ΡƒΠ³Π»ΠΎΠΉ ΠΏΡ€ΠΎΠ²ΠΎΠ»ΠΎΠΊΠΈ ΠΎΡ‡Π΅Π½ΡŒ проста:

ΠŸΠΎΡΠΊΠΎΠ»ΡŒΠΊΡƒ это Π΅Π΄ΠΈΠ½ΠΈΡ†Π° измСрСния ΠΏΠ»ΠΎΡ‰Π°Π΄ΠΈ , матСматичСская ΡΡ‚Π΅ΠΏΠ΅Π½ΡŒ 2 всС Π΅Ρ‰Π΅ дСйствуСт (ΡƒΠ΄Π²ΠΎΠ΅Π½ΠΈΠ΅ ΡˆΠΈΡ€ΠΈΠ½Ρ‹ ΠΊΡ€ΡƒΠ³Π° всСгда ΡƒΠ²Π΅Π»ΠΈΡ‡ΠΈΠ²Π°Π΅Ρ‚ Π΅Π³ΠΎ ΠΏΠ»ΠΎΡ‰Π°Π΄ΡŒ Π² Ρ‡Π΅Ρ‚Ρ‹Ρ€Π΅ Ρ€Π°Π·Π°, нСзависимо ΠΎΡ‚ Ρ‚ΠΎΠ³ΠΎ, ΠΊΠ°ΠΊΠΈΠ΅ Π΅Π΄ΠΈΠ½ΠΈΡ†Ρ‹ измСрСния ΠΈΡΠΏΠΎΠ»ΡŒΠ·ΡƒΡŽΡ‚ΡΡ, ΠΈΠ»ΠΈ Ссли ΡˆΠΈΡ€ΠΈΠ½Π° этого ΠΊΡ€ΡƒΠ³Π° Ρ€Π°Π²Π½Π° выраТаСтся Π² Π΅Π΄ΠΈΠ½ΠΈΡ†Π°Ρ… радиуса ΠΈΠ»ΠΈ Π΄ΠΈΠ°ΠΌΠ΅Ρ‚Ρ€Π°).Π§Ρ‚ΠΎΠ±Ρ‹ ΠΏΡ€ΠΎΠΈΠ»Π»ΡŽΡΡ‚Ρ€ΠΈΡ€ΠΎΠ²Π°Ρ‚ΡŒ Ρ€Π°Π·Π½ΠΈΡ†Ρƒ ΠΌΠ΅ΠΆΠ΄Ρƒ измСрСниями Π² ΠΊΠ²Π°Π΄Ρ€Π°Ρ‚Π½Ρ‹Ρ… милях ΠΈ измСрСниями Π² ΠΊΡ€ΡƒΠ³Π»Ρ‹Ρ… ΠΌΠΈΠ»Π°Ρ…, я ΡΡ€Π°Π²Π½ΡŽ ΠΊΡ€ΡƒΠ³ с ΠΊΠ²Π°Π΄Ρ€Π°Ρ‚ΠΎΠΌ, показывая ΠΏΠ»ΠΎΡ‰Π°Π΄ΡŒ ΠΊΠ°ΠΆΠ΄ΠΎΠΉ Ρ„ΠΎΡ€ΠΌΡ‹ Π² ΠΎΠ±Π΅ΠΈΡ… Π΅Π΄ΠΈΠ½ΠΈΡ†Π°Ρ… измСрСния:

А для Π΄Ρ€ΡƒΠ³ΠΎΠ³ΠΎ Ρ€Π°Π·ΠΌΠ΅Ρ€Π° ΠΏΡ€ΠΎΠ²ΠΎΠ»ΠΎΠΊΠΈ:

ΠžΡ‡Π΅Π²ΠΈΠ΄Π½ΠΎ, ΠΊΡ€ΡƒΠ³ Π·Π°Π΄Π°Π½Π½ΠΎΠ³ΠΎ Π΄ΠΈΠ°ΠΌΠ΅Ρ‚Ρ€Π° ΠΈΠΌΠ΅Π΅Ρ‚ ΠΌΠ΅Π½ΡŒΡˆΡƒΡŽ ΠΏΠ»ΠΎΡ‰Π°Π΄ΡŒ ΠΏΠΎΠΏΠ΅Ρ€Π΅Ρ‡Π½ΠΎΠ³ΠΎ сСчСния, Ρ‡Π΅ΠΌ ΠΊΠ²Π°Π΄Ρ€Π°Ρ‚ ΡˆΠΈΡ€ΠΈΠ½Ρ‹ ΠΈ высоты, Ρ€Π°Π²Π½Ρ‹ΠΉ Π΄ΠΈΠ°ΠΌΠ΅Ρ‚Ρ€Ρƒ ΠΊΡ€ΡƒΠ³Π°: это ΠΎΡ‚Ρ€Π°ΠΆΠ°ΡŽΡ‚ ΠΎΠ±Π΅ Π΅Π΄ΠΈΠ½ΠΈΡ†Ρ‹ измСрСния ΠΏΠ»ΠΎΡ‰Π°Π΄ΠΈ. Однако Π΄ΠΎΠ»ΠΆΠ½ΠΎ Π±Ρ‹Ρ‚ΡŒ ясно, Ρ‡Ρ‚ΠΎ Π΅Π΄ΠΈΠ½ΠΈΡ†Π° измСрСния Β«ΠΊΠ²Π°Π΄Ρ€Π°Ρ‚Π½Ρ‹ΠΉ ΠΌΠΈΠ»Β» Π΄Π΅ΠΉΡΡ‚Π²ΠΈΡ‚Π΅Π»ΡŒΠ½ΠΎ ΠΏΡ€Π΅Π΄Π½Π°Π·Π½Π°Ρ‡Π΅Π½Π° для ΡƒΠ΄ΠΎΠ±Π½ΠΎΠ³ΠΎ опрСдСлСния ΠΏΠ»ΠΎΡ‰Π°Π΄ΠΈ ΠΊΠ²Π°Π΄Ρ€Π°Ρ‚Π°, Π² Ρ‚ΠΎ врСмя ΠΊΠ°ΠΊ Β«ΠΊΡ€ΡƒΠ³ΠΎΠ²ΠΎΠΉ ΠΌΠΈΠ»Β» Π°Π΄Π°ΠΏΡ‚ΠΈΡ€ΠΎΠ²Π°Π½ для ΡƒΠ΄ΠΎΠ±Π½ΠΎΠ³ΠΎ опрСдСлСния ΠΏΠ»ΠΎΡ‰Π°Π΄ΠΈ ΠΊΡ€ΡƒΠ³Π°: ΡΠΎΠΎΡ‚Π²Π΅Ρ‚ΡΡ‚Π²ΡƒΡŽΡ‰ΡƒΡŽ Ρ„ΠΎΡ€ΠΌΡƒΠ»Ρƒ для ΠΊΠ°ΠΆΠ΄ΠΎΠ³ΠΎ ΠΏΡ€ΠΎΡ‰Π΅ Ρ€Π°Π±ΠΎΡ‚Π°Ρ‚ΡŒ с.Π‘Π»Π΅Π΄ΡƒΠ΅Ρ‚ ΠΏΠΎΠ½ΠΈΠΌΠ°Ρ‚ΡŒ, Ρ‡Ρ‚ΠΎ ΠΎΠ±Π΅ Π΅Π΄ΠΈΠ½ΠΈΡ†Ρ‹ Π΄Π΅ΠΉΡΡ‚Π²ΠΈΡ‚Π΅Π»ΡŒΠ½Ρ‹ для измСрСния ΠΏΠ»ΠΎΡ‰Π°Π΄ΠΈ Ρ„ΠΎΡ€ΠΌΡ‹, нСзависимо ΠΎΡ‚ Ρ‚ΠΎΠ³ΠΎ, ΠΊΠ°ΠΊΠΎΠΉ Ρ„ΠΎΡ€ΠΌΡ‹ ΠΎΠ½Π° ΠΌΠΎΠΆΠ΅Ρ‚ Π±Ρ‹Ρ‚ΡŒ. ΠŸΡ€Π΅ΠΎΠ±Ρ€Π°Π·ΠΎΠ²Π°Π½ΠΈΠ΅ ΠΌΠ΅ΠΆΠ΄Ρƒ ΠΊΡ€ΡƒΠ³ΠΎΠ²Ρ‹ΠΌΠΈ ΠΌΠΈΠ»Π°ΠΌΠΈ ΠΈ ΠΊΠ²Π°Π΄Ρ€Π°Ρ‚Π½Ρ‹ΠΌΠΈ ΠΌΠΈΠ»Π°ΠΌΠΈ прСдставляСт собой простоС ΡΠΎΠΎΡ‚Π½ΠΎΡˆΠ΅Π½ΠΈΠ΅: Π½Π° ΠΊΠ°ΠΆΠ΄Ρ‹Π΅ 4 ΠΊΡ€ΡƒΠ³ΠΎΠ²Ρ‹Ρ… ΠΌΠΈΠ»Π° приходится Ο€ (3,1415927 …) ΠΊΠ²Π°Π΄Ρ€Π°Ρ‚Π½Ρ‹Ρ… ΠΌΠΈΠ»ΠΎΠ².
Π•Ρ‰Π΅ ΠΎΠ΄Π½ΠΈΠΌ срСдством измСрСния ΠΏΠ»ΠΎΡ‰Π°Π΄ΠΈ ΠΏΠΎΠΏΠ΅Ρ€Π΅Ρ‡Π½ΠΎΠ³ΠΎ сСчСния ΠΏΡ€ΠΎΠ²ΠΎΠ΄Π° являСтся ΠΊΠ°Π»ΠΈΠ±Ρ€ ΠΊΠ°Π»ΠΈΠ±Ρ€Π° . Π¨ΠΊΠ°Π»Π° Π΄Π°Ρ‚Ρ‡ΠΈΠΊΠ° основана Π½Π° Ρ†Π΅Π»Ρ‹Ρ… числах, Π° Π½Π΅ Π½Π° Π΄Ρ€ΠΎΠ±Π½Ρ‹Ρ… ΠΈΠ»ΠΈ дСсятичных Π΄ΡŽΠΉΠΌΠ°Ρ…. Π§Π΅ΠΌ большС Π½ΠΎΠΌΠ΅Ρ€ ΠΊΠ°Π»ΠΈΠ±Ρ€Π°, Ρ‚Π΅ΠΌ Ρ‚ΠΎΠ½ΡŒΡˆΠ΅ ΠΏΡ€ΠΎΠ²ΠΎΠ΄; Ρ‡Π΅ΠΌ мСньшС Π½ΠΎΠΌΠ΅Ρ€ ΠΊΠ°Π»ΠΈΠ±Ρ€Π°, Ρ‚Π΅ΠΌ Ρ‚ΠΎΠ»Ρ‰Π΅ ΠΏΡ€ΠΎΠ²ΠΎΠ»ΠΎΠΊΠ°.Для Ρ‚Π΅Ρ…, ΠΊΡ‚ΠΎ Π·Π½Π°ΠΊΠΎΠΌ с Ρ€ΡƒΠΆΡŒΡΠΌΠΈ, эта ΠΎΠ±Ρ€Π°Ρ‚Π½ΠΎ ΠΏΡ€ΠΎΠΏΠΎΡ€Ρ†ΠΈΠΎΠ½Π°Π»ΡŒΠ½Π°Ρ шкала измСрСния Π΄ΠΎΠ»ΠΆΠ½Π° ΠΏΠΎΠΊΠ°Π·Π°Ρ‚ΡŒΡΡ Π·Π½Π°ΠΊΠΎΠΌΠΎΠΉ.
Π’Π°Π±Π»ΠΈΡ†Π° Π² ΠΊΠΎΠ½Ρ†Π΅ этого Ρ€Π°Π·Π΄Π΅Π»Π° ΠΏΡ€ΠΈΡ€Π°Π²Π½ΠΈΠ²Π°Π΅Ρ‚ ΠΊΠ°Π»ΠΈΠ±Ρ€ ΠΊ Π΄ΠΈΠ°ΠΌΠ΅Ρ‚Ρ€Ρƒ Π² Π΄ΡŽΠΉΠΌΠ°Ρ…, ΠΊΡ€ΡƒΠ³Π»Ρ‹Π΅ ΠΌΠΈΠ»Ρ‹ ΠΈ ΠΊΠ²Π°Π΄Ρ€Π°Ρ‚Π½Ρ‹Π΅ Π΄ΡŽΠΉΠΌΡ‹ для сплошной ΠΏΡ€ΠΎΠ²ΠΎΠ»ΠΎΠΊΠΈ. ΠŸΡ€ΠΎΠ²ΠΎΠ΄Π° большСго Π΄ΠΈΠ°ΠΌΠ΅Ρ‚Ρ€Π° Π΄ΠΎΡΡ‚ΠΈΠ³Π°ΡŽΡ‚ ΠΊΠΎΠ½Ρ†Π° ΠΎΠ±Ρ‰Π΅ΠΉ ΡˆΠΊΠ°Π»Ρ‹ ΠΊΠ°Π»ΠΈΠ±Ρ€Π° (которая, СстСствСнно, достигаСт максимума, Ρ€Π°Π²Π½ΠΎΠ³ΠΎ 1), ΠΈ прСдставлСны сСриСй Π½ΡƒΠ»Π΅ΠΉ. Β«3/0Β» — это Π΅Ρ‰Π΅ ΠΎΠ΄ΠΈΠ½ способ прСдставлСния Β«000Β», ΠΊΠΎΡ‚ΠΎΡ€Ρ‹ΠΉ произносится ΠΊΠ°ΠΊ Β«Ρ‚Ρ€ΠΎΠΉΠ½ΠΎΠΉ Π΄ΠΎΠ»Β». ΠžΠΏΡΡ‚ΡŒ ΠΆΠ΅, Ρ‚Π΅ΠΌ, ΠΊΡ‚ΠΎ Π·Π½Π°ΠΊΠΎΠΌ с Ρ€ΡƒΠΆΡŒΡΠΌΠΈ, слСдуСт ΠΏΡ€ΠΈΠ·Π½Π°Ρ‚ΡŒ Ρ‚Π΅Ρ€ΠΌΠΈΠ½ΠΎΠ»ΠΎΠ³ΠΈΡŽ, ΠΊΠ°ΠΊ Π±Ρ‹ странно это Π½ΠΈ Π·Π²ΡƒΡ‡Π°Π»ΠΎ.Π§Ρ‚ΠΎ Π΅Ρ‰Π΅ большС сбиваСт с Ρ‚ΠΎΠ»ΠΊΡƒ, Π² ΠΌΠΈΡ€Π΅ сущСствуСт Π±ΠΎΠ»Π΅Π΅ ΠΎΠ΄Π½ΠΎΠ³ΠΎ «стандарта» ΠΊΠ°Π»ΠΈΠ±Ρ€Π°. Для опрСдСлСния Ρ€Π°Π·ΠΌΠ΅Ρ€ΠΎΠ² элСктричСских ΠΏΡ€ΠΎΠ²ΠΎΠ΄ΠΎΠ² ΠΏΡ€Π΅Π΄ΠΏΠΎΡ‡Ρ‚ΠΈΡ‚Π΅Π»ΡŒΠ½ΠΎΠΉ систСмой измСрСния являСтся ΠΊΠ°Π»ΠΈΠ±Ρ€ American Wire Gauge (AWG), Ρ‚Π°ΠΊΠΆΠ΅ извСстный ΠΊΠ°ΠΊ ΠΊΠ°Π»ΠΈΠ±Ρ€ Brown ΠΈ Sharpe (B&S). Π’ КанадС ΠΈ Π’Π΅Π»ΠΈΠΊΠΎΠ±Ρ€ΠΈΡ‚Π°Π½ΠΈΠΈ британский стандартный ΠΊΠ°Π»ΠΈΠ±Ρ€ для ΠΏΡ€ΠΎΠ²ΠΎΠ΄ΠΎΠ² (SWG) являСтся ΠΎΡ„ΠΈΡ†ΠΈΠ°Π»ΡŒΠ½ΠΎΠΉ систСмой измСрСния элСктричСских ΠΏΡ€ΠΎΠ²ΠΎΠ΄ΠΎΠ². Π’ ΠΌΠΈΡ€Π΅ ΡΡƒΡ‰Π΅ΡΡ‚Π²ΡƒΡŽΡ‚ Π΄Ρ€ΡƒΠ³ΠΈΠ΅ систСмы ΠΊΠ°Π»ΠΈΠ±Ρ€ΠΎΠ²ΠΊΠΈ ΠΏΡ€ΠΎΠ²ΠΎΠ»ΠΎΠΊΠΈ для классификации Π΄ΠΈΠ°ΠΌΠ΅Ρ‚Ρ€Π° ΠΏΡ€ΠΎΠ²ΠΎΠ»ΠΎΠΊΠΈ, Ρ‚Π°ΠΊΠΈΠ΅ ΠΊΠ°ΠΊ ΠΊΠ°Π»ΠΈΠ±Ρ€ для ΡΡ‚Π°Π»ΡŒΠ½ΠΎΠΉ ΠΏΡ€ΠΎΠ²ΠΎΠ»ΠΎΠΊΠΈ Stubs ΠΈ ΠΊΠ°Π»ΠΈΠ±Ρ€ для ΡΡ‚Π°Π»ΡŒΠ½ΠΎΠΉ ΠΌΡƒΠ·Ρ‹ΠΊΠ°Π»ΡŒΠ½ΠΎΠΉ ΠΏΡ€ΠΎΠ²ΠΎΠ»ΠΎΠΊΠΈ (MWG), Π½ΠΎ эти систСмы измСрСния ΠΏΡ€ΠΈΠΌΠ΅Π½ΠΈΠΌΡ‹ ΠΊ нСэлСктричСским ΠΏΡ€ΠΎΠ²ΠΎΠ΄Π°ΠΌ.
БистСма измСрСния American Wire Gauge (AWG), нСсмотря Π½Π° Π΅Π΅ странности, Π±Ρ‹Π»Π° Ρ€Π°Π·Ρ€Π°Π±ΠΎΡ‚Π°Π½Π° с ΠΎΠ΄Π½ΠΎΠΉ Ρ†Π΅Π»ΡŒΡŽ: Π½Π° ΠΊΠ°ΠΆΠ΄Ρ‹Π΅ Ρ‚Ρ€ΠΈ шага Π½Π° шкалС ΠΊΠ°Π»ΠΈΠ±Ρ€Π° ΠΏΠ»ΠΎΡ‰Π°Π΄ΡŒ ΠΏΡ€ΠΎΠ²ΠΎΠ΄Π° (ΠΈ вСс Π½Π° Π΅Π΄ΠΈΠ½ΠΈΡ†Ρƒ Π΄Π»ΠΈΠ½Ρ‹) ΠΏΡ€ΠΈΠΌΠ΅Ρ€Π½ΠΎ удваиваСтся. Π­Ρ‚ΠΎ ΡƒΠ΄ΠΎΠ±Π½ΠΎΠ΅ ΠΏΡ€Π°Π²ΠΈΠ»ΠΎ, ΠΊΠΎΡ‚ΠΎΡ€ΠΎΠ΅ слСдуСт ΠΏΠΎΠΌΠ½ΠΈΡ‚ΡŒ ΠΏΡ€ΠΈ ΠΏΡ€ΠΈΠ±Π»ΠΈΠ·ΠΈΡ‚Π΅Π»ΡŒΠ½ΠΎΠΉ ΠΎΡ†Π΅Π½ΠΊΠ΅ Π΄ΠΈΠ°ΠΌΠ΅Ρ‚Ρ€Π° ΠΏΡ€ΠΎΠ²ΠΎΠ»ΠΎΠΊΠΈ!
Для ΠΏΡ€ΠΎΠ²ΠΎΠ΄ΠΎΠ² ΠΎΡ‡Π΅Π½ΡŒ Π±ΠΎΠ»ΡŒΡˆΠΈΡ… Ρ€Π°Π·ΠΌΠ΅Ρ€ΠΎΠ² (Ρ‚ΠΎΠ»Ρ‰Π΅ 4/0) ΠΎΡ‚ систСмы ΠΊΠ°Π»ΠΈΠ±Ρ€Π° ΠΏΡ€ΠΎΠ²ΠΎΠ»ΠΎΠΊΠΈ ΠΎΠ±Ρ‹Ρ‡Π½ΠΎ ΠΎΡ‚ΠΊΠ°Π·Ρ‹Π²Π°ΡŽΡ‚ΡΡ для измСрСния ΠΏΠ»ΠΎΡ‰Π°Π΄ΠΈ ΠΏΠΎΠΏΠ΅Ρ€Π΅Ρ‡Π½ΠΎΠ³ΠΎ сСчСния Π² тысячах ΠΊΡ€ΡƒΠ³Π»Ρ‹Ρ… ΠΌΠΈΠ» (MCM), заимствуя ΡΡ‚Π°Ρ€ΡƒΡŽ Ρ€ΠΈΠΌΡΠΊΡƒΡŽ Ρ†ΠΈΡ„Ρ€Ρƒ Β«MΒ» для обозначСния ΠΊΡ€Π°Ρ‚Π½ΠΎΠ³ΠΎ «тысяча» ΠΏΠ΅Ρ€Π΅Π΄ Β«CMΒ» для Β«ΠΊΡ€ΡƒΠ³ΠΎΠ²Ρ‹Ρ… ΠΌΠΈΠ».Β«Π’ ΡΠ»Π΅Π΄ΡƒΡŽΡ‰Π΅ΠΉ Ρ‚Π°Π±Π»ΠΈΡ†Π΅ сСчСния ΠΏΡ€ΠΎΠ²ΠΎΠ΄ΠΎΠ² Π½Π΅ ΡƒΠΊΠ°Π·Π°Π½Ρ‹ Ρ€Π°Π·ΠΌΠ΅Ρ€Ρ‹, ΠΏΡ€Π΅Π²Ρ‹ΡˆΠ°ΡŽΡ‰ΠΈΠ΅ ΠΊΠ°Π»ΠΈΠ±Ρ€ 4/0, ΠΏΠΎΡ‚ΠΎΠΌΡƒ Ρ‡Ρ‚ΠΎ сплошной ΠΌΠ΅Π΄Π½Ρ‹ΠΉ ΠΏΡ€ΠΎΠ²ΠΎΠ΄ становится Π½Π΅ΠΏΡ€Π°ΠΊΡ‚ΠΈΡ‡Π½Ρ‹ΠΌ Π² ΠΎΠ±Ρ€Π°Ρ‰Π΅Π½ΠΈΠΈ с Ρ‚Π°ΠΊΠΈΠΌΠΈ Ρ€Π°Π·ΠΌΠ΅Ρ€Π°ΠΌΠΈ. ВмСсто этого ΠΏΡ€Π΅Π΄ΠΏΠΎΡ‡Ρ‚Π΅Π½ΠΈΠ΅ отдаСтся ΠΌΠ½ΠΎΠ³ΠΎΠΏΡ€ΠΎΠ²ΠΎΠ»ΠΎΡ‡Π½ΠΎΠΉ конструкции.

ΠŸΠ ΠžΠ’ΠžΠ›ΠžΠ§ΠΠ«Π™ Π‘Π’ΠžΠ› Π”Π›Π― Π’Π’Π•Π Π”Π«Π₯, ΠšΠ Π£Π“Π›Π«Π₯ ΠœΠ•Π”ΠΠ«Π₯ ΠŸΠ ΠžΠ’ΠžΠ”ΠΠ˜ΠšΠžΠ’

Π Π°Π·ΠΌΠ΅Ρ€ Π”ΠΈΠ°ΠΌΠ΅Ρ‚Ρ€ ΠŸΠ»ΠΎΡ‰Π°Π΄ΡŒ ΠΏΠΎΠΏΠ΅Ρ€Π΅Ρ‡Π½ΠΎΠ³ΠΎ сСчСния ВСс
AWG Π΄ΡŽΠΉΠΌΡ‹ Cir. ΠΌΠΈΠ»Ρ‹ ΠΊΠ². дюймов Ρ„ΡƒΠ½Ρ‚ / 1000 Ρ„ΡƒΡ‚ΠΎΠ²
================================================== =============
4/0 ——— 0.4600 ——- 211 600 —— 0,1662 —— 640,5
3/0 ——— 0,4096 ——- 167,800 —— 0,1318 —— 507,9
2/0 ——— 0,3648 ——- 133,100 —— 0,1045 —— 402,8
1/0 ——— 0,3249 ——- 105,500 —— 0,08289 —— 319,5
1 ——— 0,2893 ——- 83,690 —— 0,06573 —— 253,5
2 ——— 0,2576 ——- 66,370 —— 0,05213 —— 200,9
3 ——— 0,2294 ——- 52,630 —— 0.04134 —— 159,3
4 ——— 0,2043 ——- 41,740 —— 0,03278 —— 126,4
5 ——— 0,1819 ——- 33,100 —— 0,02600 —— 100,2
6 ——— 0,1620 ——- 26,250 —— 0,02062 —— 79,46
7 ——— 0,1443 ——- 20,820 —— 0,01635 —— 63,02
8 ——— 0,1285 ——- 16,510 —— 0,01297 —— 49,97
9 ——— 0,1144 ——- 13 090 —— 0,01028 —— 39.63
10 ——— 0,1019 ——- 10,380 —— 0,008155 —— 31,43
11 ——— 0,09074 ——- 8,234 —— 0,006467 —— 24,92
12 ——— 0,08081 ——- 6,530 —— 0,005129 —— 19,77
13 ——— 0,07196 ——- 5,178 —— 0,004067 —— 15,68
14 ——— 0,06408 ——- 4,107 —— 0,003225 —— 12,43
15 ——— 0,05707 ——- 3,257 —— 0,002558 —— 9,858
16 ——— 0.05082 ——- 2,583 —— 0,002028 —— 7,818
17 ——— 0,04526 ——- 2,048 —— 0,001609 —— 6,200
18 ——— 0,04030 ——- 1,624 —— 0,001276 —— 4,917
19 ——— 0,03589 ——- 1,288 —— 0,001012 —— 3,899
20 ——— 0,03196 ——- 1,022 —— 0,0008023 —— 3,092
21 ——— 0,02846 ——- 810,1 —— 0,0006363 —— 2,452
22 ——— 0,02535 ——- 642.5 —— 0,0005046 —— 1,945
23 ——— 0,02257 ——- 509,5 —— 0,0004001 —— 1,542
24 ——— 0,02010 ——- 404,0 —— 0,0003173 —— 1,233
25 ——— 0,01790 ——- 320,4 —— 0,0002517 —— 0,9699
26 ——— 0,01594 ——- 254,1 —— 0,0001996 —— 0,7692
27 ——— 0,01420 ——- 201,5 —— 0,0001583 —— 0,6100
28 ——— 0,01264 ——- 159,8 —— 0,0001255 —— 0.4837
29 ——— 0,01126 ——- 126,7 —— 0,00009954 —- 0,3836
30 ——— 0,01003 ——- 100,5 —— 0,00007894 —- 0,3042
31 ——- 0,008928 ——- 79,70 —— 0,00006260 —- 0,2413
32 ——- 0,007950 ——- 63,21 —— 0,00004964 —- 0,1913
33 ——- 0,007080 ——- 50,13 —— 0,00003937 —- 0,1517
34 ——- 0,006305 ——- 39,75 —— 0,00003122 —- 0,1203
35 ——- 0.005615 ——- 31,52 —— 0,00002476 — 0,09542
36 ——- 0,005000 ——- 25,00 —— 0,00001963 — 0,07567
37 ——- 0,004453 ——- 19,83 —— 0,00001557 — 0,06001
38 ——- 0,003965 ——- 15,72 —— 0,00001235 — 0,04759
39 ——- 0,003531 ——- 12,47 —- 0,000009793 — 0,03774
40 ——- 0,003145 ——- 9,888 —- 0,000007766 — 0,02993
41 ——- 0,002800 ——- 7.842 —- 0,000006159 — 0,02374
42 ——- 0,002494 ——- 6,219 —- 0,000004884 — 0,01882
43 ——- 0,002221 ——- 4,932 —- 0,000003873 — 0,01493
44 ——- 0,001978 ——- 3,911 —- 0,000003072 — 0,01184

Для Π½Π΅ΠΊΠΎΡ‚ΠΎΡ€Ρ‹Ρ… ΡΠΈΠ»ΡŒΠ½ΠΎΡ‚ΠΎΡ‡Π½Ρ‹Ρ… ΠΏΡ€ΠΈΠ»ΠΎΠΆΠ΅Π½ΠΈΠΉ Ρ‚Ρ€Π΅Π±ΡƒΡŽΡ‚ΡΡ ΠΏΡ€ΠΎΠ²ΠΎΠ΄Π° сСчСниСм, ΠΏΡ€Π΅Π²Ρ‹ΡˆΠ°ΡŽΡ‰ΠΈΠΌ практичСский ΠΏΡ€Π΅Π΄Π΅Π» Ρ€Π°Π·ΠΌΠ΅Ρ€Π° ΠΊΡ€ΡƒΠ³Π»ΠΎΠ³ΠΎ ΠΏΡ€ΠΎΠ²ΠΎΠ΄Π°. Π’ этих случаях Π² качСствС ΠΏΡ€ΠΎΠ²ΠΎΠ΄Π½ΠΈΠΊΠΎΠ² ΠΈΡΠΏΠΎΠ»ΡŒΠ·ΡƒΡŽΡ‚ΡΡ толстыС ΡˆΠΈΠ½Ρ‹ ΠΈΠ· Ρ†Π΅Π»ΡŒΠ½ΠΎΠ³ΠΎ ΠΌΠ΅Ρ‚Π°Π»Π»Π°, Π½Π°Π·Ρ‹Π²Π°Π΅ΠΌΡ‹Π΅ сборными шинами .Π¨ΠΈΠ½Ρ‹ ΠΎΠ±Ρ‹Ρ‡Π½ΠΎ ΠΈΠ·Π³ΠΎΡ‚Π°Π²Π»ΠΈΠ²Π°ΡŽΡ‚ΡΡ ΠΈΠ· ΠΌΠ΅Π΄ΠΈ ΠΈΠ»ΠΈ алюминия ΠΈ Ρ‡Π°Ρ‰Π΅ всСго Π½Π΅ΠΈΠ·ΠΎΠ»ΠΈΡ€ΠΎΠ²Π°Π½Ρ‹. Они физичСски ΠΏΠΎΠ΄Π΄Π΅Ρ€ΠΆΠΈΠ²Π°ΡŽΡ‚ΡΡ Π²Π΄Π°Π»ΠΈ ΠΎΡ‚ каркаса ΠΈΠ»ΠΈ конструкции, ΡƒΠ΄Π΅Ρ€ΠΆΠΈΠ²Π°ΡŽΡ‰Π΅ΠΉ ΠΈΡ…, с ΠΏΠΎΠΌΠΎΡ‰ΡŒΡŽ ΠΎΠΏΠΎΡ€ изолятора. Π₯отя ΠΊΠ²Π°Π΄Ρ€Π°Ρ‚Π½ΠΎΠ΅ ΠΈΠ»ΠΈ ΠΏΡ€ΡΠΌΠΎΡƒΠ³ΠΎΠ»ΡŒΠ½ΠΎΠ΅ ΠΏΠΎΠΏΠ΅Ρ€Π΅Ρ‡Π½ΠΎΠ΅ сСчСниС ΠΎΡ‡Π΅Π½ΡŒ распространСно для Ρ„ΠΎΡ€ΠΌΡ‹ шин, ΠΈΡΠΏΠΎΠ»ΡŒΠ·ΡƒΡŽΡ‚ΡΡ Ρ‚Π°ΠΊΠΆΠ΅ ΠΈ Π΄Ρ€ΡƒΠ³ΠΈΠ΅ Ρ„ΠΎΡ€ΠΌΡ‹. ΠŸΠ»ΠΎΡ‰Π°Π΄ΡŒ ΠΏΠΎΠΏΠ΅Ρ€Π΅Ρ‡Π½ΠΎΠ³ΠΎ сСчСния сборных шин ΠΎΠ±Ρ‹Ρ‡Π½ΠΎ измСряСтся Π² ΠΊΡ€ΡƒΠ³Π»Ρ‹Ρ… ΠΌΠΈΠ»Π°Ρ… (Π΄Π°ΠΆΠ΅ для ΠΊΠ²Π°Π΄Ρ€Π°Ρ‚Π½Ρ‹Ρ… ΠΈ ΠΏΡ€ΡΠΌΠΎΡƒΠ³ΠΎΠ»ΡŒΠ½Ρ‹Ρ… шин!), Π‘ΠΊΠΎΡ€Π΅Π΅ всСго, для удобства возмоТности Π½Π°ΠΏΡ€ΡΠΌΡƒΡŽ ΠΏΡ€ΠΈΡ€Π°Π²Π½ΡΡ‚ΡŒ Ρ€Π°Π·ΠΌΠ΅Ρ€ ΡˆΠΈΠ½Ρ‹ ΠΊ ΠΊΡ€ΡƒΠ³Π»ΠΎΠΌΡƒ ΠΏΡ€ΠΎΠ²ΠΎΠ΄Ρƒ.
ΠžΠ‘Π—ΠžΠ :
Π­Π»Π΅ΠΊΡ‚Ρ€ΠΎΠ½Ρ‹ проходят Ρ‡Π΅Ρ€Π΅Π· ΠΏΡ€ΠΎΠ²ΠΎΠ»ΠΎΠΊΡƒ большого Π΄ΠΈΠ°ΠΌΠ΅Ρ‚Ρ€Π° Π»Π΅Π³Ρ‡Π΅, Ρ‡Π΅ΠΌ Ρ‡Π΅Ρ€Π΅Π· ΠΏΡ€ΠΎΠ²ΠΎΠ»ΠΎΠΊΡƒ ΠΌΠ°Π»ΠΎΠ³ΠΎ Π΄ΠΈΠ°ΠΌΠ΅Ρ‚Ρ€Π°, ΠΈΠ·-Π·Π° большСй ΠΏΠ»ΠΎΡ‰Π°Π΄ΠΈ ΠΏΠΎΠΏΠ΅Ρ€Π΅Ρ‡Π½ΠΎΠ³ΠΎ сСчСния, Π² ΠΊΠΎΡ‚ΠΎΡ€ΠΎΠΉ ΠΎΠ½ΠΈ ΠΌΠΎΠ³ΡƒΡ‚ Π΄Π²ΠΈΠ³Π°Ρ‚ΡŒΡΡ.
ВмСсто Ρ‚ΠΎΠ³ΠΎ, Ρ‡Ρ‚ΠΎΠ±Ρ‹ ΠΈΠ·ΠΌΠ΅Ρ€ΡΡ‚ΡŒ нСбольшиС Ρ€Π°Π·ΠΌΠ΅Ρ€Ρ‹ ΠΏΡ€ΠΎΠ²ΠΎΠ»ΠΎΠΊΠΈ Π² Π΄ΡŽΠΉΠΌΠ°Ρ…, часто ΠΈΡΠΏΠΎΠ»ΡŒΠ·ΡƒΠ΅Ρ‚ΡΡ Π΅Π΄ΠΈΠ½ΠΈΡ†Π° измСрСния Β«ΠΌΠΈΠ»Β» (1/1000 дюйма).
ΠŸΠ»ΠΎΡ‰Π°Π΄ΡŒ ΠΏΠΎΠΏΠ΅Ρ€Π΅Ρ‡Π½ΠΎΠ³ΠΎ сСчСния ΠΏΡ€ΠΎΠ²ΠΎΠ΄Π° ΠΌΠΎΠΆΠ΅Ρ‚ Π±Ρ‹Ρ‚ΡŒ Π²Ρ‹Ρ€Π°ΠΆΠ΅Π½Π° Π² ΠΊΠ²Π°Π΄Ρ€Π°Ρ‚Π½Ρ‹Ρ… Π΅Π΄ΠΈΠ½ΠΈΡ†Π°Ρ… (ΠΊΠ²Π°Π΄Ρ€Π°Ρ‚Π½Ρ‹Π΅ Π΄ΡŽΠΉΠΌΡ‹ ΠΈΠ»ΠΈ ΠΊΠ²Π°Π΄Ρ€Π°Ρ‚Π½Ρ‹Π΅ ΠΌΠΈΠ»Ρ‹), ΠΊΡ€ΡƒΠ³ΠΎΠ²Ρ‹Π΅ ΠΌΠΈΠ»Ρ‹ ΠΈΠ»ΠΈ «калибровочная» шкала.
РасчСт ΠΏΠ»ΠΎΡ‰Π°Π΄ΠΈ ΠΊΠ²Π°Π΄Ρ€Π°Ρ‚Π½ΠΎΠΉ Π΅Π΄ΠΈΠ½ΠΈΡ†Ρ‹ для ΠΊΡ€ΡƒΠ³Π»ΠΎΠ³ΠΎ ΠΏΡ€ΠΎΠ²ΠΎΠ΄Π° выполняСтся ΠΏΠΎ Ρ„ΠΎΡ€ΠΌΡƒΠ»Π΅ ΠΏΠ»ΠΎΡ‰Π°Π΄ΠΈ ΠΊΡ€ΡƒΠ³Π°:

РасчСт сСчСния ΠΊΡ€ΡƒΠ³Π»ΠΎΠΉ ΠΏΡ€ΠΎΠ²ΠΎΠ»ΠΎΠΊΠΈ Π² ΠΌΠΈΠ»Π΅ для ΠΊΡ€ΡƒΠ³Π»ΠΎΠΉ ΠΏΡ€ΠΎΠ²ΠΎΠ»ΠΎΠΊΠΈ Π½Π°ΠΌΠ½ΠΎΠ³ΠΎ ΠΏΡ€ΠΎΡ‰Π΅ ΠΈΠ·-Π·Π° Ρ‚ΠΎΠ³ΠΎ, Ρ‡Ρ‚ΠΎ Π΅Π΄ΠΈΠ½ΠΈΡ†Π° измСрСния Β«ΠΊΡ€ΡƒΠ³ΠΎΠ²Ρ‹Ρ… ΠΌΠΈΠ»ΠΎΠ²Β» Π±Ρ‹Π»Π° Π²Ρ‹Π±Ρ€Π°Π½Π° ΠΈΠΌΠ΅Π½Π½ΠΎ для этой Ρ†Π΅Π»ΠΈ: Ρ‡Ρ‚ΠΎΠ±Ρ‹ ΠΈΡΠΊΠ»ΡŽΡ‡ΠΈΡ‚ΡŒ коэффициСнты Β«ΠΏΠΈΒ» ΠΈ d / 2 (радиус) Π² Ρ„ΠΎΡ€ΠΌΡƒΠ»Π°.

На ΠΊΠ°ΠΆΠ΄Ρ‹Π΅ 4 ΠΊΡ€ΡƒΠ³ΠΎΠ²Ρ‹Ρ… ΠΌΠΈΠ» приходится Ο€ (3,1416) ΠΊΠ²Π°Π΄Ρ€Π°Ρ‚Π½Ρ‹Ρ… ΠΌΠΈΠ»ΠΎΠ².
БистСма ΠΊΠ°Π»ΠΈΠ±Ρ€ΠΎΠ²ΠΊΠΈ ΠΏΡ€ΠΎΠ²ΠΎΠ΄ΠΎΠ² ΠΊΠ°Π»ΠΈΠ±Ρ€Π° основана Π½Π° Ρ†Π΅Π»Ρ‹Ρ… числах, большиС числа ΠΏΡ€Π΅Π΄ΡΡ‚Π°Π²Π»ΡΡŽΡ‚ ΠΏΡ€ΠΎΠ²ΠΎΠ΄Π° мСньшСй ΠΏΠ»ΠΎΡ‰Π°Π΄ΠΈ ΠΈ Π½Π°ΠΎΠ±ΠΎΡ€ΠΎΡ‚. ΠŸΡ€ΠΎΠ²ΠΎΠ΄Π° Ρ‚ΠΎΠ»Ρ‰Π΅ 1 ΠΊΠ°Π»ΠΈΠ±Ρ€Π° ΠΎΠ±ΠΎΠ·Π½Π°Ρ‡Π°ΡŽΡ‚ΡΡ нулями: 0, 00, 000 ΠΈ 0000 (произносятся «одинарная», «двойная», «тройная» ΠΈ «чСтвСрная». ΠžΡ‡Π΅Π½ΡŒ большиС сСчСния ΠΏΡ€ΠΎΠ²ΠΎΠ΄ΠΎΠ² ΠΈΠ·ΠΌΠ΅Ρ€ΡΡŽΡ‚ΡΡ Π² тысячах ΠΊΡ€ΡƒΠ³Π»Ρ‹Ρ… ΠΌΠΈΠ»ΠΎΠ² (MCM), Ρ‡Ρ‚ΠΎ Ρ‚ΠΈΠΏΠΈΡ‡Π½ΠΎ для шин ΠΈ ΠΏΡ€ΠΎΠ²ΠΎΠ΄ΠΎΠ² сСчСниСм Π²Ρ‹ΡˆΠ΅ 4/0.
Π¨ΠΈΠ½Ρ‹ — это ΡΠΏΠ»ΠΎΡˆΠ½Ρ‹Π΅ ΡˆΠΈΠ½Ρ‹ ΠΈΠ· ΠΌΠ΅Π΄ΠΈ ΠΈΠ»ΠΈ алюминия, ΠΈΡΠΏΠΎΠ»ΡŒΠ·ΡƒΠ΅ΠΌΡ‹Π΅ Π² конструкции ΡΠΈΠ»ΡŒΠ½ΠΎΡ‚ΠΎΡ‡Π½Ρ‹Ρ… Ρ†Π΅ΠΏΠ΅ΠΉ. БоСдинСния, выполняСмыС с шинами, ΠΎΠ±Ρ‹Ρ‡Π½ΠΎ ΡΠ²Π»ΡΡŽΡ‚ΡΡ сварными ΠΈΠ»ΠΈ Π±ΠΎΠ»Ρ‚ΠΎΠ²Ρ‹ΠΌΠΈ, Π° ΡˆΠΈΠ½Ρ‹ часто Π³ΠΎΠ»Ρ‹Π΅ (Π½Π΅ΠΈΠ·ΠΎΠ»ΠΈΡ€ΠΎΠ²Π°Π½Π½Ρ‹Π΅) ΠΈ ΠΏΠΎΠ΄Π΄Π΅Ρ€ΠΆΠΈΠ²Π°ΡŽΡ‚ΡΡ Π²Π΄Π°Π»ΠΈ ΠΎΡ‚ мСталличСских каркасов Π·Π° счСт использования ΠΈΠ·ΠΎΠ»ΠΈΡ€ΡƒΡŽΡ‰ΠΈΡ… стоСк.

Π˜ΡΡ‚ΠΎΡ‡Π½ΠΈΠΊ: http://www.htk.tlu.ee/ictcert/intranet/curriculum/6-developing-training-material-for-the-modules/electricity-konspekt.doc

Бсылка Π½Π° Π²Π΅Π±-сайт для посСщСния: http: // www.htk.tlu.ee/ictcert/

ΠšΠ»ΡŽΡ‡Π΅Π²ΠΎΠ΅ слово Google: Π Π°Π·ΠΌΠ΅Ρ€ ΠΏΡ€ΠΎΠ²ΠΎΠ΄Π½ΠΈΠΊΠ° Π’ΠΈΠΏ Ρ„Π°ΠΉΠ»Π°: doc

Автор: Π² исходном Π΄ΠΎΠΊΡƒΠΌΠ΅Π½Ρ‚Π΅ Π½Π΅ ΡƒΠΊΠ°Π·Π°Π½ тСкст

Если Π²Ρ‹ ΡΠ²Π»ΡΠ΅Ρ‚Π΅ΡΡŒ Π°Π²Ρ‚ΠΎΡ€ΠΎΠΌ ΠΏΡ€ΠΈΠ²Π΅Π΄Π΅Π½Π½ΠΎΠ³ΠΎ Π²Ρ‹ΡˆΠ΅ тСкста ΠΈ Π½Π΅ ΡΠΎΠ³Π»Π°ΡˆΠ°Π΅Ρ‚Π΅ΡΡŒ Π΄Π΅Π»ΠΈΡ‚ΡŒΡΡ своими знаниями для обучСния, исслСдований, стипСндий (для добросовСстного использования, ΠΊΠ°ΠΊ ΡƒΠΊΠ°Π·Π°Π½ΠΎ Π² авторских ΠΏΡ€Π°Π²Π°Ρ… БША), ΠΎΡ‚ΠΏΡ€Π°Π²ΡŒΡ‚Π΅ Π½Π°ΠΌ элСктронноС письмо, ΠΈ ΠΌΡ‹ ΡƒΠ΄Π°Π»ΠΈΠΌ ваши тСкст быстро.

Π Π°Π·ΠΌΠ΅Ρ€ ΠΏΡ€ΠΎΠ²ΠΎΠ΄Π½ΠΈΠΊΠ°

Если Π²Ρ‹ Ρ…ΠΎΡ‚ΠΈΡ‚Π΅ быстро Π½Π°ΠΉΡ‚ΠΈ страницы ΠΏΠΎ ΠΎΠΏΡ€Π΅Π΄Π΅Π»Π΅Π½Π½ΠΎΠΉ Ρ‚Π΅ΠΌΠ΅ Π² качСствС Ρ€Π°Π·ΠΌΠ΅Ρ€Π° ΠΏΡ€ΠΎΠ²ΠΎΠ΄Π½ΠΈΠΊΠ°, ΠΈΡΠΏΠΎΠ»ΡŒΠ·ΡƒΠΉΡ‚Π΅ ΡΠ»Π΅Π΄ΡƒΡŽΡ‰ΡƒΡŽ ΠΏΠΎΠΈΡΠΊΠΎΠ²ΡƒΡŽ систСму:

Π Π°Π·ΠΌΠ΅Ρ€ ΠΏΡ€ΠΎΠ²ΠΎΠ΄Π½ΠΈΠΊΠ°

ΠŸΠΎΡΠ΅Ρ‚ΠΈΡ‚Π΅ Π½Π°ΡˆΡƒ домашнюю страницу

ЛарапСдия.com Условия использования ΠΈ страница ΠΊΠΎΠ½Ρ„ΠΈΠ΄Π΅Π½Ρ†ΠΈΠ°Π»ΡŒΠ½ΠΎΡΡ‚ΠΈ

9.3 УдСльноС сопротивлСниС ΠΈ сопротивлСниС — University Physics Volume 2

Π—Π°Π΄Π°Ρ‡ΠΈ обучСния

К ΠΊΠΎΠ½Ρ†Ρƒ этого Ρ€Π°Π·Π΄Π΅Π»Π° Π²Ρ‹ смоТСтС:

  • Различия ΠΌΠ΅ΠΆΠ΄Ρƒ сопротивлСниСм ΠΈ ΡƒΠ΄Π΅Π»ΡŒΠ½Ρ‹ΠΌ сопротивлСниСм
  • ΠžΠΏΡ€Π΅Π΄Π΅Π»ΠΈΡ‚Π΅ Ρ‚Π΅Ρ€ΠΌΠΈΠ½ ΠΏΡ€ΠΎΠ²ΠΎΠ΄ΠΈΠΌΠΎΡΡ‚ΡŒ
  • ΠžΠΏΠΈΡˆΠΈΡ‚Π΅ элСктричСский ΠΊΠΎΠΌΠΏΠΎΠ½Π΅Π½Ρ‚, извСстный ΠΊΠ°ΠΊ рСзистор
  • Π£ΠΊΠ°ΠΆΠΈΡ‚Π΅ взаимосвязь ΠΌΠ΅ΠΆΠ΄Ρƒ сопротивлСниСм рСзистора ΠΈ Π΅Π³ΠΎ Π΄Π»ΠΈΠ½ΠΎΠΉ, ΠΏΠ»ΠΎΡ‰Π°Π΄ΡŒΡŽ ΠΏΠΎΠΏΠ΅Ρ€Π΅Ρ‡Π½ΠΎΠ³ΠΎ сСчСния ΠΈ ΡƒΠ΄Π΅Π»ΡŒΠ½Ρ‹ΠΌ сопротивлСниСм
  • Π£ΠΊΠ°ΠΆΠΈΡ‚Π΅ взаимосвязь ΠΌΠ΅ΠΆΠ΄Ρƒ ΡƒΠ΄Π΅Π»ΡŒΠ½Ρ‹ΠΌ сопротивлСниСм ΠΈ Ρ‚Π΅ΠΌΠΏΠ΅Ρ€Π°Ρ‚ΡƒΡ€ΠΎΠΉ

Π§Ρ‚ΠΎ Π΄Π²ΠΈΠΆΠ΅Ρ‚ Ρ‚ΠΎΠΊΠΎΠΌ? ΠœΡ‹ ΠΌΠΎΠΆΠ΅ΠΌ Π΄ΡƒΠΌΠ°Ρ‚ΡŒ ΠΎ Ρ€Π°Π·Π»ΠΈΡ‡Π½Ρ‹Ρ… устройствах, Ρ‚Π°ΠΊΠΈΡ… ΠΊΠ°ΠΊ Π±Π°Ρ‚Π°Ρ€Π΅ΠΈ, Π³Π΅Π½Π΅Ρ€Π°Ρ‚ΠΎΡ€Ρ‹, Ρ€ΠΎΠ·Π΅Ρ‚ΠΊΠΈ ΠΈ Ρ‚. Π”., ΠšΠΎΡ‚ΠΎΡ€Ρ‹Π΅ Π½Π΅ΠΎΠ±Ρ…ΠΎΠ΄ΠΈΠΌΡ‹ для поддСрТания Ρ‚ΠΎΠΊΠ°.ВсС Ρ‚Π°ΠΊΠΈΠ΅ устройства ΡΠΎΠ·Π΄Π°ΡŽΡ‚ Ρ€Π°Π·Π½ΠΎΡΡ‚ΡŒ ΠΏΠΎΡ‚Π΅Π½Ρ†ΠΈΠ°Π»ΠΎΠ² ΠΈ Π½Π°Π·Ρ‹Π²Π°ΡŽΡ‚ΡΡ источниками напряТСния. Когда источник напряТСния ΠΏΠΎΠ΄ΠΊΠ»ΡŽΡ‡Π΅Π½ ΠΊ ΠΏΡ€ΠΎΠ²ΠΎΠ΄Π½ΠΈΠΊΡƒ, ΠΎΠ½ ΠΏΡ€ΠΈΠΊΠ»Π°Π΄Ρ‹Π²Π°Π΅Ρ‚ Ρ€Π°Π·Π½ΠΎΡΡ‚ΡŒ ΠΏΠΎΡ‚Π΅Π½Ρ†ΠΈΠ°Π»ΠΎΠ² Π’ , которая создаСт элСктричСскоС ΠΏΠΎΠ»Π΅. ЭлСктричСскоС ΠΏΠΎΠ»Π΅, Π² свою ΠΎΡ‡Π΅Ρ€Π΅Π΄ΡŒ, воздСйствуСт Π½Π° свободныС заряды, вызывая Ρ‚ΠΎΠΊ. Π’Π΅Π»ΠΈΡ‡ΠΈΠ½Π° Ρ‚ΠΎΠΊΠ° зависит Π½Π΅ Ρ‚ΠΎΠ»ΡŒΠΊΠΎ ΠΎΡ‚ Π²Π΅Π»ΠΈΡ‡ΠΈΠ½Ρ‹ напряТСния, Π½ΠΎ ΠΈ ΠΎΡ‚ характСристик ΠΌΠ°Ρ‚Π΅Ρ€ΠΈΠ°Π»Π°, Ρ‡Π΅Ρ€Π΅Π· ΠΊΠΎΡ‚ΠΎΡ€Ρ‹ΠΉ ΠΏΡ€ΠΎΡ‚Π΅ΠΊΠ°Π΅Ρ‚ Ρ‚ΠΎΠΊ. ΠœΠ°Ρ‚Π΅Ρ€ΠΈΠ°Π» ΠΌΠΎΠΆΠ΅Ρ‚ ΡΠΎΠΏΡ€ΠΎΡ‚ΠΈΠ²Π»ΡΡ‚ΡŒΡΡ ΠΏΠΎΡ‚ΠΎΠΊΡƒ зарядов, ΠΈ ΠΌΠ΅Ρ€Π° Ρ‚ΠΎΠ³ΠΎ, насколько ΠΌΠ°Ρ‚Π΅Ρ€ΠΈΠ°Π» сопротивляСтся ΠΏΠΎΡ‚ΠΎΠΊΡƒ зарядов, извСстна ΠΊΠ°ΠΊ ΡƒΠ΄Π΅Π»ΡŒΠ½ΠΎΠ΅ сопротивлСниС .Π­Ρ‚ΠΎ ΡƒΠ΄Π΅Π»ΡŒΠ½ΠΎΠ΅ сопротивлСниС Π³Ρ€ΡƒΠ±ΠΎ Π°Π½Π°Π»ΠΎΠ³ΠΈΡ‡Π½ΠΎ Ρ‚Ρ€Π΅Π½ΠΈΡŽ ΠΌΠ΅ΠΆΠ΄Ρƒ двумя ΠΌΠ°Ρ‚Π΅Ρ€ΠΈΠ°Π»Π°ΠΌΠΈ, ΠΊΠΎΡ‚ΠΎΡ€Ρ‹Π΅ ΡΠΎΠΏΡ€ΠΎΡ‚ΠΈΠ²Π»ΡΡŽΡ‚ΡΡ двиТСнию.

УдСльноС сопротивлСниС

Когда ΠΊ ΠΏΡ€ΠΎΠ²ΠΎΠ΄Π½ΠΈΠΊΡƒ прикладываСтся напряТСниС, создаСтся элСктричСскоС ΠΏΠΎΠ»Π΅ E β†’ E β†’, ΠΈ заряды Π² ΠΏΡ€ΠΎΠ²ΠΎΠ΄Π½ΠΈΠΊΠ΅ ΠΎΡ‰ΡƒΡ‰Π°ΡŽΡ‚ силу, ΡΠΎΠ·Π΄Π°Π²Π°Π΅ΠΌΡƒΡŽ элСктричСским ΠΏΠΎΠ»Π΅ΠΌ. ΠŸΠΎΠ»ΡƒΡ‡Π΅Π½Π½Π°Ρ ΠΏΠ»ΠΎΡ‚Π½ΠΎΡΡ‚ΡŒ Ρ‚ΠΎΠΊΠ° J β†’ J β†’ зависит ΠΎΡ‚ элСктричСского поля ΠΈ свойств ΠΌΠ°Ρ‚Π΅Ρ€ΠΈΠ°Π»Π°. Π­Ρ‚Π° Π·Π°Π²ΠΈΡΠΈΠΌΠΎΡΡ‚ΡŒ ΠΌΠΎΠΆΠ΅Ρ‚ Π±Ρ‹Ρ‚ΡŒ ΠΎΡ‡Π΅Π½ΡŒ слоТной. Π’ Π½Π΅ΠΊΠΎΡ‚ΠΎΡ€Ρ‹Ρ… ΠΌΠ°Ρ‚Π΅Ρ€ΠΈΠ°Π»Π°Ρ…, Π²ΠΊΠ»ΡŽΡ‡Π°Ρ ΠΌΠ΅Ρ‚Π°Π»Π»Ρ‹ ΠΏΡ€ΠΈ Π΄Π°Π½Π½ΠΎΠΉ Ρ‚Π΅ΠΌΠΏΠ΅Ρ€Π°Ρ‚ΡƒΡ€Π΅, ΠΏΠ»ΠΎΡ‚Π½ΠΎΡΡ‚ΡŒ Ρ‚ΠΎΠΊΠ° ΠΏΡ€ΠΈΠ±Π»ΠΈΠ·ΠΈΡ‚Π΅Π»ΡŒΠ½ΠΎ ΠΏΡ€ΠΎΠΏΠΎΡ€Ρ†ΠΈΠΎΠ½Π°Π»ΡŒΠ½Π° элСктричСскому полю.Π’ этих случаях ΠΏΠ»ΠΎΡ‚Π½ΠΎΡΡ‚ΡŒ Ρ‚ΠΎΠΊΠ° ΠΌΠΎΠΆΠ½ΠΎ ΡΠΌΠΎΠ΄Π΅Π»ΠΈΡ€ΠΎΠ²Π°Ρ‚ΡŒ ΠΊΠ°ΠΊ

Π³Π΄Π΅ σσ — ΡƒΠ΄Π΅Π»ΡŒΠ½Π°Ρ ΡΠ»Π΅ΠΊΡ‚Ρ€ΠΎΠΏΡ€ΠΎΠ²ΠΎΠ΄Π½ΠΎΡΡ‚ΡŒ. Π­Π»Π΅ΠΊΡ‚Ρ€ΠΎΠΏΡ€ΠΎΠ²ΠΎΠ΄Π½ΠΎΡΡ‚ΡŒ Π°Π½Π°Π»ΠΎΠ³ΠΈΡ‡Π½Π° тСплопроводности ΠΈ являСтся ΠΌΠ΅Ρ€ΠΎΠΉ способности ΠΌΠ°Ρ‚Π΅Ρ€ΠΈΠ°Π»Π° ΠΏΡ€ΠΎΠ²ΠΎΠ΄ΠΈΡ‚ΡŒ ΠΈΠ»ΠΈ ΠΏΠ΅Ρ€Π΅Π΄Π°Π²Π°Ρ‚ΡŒ элСктричСство. ΠŸΡ€ΠΎΠ²ΠΎΠ΄Π½ΠΈΠΊΠΈ ΠΈΠΌΠ΅ΡŽΡ‚ Π±ΠΎΠ»Π΅Π΅ Π²Ρ‹ΡΠΎΠΊΡƒΡŽ ΡΠ»Π΅ΠΊΡ‚Ρ€ΠΎΠΏΡ€ΠΎΠ²ΠΎΠ΄Π½ΠΎΡΡ‚ΡŒ, Ρ‡Π΅ΠΌ изоляторы. ΠŸΠΎΡΠΊΠΎΠ»ΡŒΠΊΡƒ ΡƒΠ΄Π΅Π»ΡŒΠ½Π°Ρ ΡΠ»Π΅ΠΊΡ‚Ρ€ΠΎΠΏΡ€ΠΎΠ²ΠΎΠ΄Π½ΠΎΡΡ‚ΡŒ Οƒ = J / EΟƒ = J / E, Π΅Π΄ΠΈΠ½ΠΈΡ†Ρ‹ Ρ€Π°Π²Π½Ρ‹

Οƒ = [Π”ΠΆ] [Π­] = А / ΠΌ2Π’ / ΠΌ = АВ Β· ΠΌ. Οƒ = [Π”ΠΆ] [Π­] = А / ΠΌ2Π’ / ΠΌ = АВ Β· ΠΌ.

Π—Π΄Π΅ΡΡŒ ΠΌΡ‹ опрСдСляСм Π΅Π΄ΠΈΠ½ΠΈΡ†Ρƒ, Π½Π°Π·Ρ‹Π²Π°Π΅ΠΌΡƒΡŽ ΠΎΠΌ с грСчСским символом ΠΎΠΌΠ΅Π³Π° Π² Π²Π΅Ρ€Ρ…Π½Π΅ΠΌ рСгистрС, ΩΩ.Устройство Π½Π°Π·Π²Π°Π½ΠΎ Π² Ρ‡Π΅ΡΡ‚ΡŒ Π“Π΅ΠΎΡ€Π³Π° Π‘ΠΈΠΌΠΎΠ½Π° Ома, ΠΎ ΠΊΠΎΡ‚ΠΎΡ€ΠΎΠΌ ΠΌΡ‹ ΠΏΠΎΠ³ΠΎΠ²ΠΎΡ€ΠΈΠΌ ΠΏΠΎΠ·ΠΆΠ΅ Π² этой Π³Π»Π°Π²Π΅. ΩΩ ΠΈΡΠΏΠΎΠ»ΡŒΠ·ΡƒΠ΅Ρ‚ΡΡ, Ρ‡Ρ‚ΠΎΠ±Ρ‹ ΠΈΠ·Π±Π΅ΠΆΠ°Ρ‚ΡŒ ΠΏΡƒΡ‚Π°Π½ΠΈΡ†Ρ‹ с числом 0. Один Ом Ρ€Π°Π²Π΅Π½ ΠΎΠ΄Π½ΠΎΠΌΡƒ Π²ΠΎΠ»ΡŒΡ‚Ρƒ Π½Π° Π°ΠΌΠΏΠ΅Ρ€: 1Ξ© = 1V / A1Ξ© = 1V / A. Π’Π°ΠΊΠΈΠΌ ΠΎΠ±Ρ€Π°Π·ΠΎΠΌ, Π΅Π΄ΠΈΠ½ΠΈΡ†Ρ‹ элСктропроводности Ρ€Π°Π²Π½Ρ‹ (Ом Β· ΠΌ) -1 (Ом Β· ΠΌ) -1.

Π­Π»Π΅ΠΊΡ‚Ρ€ΠΎΠΏΡ€ΠΎΠ²ΠΎΠ΄Π½ΠΎΡΡ‚ΡŒ — это Π²Π½ΡƒΡ‚Ρ€Π΅Π½Π½Π΅Π΅ свойство ΠΌΠ°Ρ‚Π΅Ρ€ΠΈΠ°Π»Π°. Π”Ρ€ΡƒΠ³ΠΈΠΌ Π²Π½ΡƒΡ‚Ρ€Π΅Π½Π½ΠΈΠΌ свойством ΠΌΠ°Ρ‚Π΅Ρ€ΠΈΠ°Π»Π° являСтся ΡƒΠ΄Π΅Π»ΡŒΠ½ΠΎΠ΅ сопротивлСниС ΠΈΠ»ΠΈ ΡƒΠ΄Π΅Π»ΡŒΠ½ΠΎΠ΅ элСктричСскоС сопротивлСниС. УдСльноС сопротивлСниС ΠΌΠ°Ρ‚Π΅Ρ€ΠΈΠ°Π»Π° — это ΠΌΠ΅Ρ€Π° Ρ‚ΠΎΠ³ΠΎ, насколько сильно ΠΌΠ°Ρ‚Π΅Ρ€ΠΈΠ°Π» противостоит ΠΏΡ€ΠΎΡ…ΠΎΠΆΠ΄Π΅Π½ΠΈΡŽ элСктричСского Ρ‚ΠΎΠΊΠ°.Π‘ΠΈΠΌΠ²ΠΎΠ» ΡƒΠ΄Π΅Π»ΡŒΠ½ΠΎΠ³ΠΎ сопротивлСния — строчная грСчСская Π±ΡƒΠΊΠ²Π° Ρ€ΠΎ, ρρ, Π° ΡƒΠ΄Π΅Π»ΡŒΠ½ΠΎΠ΅ сопротивлСниС — Π²Π΅Π»ΠΈΡ‡ΠΈΠ½Π°, обратная ΡƒΠ΄Π΅Π»ΡŒΠ½ΠΎΠΉ элСктропроводности:

.

Π•Π΄ΠΈΠ½ΠΈΡ†Π΅ΠΉ измСрСния ΡƒΠ΄Π΅Π»ΡŒΠ½ΠΎΠ³ΠΎ сопротивлСния Π² систСмС БИ являСтся ΠΎΠΌ-ΠΌΠ΅Ρ‚Ρ€ (Ом Β· ΠΌ) (Ом Β· ΠΌ). ΠœΡ‹ ΠΌΠΎΠΆΠ΅ΠΌ ΠΎΠΏΡ€Π΅Π΄Π΅Π»ΠΈΡ‚ΡŒ ΡƒΠ΄Π΅Π»ΡŒΠ½ΠΎΠ΅ сопротивлСниС Ρ‡Π΅Ρ€Π΅Π· элСктричСскоС ΠΏΠΎΠ»Π΅ ΠΈ ΠΏΠ»ΠΎΡ‚Π½ΠΎΡΡ‚ΡŒ Ρ‚ΠΎΠΊΠ°,

Π§Π΅ΠΌ большС ΡƒΠ΄Π΅Π»ΡŒΠ½ΠΎΠ΅ сопротивлСниС, Ρ‚Π΅ΠΌ большСС ΠΏΠΎΠ»Π΅ Π½Π΅ΠΎΠ±Ρ…ΠΎΠ΄ΠΈΠΌΠΎ для создания Π·Π°Π΄Π°Π½Π½ΠΎΠΉ плотности Ρ‚ΠΎΠΊΠ°. Π§Π΅ΠΌ Π½ΠΈΠΆΠ΅ ΡƒΠ΄Π΅Π»ΡŒΠ½ΠΎΠ΅ сопротивлСниС, Ρ‚Π΅ΠΌ большС ΠΏΠ»ΠΎΡ‚Π½ΠΎΡΡ‚ΡŒ Ρ‚ΠΎΠΊΠ°, создаваСмого Π΄Π°Π½Π½Ρ‹ΠΌ элСктричСским ΠΏΠΎΠ»Π΅ΠΌ. Π₯ΠΎΡ€ΠΎΡˆΠΈΠ΅ ΠΏΡ€ΠΎΠ²ΠΎΠ΄Π½ΠΈΠΊΠΈ ΠΎΠ±Π»Π°Π΄Π°ΡŽΡ‚ высокой ΠΏΡ€ΠΎΠ²ΠΎΠ΄ΠΈΠΌΠΎΡΡ‚ΡŒΡŽ ΠΈ Π½ΠΈΠ·ΠΊΠΈΠΌ ΡƒΠ΄Π΅Π»ΡŒΠ½Ρ‹ΠΌ сопротивлСниСм.Π₯ΠΎΡ€ΠΎΡˆΠΈΠ΅ изоляторы ΠΎΠ±Π»Π°Π΄Π°ΡŽΡ‚ Π½ΠΈΠ·ΠΊΠΎΠΉ ΠΏΡ€ΠΎΠ²ΠΎΠ΄ΠΈΠΌΠΎΡΡ‚ΡŒΡŽ ΠΈ высоким ΡƒΠ΄Π΅Π»ΡŒΠ½Ρ‹ΠΌ сопротивлСниСм. Π’ Ρ‚Π°Π±Π»ΠΈΡ†Π΅ 9.1 ΠΏΡ€ΠΈΠ²Π΅Π΄Π΅Π½Ρ‹ значСния ΡƒΠ΄Π΅Π»ΡŒΠ½ΠΎΠ³ΠΎ сопротивлСния ΠΈ проводимости для Ρ€Π°Π·Π»ΠΈΡ‡Π½Ρ‹Ρ… ΠΌΠ°Ρ‚Π΅Ρ€ΠΈΠ°Π»ΠΎΠ².

ΠœΠ°Ρ‚Π΅Ρ€ΠΈΠ°Π» Π­Π»Π΅ΠΊΡ‚Ρ€ΠΎΠΏΡ€ΠΎΠ²ΠΎΠ΄Π½ΠΎΡΡ‚ΡŒ, σσ
(Ом Β· ΠΌ) βˆ’1 (Ом Β· ΠΌ) βˆ’1
УдСльноС сопротивлСниС, ρρ
(Ом · м) (Ом · м)
Π’Π΅ΠΌΠΏΠ΅Ρ€Π°Ρ‚ΡƒΡ€Π°
ΠšΠΎΡΡ„Ρ„ΠΈΡ†ΠΈΠ΅Π½Ρ‚, Ξ±Ξ±
(Β° C) -1 (Β° C) -1
ΠŸΡ€ΠΎΠ²ΠΎΠ΄Π½ΠΈΠΊΠΈ
Π‘Π΅Ρ€Π΅Π±Ρ€ΠΎ 6.29 Γ— 1076,29 Γ— 107 1,59 Γ— 10–81,59 Γ— 10–8 0,0038
МСдь 5,95 Γ— 1075,95 Γ— 107 1,68 Γ— 10–81,68 Γ— 10–8 0,0039
Π—ΠΎΠ»ΠΎΡ‚ΠΎ 4,10 Γ— 1074,10 Γ— 107 2,44 Γ— 10–82,44 Γ— 10–8 0,0034
Алюминий 3,77 Γ— 1073,77 Γ— 107 2,65 Γ— 10–82,65 Γ— 10–8 0,0039
Π’ΠΎΠ»ΡŒΡ„Ρ€Π°ΠΌ 1,79 Γ— 1071,79 Γ— 107 5.60 Γ— 10–85,60 Γ— 10–8 0,0045
Π£Ρ‚ΡŽΠ³ 1,03 Γ— 1071,03 Γ— 107 9,71 Γ— 10–89,71 Γ— 10–8 0,0065
ΠŸΠ»Π°Ρ‚ΠΈΠ½Π° 0,94 Γ— 1070,94 Γ— 107 10,60 Γ— 10-8 10,60 Γ— 10-8 0,0039
Π‘Ρ‚Π°Π»ΡŒ 0,50 Γ— 1070,50 Γ— 107 20,00 Γ— 10-820,00 Γ— 10-8
Π‘Π²ΠΈΠ½Π΅Ρ† 0,45 Γ— 1070,45 Γ— 107 22,00 Γ— 10-822,00 Γ— 10-8
Манганин (сплав Cu, Mn, Ni) 0.21 Γ— 1070,21 Γ— 107 48,20 Γ— 10-848,20 Γ— 10-8 0,000002
ΠšΠΎΠ½ΡΡ‚Π°Π½Ρ‚Π°Π½ (сплав Cu, Ni) 0,20 Γ— 1070,20 Γ— 107 49,00 Γ— 10–849,00 Γ— 10–8 0,00003
ΠœΠ΅Ρ€ΠΊΡƒΡ€ΠΈΠΉ 0,10 Γ— 1070,10 Γ— 107 98,00 Γ— 10-898,00 Γ— 10-8 0,0009
Нихром (сплав Ni, Fe, Cr) 0,10 Γ— 1070,10 Γ— 107 100,00 Γ— 10-8 100,00 Γ— 10-8 0,0004
ΠŸΠΎΠ»ΡƒΠΏΡ€ΠΎΠ²ΠΎΠ΄Π½ΠΈΠΊΠΈ [1]
Π£Π³Π»Π΅Ρ€ΠΎΠ΄ (чистый) 2.86 Γ— 1042,86 Γ— 104 3,50 Γ— 10βˆ’53,50 Γ— 10βˆ’5 -0,0005
Π£Π³Π»Π΅Ρ€ΠΎΠ΄ (2,86–1,67) Γ— 10–6 (2,86–1,67) Γ— 10–6 (3,5-60) Γ— 10-5 (3,5-60) Γ— 10-5 -0,0005
Π“Π΅Ρ€ΠΌΠ°Π½ΠΈΠΉ (чистый) 600 Γ— 10βˆ’3600 Γ— 10βˆ’3 -0,048
Π“Π΅Ρ€ΠΌΠ°Π½ΠΈΠΉ (1-600) Γ— 10-3 (1-600) Γ— 10-3 -0,050
ΠšΡ€Π΅ΠΌΠ½ΠΈΠΉ (чистый) 2300 βˆ’0.075
ΠšΡ€Π΅ΠΌΠ½ΠΈΠΉ 0,1βˆ’23000,1βˆ’2300 -0,07
Π˜Π·ΠΎΠ»ΡΡ‚ΠΎΡ€Ρ‹
Π―Π½Ρ‚Π°Ρ€ΡŒ 2,00 Γ— 10–152,00 Γ— 10–15 5 Γ— 10145 Γ— 1014
Π‘Ρ‚Π΅ΠΊΠ»ΠΎ 10βˆ’9βˆ’10βˆ’1410βˆ’9βˆ’10βˆ’14 109βˆ’1014109βˆ’1014
Π›ΡŽΡ†ΠΈΡ‚ <10-13 <10-13> 1013> 1013
Блюда 10-11-10-1510-11-10-15 1011βˆ’10151011βˆ’1015
ΠšΠ²Π°Ρ€Ρ† (ΠΏΠ»Π°Π²Π»Π΅Π½Ρ‹ΠΉ) 1.33 Γ— 10–181,33 Γ— 10–18 75 Γ— 101675 Γ— 1016
Π Π΅Π·ΠΈΠ½Π° (твСрдая) 10-13-10-1610-13-10-16 1013βˆ’10161013βˆ’1016
сСра 10-15 10-15 10151015
Π’Π΅Ρ„Π»ΠΎΠ½ TM <10-13 <10-13> 1013> 1013
Π”Π΅Ρ€Π΅Π²ΠΎ 10-8-10-1110-8-10-11 108βˆ’1011108βˆ’1011

Π‘Ρ‚ΠΎΠ» 9.1 УдСльноС сопротивлСниС ΠΈ ΠΏΡ€ΠΎΠ²ΠΎΠ΄ΠΈΠΌΠΎΡΡ‚ΡŒ Ρ€Π°Π·Π»ΠΈΡ‡Π½Ρ‹Ρ… ΠΌΠ°Ρ‚Π΅Ρ€ΠΈΠ°Π»ΠΎΠ² ΠΏΡ€ΠΈ 20 Β° C [1] ЗначСния сильно зависят ΠΎΡ‚ количСства ΠΈ Ρ‚ΠΈΠΏΠΎΠ² примСсСй.

ΠœΠ°Ρ‚Π΅Ρ€ΠΈΠ°Π»Ρ‹, пСрСчислСнныС Π² Ρ‚Π°Π±Π»ΠΈΡ†Π΅, Ρ€Π°Π·Π΄Π΅Π»Π΅Π½Ρ‹ Π½Π° ΠΊΠ°Ρ‚Π΅Π³ΠΎΡ€ΠΈΠΈ ΠΏΡ€ΠΎΠ²ΠΎΠ΄Π½ΠΈΠΊΠΎΠ², ΠΏΠΎΠ»ΡƒΠΏΡ€ΠΎΠ²ΠΎΠ΄Π½ΠΈΠΊΠΎΠ² ΠΈ изоляторов Π½Π° основС ΡˆΠΈΡ€ΠΎΠΊΠΈΡ… Π³Ρ€ΡƒΠΏΠΏ ΡƒΠ΄Π΅Π»ΡŒΠ½ΠΎΠ³ΠΎ сопротивлСния. Π£ ΠΏΡ€ΠΎΠ²ΠΎΠ΄Π½ΠΈΠΊΠΎΠ² наимСньшСС ΡƒΠ΄Π΅Π»ΡŒΠ½ΠΎΠ΅ сопротивлСниС, Π° Ρƒ изоляторов наибольшСС; ΠΏΠΎΠ»ΡƒΠΏΡ€ΠΎΠ²ΠΎΠ΄Π½ΠΈΠΊΠΈ ΠΈΠΌΠ΅ΡŽΡ‚ ΠΏΡ€ΠΎΠΌΠ΅ΠΆΡƒΡ‚ΠΎΡ‡Π½ΠΎΠ΅ ΡƒΠ΄Π΅Π»ΡŒΠ½ΠΎΠ΅ сопротивлСниС. ΠŸΡ€ΠΎΠ²ΠΎΠ΄Π½ΠΈΠΊΠΈ ΠΈΠΌΠ΅ΡŽΡ‚ Ρ€Π°Π·Π½ΡƒΡŽ, Π½ΠΎ Π±ΠΎΠ»ΡŒΡˆΡƒΡŽ ΠΏΠ»ΠΎΡ‚Π½ΠΎΡΡ‚ΡŒ свободных зарядов, Ρ‚ΠΎΠ³Π΄Π° ΠΊΠ°ΠΊ Π±ΠΎΠ»ΡŒΡˆΠΈΠ½ΡΡ‚Π²ΠΎ зарядов Π² изоляторах связаны с Π°Ρ‚ΠΎΠΌΠ°ΠΌΠΈ ΠΈ Π½Π΅ ΠΌΠΎΠ³ΡƒΡ‚ Π΄Π²ΠΈΠ³Π°Ρ‚ΡŒΡΡ.ΠŸΠΎΠ»ΡƒΠΏΡ€ΠΎΠ²ΠΎΠ΄Π½ΠΈΠΊΠΈ ΡΠ²Π»ΡΡŽΡ‚ΡΡ ΠΏΡ€ΠΎΠΌΠ΅ΠΆΡƒΡ‚ΠΎΡ‡Π½Ρ‹ΠΌΠΈ, ΠΈΠΌΠ΅ΡŽΡ‚ Π³ΠΎΡ€Π°Π·Π΄ΠΎ мСньшС свободных зарядов, Ρ‡Π΅ΠΌ ΠΏΡ€ΠΎΠ²ΠΎΠ΄Π½ΠΈΠΊΠΈ, Π½ΠΎ ΠΎΠ±Π»Π°Π΄Π°ΡŽΡ‚ свойствами, ΠΈΠ·-Π·Π° ΠΊΠΎΡ‚ΠΎΡ€Ρ‹Ρ… количСство свободных зарядов сильно зависит ΠΎΡ‚ Ρ‚ΠΈΠΏΠ° ΠΈ количСства примСсСй Π² ΠΏΠΎΠ»ΡƒΠΏΡ€ΠΎΠ²ΠΎΠ΄Π½ΠΈΠΊΠ΅. Π­Ρ‚ΠΈ ΡƒΠ½ΠΈΠΊΠ°Π»ΡŒΠ½Ρ‹Π΅ свойства ΠΏΠΎΠ»ΡƒΠΏΡ€ΠΎΠ²ΠΎΠ΄Π½ΠΈΠΊΠΎΠ² находят ΠΏΡ€ΠΈΠΌΠ΅Π½Π΅Π½ΠΈΠ΅ Π² соврСмСнной элСктроникС, ΠΎ Ρ‡Π΅ΠΌ ΠΌΡ‹ ΠΏΠΎΠ³ΠΎΠ²ΠΎΡ€ΠΈΠΌ Π² ΡΠ»Π΅Π΄ΡƒΡŽΡ‰ΠΈΡ… Π³Π»Π°Π²Π°Ρ….

ΠŸΡ€ΠΎΠ²Π΅Ρ€ΡŒΡ‚Π΅ своС ΠΏΠΎΠ½ΠΈΠΌΠ°Π½ΠΈΠ΅ 9,5

ΠŸΡ€ΠΎΠ²Π΅Ρ€ΡŒΡ‚Π΅ своС ΠΏΠΎΠ½ΠΈΠΌΠ°Π½ΠΈΠ΅ ΠœΠ΅Π΄Π½Ρ‹Π΅ ΠΏΡ€ΠΎΠ²ΠΎΠ΄Π° ΠΎΠ±Ρ‹Ρ‡Π½ΠΎ ΠΈΡΠΏΠΎΠ»ΡŒΠ·ΡƒΡŽΡ‚ΡΡ для ΡƒΠ΄Π»ΠΈΠ½ΠΈΡ‚Π΅Π»Π΅ΠΉ ΠΈ домашнСй элСктропроводки ΠΏΠΎ нСскольким ΠΏΡ€ΠΈΡ‡ΠΈΠ½Π°ΠΌ.МСдь ΠΈΠΌΠ΅Π΅Ρ‚ самый высокий Ρ€Π΅ΠΉΡ‚ΠΈΠ½Π³ элСктропроводности ΠΈ, ΡΠ»Π΅Π΄ΠΎΠ²Π°Ρ‚Π΅Π»ΡŒΠ½ΠΎ, самый Π½ΠΈΠ·ΠΊΠΈΠΉ Ρ€Π΅ΠΉΡ‚ΠΈΠ½Π³ ΡƒΠ΄Π΅Π»ΡŒΠ½ΠΎΠ³ΠΎ сопротивлСния срСди всСх Π½Π΅Π΄Ρ€Π°Π³ΠΎΡ†Π΅Π½Π½Ρ‹Ρ… ΠΌΠ΅Ρ‚Π°Π»Π»ΠΎΠ². Π’Π°ΠΊΠΆΠ΅ Π²Π°ΠΆΠ½Π° ΠΏΡ€ΠΎΡ‡Π½ΠΎΡΡ‚ΡŒ Π½Π° Ρ€Π°Π·Ρ€Ρ‹Π², Π³Π΄Π΅ ΠΏΡ€ΠΎΡ‡Π½ΠΎΡΡ‚ΡŒ Π½Π° Ρ€Π°Π·Ρ€Ρ‹Π² являСтся ΠΌΠ΅Ρ€ΠΎΠΉ силы, Π½Π΅ΠΎΠ±Ρ…ΠΎΠ΄ΠΈΠΌΠΎΠΉ для Ρ‚ΠΎΠ³ΠΎ, Ρ‡Ρ‚ΠΎΠ±Ρ‹ ΠΏΠΎΠ΄Ρ‚ΡΠ½ΡƒΡ‚ΡŒ ΠΎΠ±ΡŠΠ΅ΠΊΡ‚ ΠΊ Ρ‚ΠΎΡ‡ΠΊΠ΅, Π³Π΄Π΅ ΠΎΠ½ сломаСтся. ΠŸΡ€ΠΎΡ‡Π½ΠΎΡΡ‚ΡŒ ΠΌΠ°Ρ‚Π΅Ρ€ΠΈΠ°Π»Π° Π½Π° Ρ€Π°Π·Ρ€Ρ‹Π² — это максимальноС Π·Π½Π°Ρ‡Π΅Π½ΠΈΠ΅ Ρ€Π°ΡΡ‚ΡΠ³ΠΈΠ²Π°ΡŽΡ‰Π΅Π³ΠΎ напряТСния, ΠΊΠΎΡ‚ΠΎΡ€ΠΎΠ΅ ΠΎΠ½ ΠΌΠΎΠΆΠ΅Ρ‚ Π²Ρ‹Π΄Π΅Ρ€ΠΆΠ°Ρ‚ΡŒ ΠΏΠ΅Ρ€Π΅Π΄ Ρ€Π°Π·Ρ€ΡƒΡˆΠ΅Π½ΠΈΠ΅ΠΌ. МСдь ΠΈΠΌΠ΅Π΅Ρ‚ высокий ΠΏΡ€Π΅Π΄Π΅Π» прочности Π½Π° Ρ€Π°Π·Ρ€Ρ‹Π², 2 Γ— 108 Нм22 Γ— 108 Нм2. Π’Ρ€Π΅Ρ‚ΡŒΡ ваТная характСристика — ΠΏΠ»Π°ΡΡ‚ΠΈΡ‡Π½ΠΎΡΡ‚ΡŒ. ΠŸΠ»Π°ΡΡ‚ΠΈΡ‡Π½ΠΎΡΡ‚ΡŒ — это ΠΌΠ΅Ρ€Π° способности ΠΌΠ°Ρ‚Π΅Ρ€ΠΈΠ°Π»Π° Π²Ρ‹Ρ‚ΡΠ³ΠΈΠ²Π°Ρ‚ΡŒΡΡ Π² ΠΏΡ€ΠΎΠ²ΠΎΠ»ΠΎΠΊΡƒ ΠΈ ΠΌΠ΅Ρ€Π° гибкости ΠΌΠ°Ρ‚Π΅Ρ€ΠΈΠ°Π»Π°, Π° мСдь ΠΎΠ±Π»Π°Π΄Π°Π΅Ρ‚ высокой ΠΏΠ»Π°ΡΡ‚ΠΈΡ‡Π½ΠΎΡΡ‚ΡŒΡŽ.Подводя ΠΈΡ‚ΠΎΠ³, ΠΌΠΎΠΆΠ½ΠΎ ΡΠΊΠ°Π·Π°Ρ‚ΡŒ, Ρ‡Ρ‚ΠΎ ΠΏΡ€ΠΎΠ²ΠΎΠ΄Π½ΠΈΠΊ являСтся подходящим ΠΊΠ°Π½Π΄ΠΈΠ΄Π°Ρ‚ΠΎΠΌ для изготовлСния ΠΏΡ€ΠΎΠ²ΠΎΠ»ΠΎΠΊΠΈ, ΠΏΠΎ ΠΊΡ€Π°ΠΉΠ½Π΅ΠΉ ΠΌΠ΅Ρ€Π΅, с трСмя Π²Π°ΠΆΠ½Ρ‹ΠΌΠΈ характСристиками: Π½ΠΈΠ·ΠΊΠΈΠΌ ΡƒΠ΄Π΅Π»ΡŒΠ½Ρ‹ΠΌ сопротивлСниСм, высокой ΠΏΡ€ΠΎΡ‡Π½ΠΎΡΡ‚ΡŒΡŽ Π½Π° Ρ€Π°Π·Ρ€Ρ‹Π² ΠΈ высокой ΠΏΠ»Π°ΡΡ‚ΠΈΡ‡Π½ΠΎΡΡ‚ΡŒΡŽ. КакиС Π΅Ρ‰Π΅ ΠΌΠ°Ρ‚Π΅Ρ€ΠΈΠ°Π»Ρ‹ ΠΈΡΠΏΠΎΠ»ΡŒΠ·ΡƒΡŽΡ‚ΡΡ для элСктромонтаТа ΠΈ Π² Ρ‡Π΅ΠΌ ΠΈΡ… достоинства ΠΈ нСдостатки?

ВСмпСратурная Π·Π°Π²ΠΈΡΠΈΠΌΠΎΡΡ‚ΡŒ ΡƒΠ΄Π΅Π»ΡŒΠ½ΠΎΠ³ΠΎ сопротивлСния

Π’Π΅Ρ€Π½ΡƒΠ²ΡˆΠΈΡΡŒ ΠΊ Ρ‚Π°Π±Π»ΠΈΡ†Π΅ 9.1, Π²Ρ‹ ΡƒΠ²ΠΈΠ΄ΠΈΡ‚Π΅ столбСц Β«Π’Π΅ΠΌΠΏΠ΅Ρ€Π°Ρ‚ΡƒΡ€Π½Ρ‹ΠΉ коэффициСнт». УдСльноС сопротивлСниС Π½Π΅ΠΊΠΎΡ‚ΠΎΡ€Ρ‹Ρ… ΠΌΠ°Ρ‚Π΅Ρ€ΠΈΠ°Π»ΠΎΠ² сильно зависит ΠΎΡ‚ Ρ‚Π΅ΠΌΠΏΠ΅Ρ€Π°Ρ‚ΡƒΡ€Ρ‹. Π’ Π½Π΅ΠΊΠΎΡ‚ΠΎΡ€Ρ‹Ρ… ΠΌΠ°Ρ‚Π΅Ρ€ΠΈΠ°Π»Π°Ρ…, Ρ‚Π°ΠΊΠΈΡ… ΠΊΠ°ΠΊ мСдь, ΡƒΠ΄Π΅Π»ΡŒΠ½ΠΎΠ΅ сопротивлСниС увСличиваСтся с ΠΏΠΎΠ²Ρ‹ΡˆΠ΅Π½ΠΈΠ΅ΠΌ Ρ‚Π΅ΠΌΠΏΠ΅Ρ€Π°Ρ‚ΡƒΡ€Ρ‹.ЀактичСски, Π² Π±ΠΎΠ»ΡŒΡˆΠΈΠ½ΡΡ‚Π²Π΅ проводящих ΠΌΠ΅Ρ‚Π°Π»Π»ΠΎΠ² ΡƒΠ΄Π΅Π»ΡŒΠ½ΠΎΠ΅ сопротивлСниС увСличиваСтся с ΠΏΠΎΠ²Ρ‹ΡˆΠ΅Π½ΠΈΠ΅ΠΌ Ρ‚Π΅ΠΌΠΏΠ΅Ρ€Π°Ρ‚ΡƒΡ€Ρ‹. ΠŸΠΎΠ²Ρ‹ΡˆΠ΅Π½ΠΈΠ΅ Ρ‚Π΅ΠΌΠΏΠ΅Ρ€Π°Ρ‚ΡƒΡ€Ρ‹ Π²Ρ‹Π·Ρ‹Π²Π°Π΅Ρ‚ ΠΏΠΎΠ²Ρ‹ΡˆΠ΅Π½Π½Ρ‹Π΅ колСбания Π°Ρ‚ΠΎΠΌΠΎΠ² Π² Ρ€Π΅ΡˆΠ΅Ρ‚Ρ‡Π°Ρ‚ΠΎΠΉ ​​структурС ΠΌΠ΅Ρ‚Π°Π»Π»ΠΎΠ², ΠΊΠΎΡ‚ΠΎΡ€Ρ‹Π΅ ΠΏΡ€Π΅ΠΏΡΡ‚ΡΡ‚Π²ΡƒΡŽΡ‚ двиТСнию элСктронов. Π’ Π΄Ρ€ΡƒΠ³ΠΈΡ… ΠΌΠ°Ρ‚Π΅Ρ€ΠΈΠ°Π»Π°Ρ…, Ρ‚Π°ΠΊΠΈΡ… ΠΊΠ°ΠΊ ΡƒΠ³Π»Π΅Ρ€ΠΎΠ΄, ΡƒΠ΄Π΅Π»ΡŒΠ½ΠΎΠ΅ сопротивлСниС ΡƒΠΌΠ΅Π½ΡŒΡˆΠ°Π΅Ρ‚ΡΡ с ΠΏΠΎΠ²Ρ‹ΡˆΠ΅Π½ΠΈΠ΅ΠΌ Ρ‚Π΅ΠΌΠΏΠ΅Ρ€Π°Ρ‚ΡƒΡ€Ρ‹. Π’ΠΎ ΠΌΠ½ΠΎΠ³ΠΈΡ… ΠΌΠ°Ρ‚Π΅Ρ€ΠΈΠ°Π»Π°Ρ… Π·Π°Π²ΠΈΡΠΈΠΌΠΎΡΡ‚ΡŒ являСтся ΠΏΡ€ΠΈΠ±Π»ΠΈΠ·ΠΈΡ‚Π΅Π»ΡŒΠ½ΠΎ Π»ΠΈΠ½Π΅ΠΉΠ½ΠΎΠΉ ΠΈ ΠΌΠΎΠΆΠ΅Ρ‚ Π±Ρ‹Ρ‚ΡŒ смодСлирована с ΠΏΠΎΠΌΠΎΡ‰ΡŒΡŽ Π»ΠΈΠ½Π΅ΠΉΠ½ΠΎΠ³ΠΎ уравнСния:

Οβ‰ˆΟ0 [1 + Ξ± (T βˆ’ T0)], Οβ‰ˆΟ0 [1 + Ξ± (T βˆ’ T0)],

9,7

, Π³Π΄Π΅ ρρ — ΡƒΠ΄Π΅Π»ΡŒΠ½ΠΎΠ΅ сопротивлСниС ΠΌΠ°Ρ‚Π΅Ρ€ΠΈΠ°Π»Π° ΠΏΡ€ΠΈ Ρ‚Π΅ΠΌΠΏΠ΅Ρ€Π°Ρ‚ΡƒΡ€Π΅ T , Ξ±Ξ± — Ρ‚Π΅ΠΌΠΏΠ΅Ρ€Π°Ρ‚ΡƒΡ€Π½Ρ‹ΠΉ коэффициСнт ΠΌΠ°Ρ‚Π΅Ρ€ΠΈΠ°Π»Π°, Π° ρ0ρ0 — ΡƒΠ΄Π΅Π»ΡŒΠ½ΠΎΠ΅ сопротивлСниС ΠΏΡ€ΠΈ T0T0, ΠΎΠ±Ρ‹Ρ‡Π½ΠΎ ΠΏΡ€ΠΈΠ½ΠΈΠΌΠ°Π΅ΠΌΠΎΠ΅ ΠΊΠ°ΠΊ T0 = 20.00 Β° CT0 = 20,00 Β° C.

ΠžΡ‚ΠΌΠ΅Ρ‚ΠΈΠΌ Ρ‚Π°ΠΊΠΆΠ΅, Ρ‡Ρ‚ΠΎ Ρ‚Π΅ΠΌΠΏΠ΅Ρ€Π°Ρ‚ΡƒΡ€Π½Ρ‹ΠΉ коэффициСнт Ξ±Ξ± ΠΎΡ‚Ρ€ΠΈΡ†Π°Ρ‚Π΅Π»Π΅Π½ для ΠΏΠΎΠ»ΡƒΠΏΡ€ΠΎΠ²ΠΎΠ΄Π½ΠΈΠΊΠΎΠ², пСрСчислСнных Π² Π’Π°Π±Π»ΠΈΡ†Π΅ 9.1, Ρ‡Ρ‚ΠΎ ΠΎΠ·Π½Π°Ρ‡Π°Π΅Ρ‚, Ρ‡Ρ‚ΠΎ ΠΈΡ… ΡƒΠ΄Π΅Π»ΡŒΠ½ΠΎΠ΅ сопротивлСниС ΡƒΠΌΠ΅Π½ΡŒΡˆΠ°Π΅Ρ‚ΡΡ с ΡƒΠ²Π΅Π»ΠΈΡ‡Π΅Π½ΠΈΠ΅ΠΌ Ρ‚Π΅ΠΌΠΏΠ΅Ρ€Π°Ρ‚ΡƒΡ€Ρ‹. Они становятся Π»ΡƒΡ‡ΡˆΠΈΠΌΠΈ ΠΏΡ€ΠΎΠ²ΠΎΠ΄Π½ΠΈΠΊΠ°ΠΌΠΈ ΠΏΡ€ΠΈ Π±ΠΎΠ»Π΅Π΅ высоких Ρ‚Π΅ΠΌΠΏΠ΅Ρ€Π°Ρ‚ΡƒΡ€Π°Ρ…, ΠΏΠΎΡ‚ΠΎΠΌΡƒ Ρ‡Ρ‚ΠΎ ΠΏΠΎΠ²Ρ‹ΡˆΠ΅Π½Π½ΠΎΠ΅ Ρ‚Π΅ΠΏΠ»ΠΎΠ²ΠΎΠ΅ ΠΏΠ΅Ρ€Π΅ΠΌΠ΅ΡˆΠΈΠ²Π°Π½ΠΈΠ΅ ΡƒΠ²Π΅Π»ΠΈΡ‡ΠΈΠ²Π°Π΅Ρ‚ количСство свободных зарядов, доступных для пСрСноса Ρ‚ΠΎΠΊΠ°. Π­Ρ‚ΠΎ свойство ΡƒΠΌΠ΅Π½ΡŒΡˆΠ΅Π½ΠΈΡ ρρ с Ρ‚Π΅ΠΌΠΏΠ΅Ρ€Π°Ρ‚ΡƒΡ€ΠΎΠΉ Ρ‚Π°ΠΊΠΆΠ΅ связано с Ρ‚ΠΈΠΏΠΎΠΌ ΠΈ количСством примСсСй, ΠΏΡ€ΠΈΡΡƒΡ‚ΡΡ‚Π²ΡƒΡŽΡ‰ΠΈΡ… Π² ΠΏΠΎΠ»ΡƒΠΏΡ€ΠΎΠ²ΠΎΠ΄Π½ΠΈΠΊΠ°Ρ….

Π‘ΠΎΠΏΡ€ΠΎΡ‚ΠΈΠ²Π»Π΅Π½ΠΈΠ΅

Π’Π΅ΠΏΠ΅Ρ€ΡŒ рассмотрим сопротивлСниС ΠΏΡ€ΠΎΠ²ΠΎΠ΄Π° ΠΈΠ»ΠΈ ΠΊΠΎΠΌΠΏΠΎΠ½Π΅Π½Ρ‚Π°.Π‘ΠΎΠΏΡ€ΠΎΡ‚ΠΈΠ²Π»Π΅Π½ΠΈΠ΅ — это ΠΌΠ΅Ρ€Π° Ρ‚ΠΎΠ³ΠΎ, насколько слоТно ΠΏΡ€ΠΎΠΏΡƒΡΡ‚ΠΈΡ‚ΡŒ Ρ‚ΠΎΠΊ Ρ‡Π΅Ρ€Π΅Π· ΠΏΡ€ΠΎΠ²ΠΎΠ΄ ΠΈΠ»ΠΈ ΠΊΠΎΠΌΠΏΠΎΠ½Π΅Π½Ρ‚. Π‘ΠΎΠΏΡ€ΠΎΡ‚ΠΈΠ²Π»Π΅Π½ΠΈΠ΅ зависит ΠΎΡ‚ ΡƒΠ΄Π΅Π»ΡŒΠ½ΠΎΠ³ΠΎ сопротивлСния. УдСльноС сопротивлСниС являСтся характСристикой ΠΌΠ°Ρ‚Π΅Ρ€ΠΈΠ°Π»Π°, ΠΈΡΠΏΠΎΠ»ΡŒΠ·ΡƒΠ΅ΠΌΠΎΠ³ΠΎ для изготовлСния ΠΏΡ€ΠΎΠ²ΠΎΠ΄Π° ΠΈΠ»ΠΈ Π΄Ρ€ΡƒΠ³ΠΎΠ³ΠΎ элСктричСского ΠΊΠΎΠΌΠΏΠΎΠ½Π΅Π½Ρ‚Π°, Ρ‚ΠΎΠ³Π΄Π° ΠΊΠ°ΠΊ сопротивлСниС являСтся характСристикой ΠΏΡ€ΠΎΠ²ΠΎΠ΄Π° ΠΈΠ»ΠΈ ΠΊΠΎΠΌΠΏΠΎΠ½Π΅Π½Ρ‚Π°.

Для расчСта сопротивлСния рассмотрим сСчСниС токопроводящСго ΠΏΡ€ΠΎΠ²ΠΎΠ΄Π° с ΠΏΠ»ΠΎΡ‰Π°Π΄ΡŒΡŽ ΠΏΠΎΠΏΠ΅Ρ€Π΅Ρ‡Π½ΠΎΠ³ΠΎ сСчСния A , Π΄Π»ΠΈΠ½ΠΎΠΉ L ΠΈ ΡƒΠ΄Π΅Π»ΡŒΠ½Ρ‹ΠΌ сопротивлСниСм ρ.ρ. БатарСя ΠΏΠΎΠ΄ΠΊΠ»ΡŽΡ‡Π°Π΅Ρ‚ΡΡ ΠΊ ΠΏΡ€ΠΎΠ²ΠΎΠ΄Π½ΠΈΠΊΡƒ, обСспСчивая Π½Π° Π½Π΅ΠΌ Ρ€Π°Π·Π½ΠΎΡΡ‚ΡŒ ΠΏΠΎΡ‚Π΅Π½Ρ†ΠΈΠ°Π»ΠΎΠ² Ξ”VΞ”V (Рисунок 9.13). Π Π°Π·Π½ΠΎΡΡ‚ΡŒ ΠΏΠΎΡ‚Π΅Π½Ρ†ΠΈΠ°Π»ΠΎΠ² создаСт элСктричСскоС ΠΏΠΎΠ»Π΅, ΠΊΠΎΡ‚ΠΎΡ€ΠΎΠ΅ ΠΏΡ€ΠΎΠΏΠΎΡ€Ρ†ΠΈΠΎΠ½Π°Π»ΡŒΠ½ΠΎ плотности Ρ‚ΠΎΠΊΠ°, согласно E β†’ = ρJ β†’ E β†’ = ρJ β†’.

Π€ΠΈΠ³ΡƒΡ€Π° 9,13 ΠŸΠΎΡ‚Π΅Π½Ρ†ΠΈΠ°Π», обСспСчиваСмый Π±Π°Ρ‚Π°Ρ€Π΅Π΅ΠΉ, прикладываСтся ΠΊ сСгмСнту ΠΏΡ€ΠΎΠ²ΠΎΠ΄Π½ΠΈΠΊΠ° с ΠΏΠ»ΠΎΡ‰Π°Π΄ΡŒΡŽ ΠΏΠΎΠΏΠ΅Ρ€Π΅Ρ‡Π½ΠΎΠ³ΠΎ сСчСния A ΠΈ Π΄Π»ΠΈΠ½ΠΎΠΉ L .

Π’Π΅Π»ΠΈΡ‡ΠΈΠ½Π° элСктричСского поля Π½Π° ΠΎΡ‚Ρ€Π΅Π·ΠΊΠ΅ ΠΏΡ€ΠΎΠ²ΠΎΠ΄Π½ΠΈΠΊΠ° Ρ€Π°Π²Π½Π° Π½Π°ΠΏΡ€ΡΠΆΠ΅Π½ΠΈΡŽ, Π΄Π΅Π»Π΅Π½Π½ΠΎΠΌΡƒ Π½Π° Π΄Π»ΠΈΠ½Ρƒ, E = V / LE = V / L, Π° Π²Π΅Π»ΠΈΡ‡ΠΈΠ½Π° плотности Ρ‚ΠΎΠΊΠ° Ρ€Π°Π²Π½Π° Ρ‚ΠΎΠΊΡƒ, Π΄Π΅Π»Π΅Π½Π½ΠΎΠΌΡƒ Π½Π° ΠΏΠΎΠΏΠ΅Ρ€Π΅Ρ‡Π½ΡƒΡŽ ΠΏΠ»ΠΎΡ‰Π°Π΄ΡŒ сСчСния, J = I / A.J = I / A. Π˜ΡΠΏΠΎΠ»ΡŒΠ·ΡƒΡ эту ΠΈΠ½Ρ„ΠΎΡ€ΠΌΠ°Ρ†ΠΈΡŽ ΠΈ вспоминая, Ρ‡Ρ‚ΠΎ элСктричСскоС ΠΏΠΎΠ»Π΅ ΠΏΡ€ΠΎΠΏΠΎΡ€Ρ†ΠΈΠΎΠ½Π°Π»ΡŒΠ½ΠΎ ΡƒΠ΄Π΅Π»ΡŒΠ½ΠΎΠΌΡƒ ΡΠΎΠΏΡ€ΠΎΡ‚ΠΈΠ²Π»Π΅Π½ΠΈΡŽ ΠΈ плотности Ρ‚ΠΎΠΊΠ°, ΠΌΡ‹ ΠΌΠΎΠΆΠ΅ΠΌ Π²ΠΈΠ΄Π΅Ρ‚ΡŒ, Ρ‡Ρ‚ΠΎ напряТСниС ΠΏΡ€ΠΎΠΏΠΎΡ€Ρ†ΠΈΠΎΠ½Π°Π»ΡŒΠ½ΠΎ Ρ‚ΠΎΠΊΡƒ:

E = ρJVL = ρIAV = (ρLA) I.E = ρJVL = ρIAV = (ρLA) I.

Π‘ΠΎΠΏΡ€ΠΎΡ‚ΠΈΠ²Π»Π΅Π½ΠΈΠ΅

ΠžΡ‚Π½ΠΎΡˆΠ΅Π½ΠΈΠ΅ напряТСния ΠΊ Ρ‚ΠΎΠΊΡƒ опрСдСляСтся ΠΊΠ°ΠΊ сопротивлСниС R :

Π‘ΠΎΠΏΡ€ΠΎΡ‚ΠΈΠ²Π»Π΅Π½ΠΈΠ΅ цилиндричСского сСгмСнта ΠΏΡ€ΠΎΠ²ΠΎΠ΄Π½ΠΈΠΊΠ° Ρ€Π°Π²Π½ΠΎ ΡƒΠ΄Π΅Π»ΡŒΠ½ΠΎΠΌΡƒ ΡΠΎΠΏΡ€ΠΎΡ‚ΠΈΠ²Π»Π΅Π½ΠΈΡŽ ΠΌΠ°Ρ‚Π΅Ρ€ΠΈΠ°Π»Π°, ΡƒΠΌΠ½ΠΎΠΆΠ΅Π½Π½ΠΎΠΌΡƒ Π½Π° Π΄Π»ΠΈΠ½Ρƒ, Π΄Π΅Π»Π΅Π½Π½ΡƒΡŽ Π½Π° ΠΏΠ»ΠΎΡ‰Π°Π΄ΡŒ:

R≑VI = ρLA.R≑VI = ρLA.

9.9

Π•Π΄ΠΈΠ½ΠΈΡ†Π΅ΠΉ измСрСния сопротивлСния являСтся ΠΎΠΌ, ΩΩ. Для Π·Π°Π΄Π°Π½Π½ΠΎΠ³ΠΎ напряТСния Ρ‡Π΅ΠΌ Π²Ρ‹ΡˆΠ΅ сопротивлСниС, Ρ‚Π΅ΠΌ Π½ΠΈΠΆΠ΅ Ρ‚ΠΎΠΊ.

РСзисторы

ΠžΠ±Ρ‹Ρ‡Π½Ρ‹ΠΌ ΠΊΠΎΠΌΠΏΠΎΠ½Π΅Π½Ρ‚ΠΎΠΌ элСктронных схСм являСтся рСзистор. РСзистор ΠΌΠΎΠΆΠ½ΠΎ ΠΈΡΠΏΠΎΠ»ΡŒΠ·ΠΎΠ²Π°Ρ‚ΡŒ для ΡƒΠΌΠ΅Π½ΡŒΡˆΠ΅Π½ΠΈΡ протСкания Ρ‚ΠΎΠΊΠ° ΠΈΠ»ΠΈ обСспСчСния падСния напряТСния. На рисункС 9.14 ΠΏΠΎΠΊΠ°Π·Π°Π½Ρ‹ символы, ΠΈΡΠΏΠΎΠ»ΡŒΠ·ΡƒΠ΅ΠΌΡ‹Π΅ для рСзистора Π² ΠΏΡ€ΠΈΠ½Ρ†ΠΈΠΏΠΈΠ°Π»ΡŒΠ½Ρ‹Ρ… схСмах Ρ†Π΅ΠΏΠΈ. Π”Π²Π° ΠΎΠ±Ρ‹Ρ‡Π½ΠΎ ΠΈΡΠΏΠΎΠ»ΡŒΠ·ΡƒΠ΅ΠΌΡ‹Ρ… стандарта для ΠΏΡ€ΠΈΠ½Ρ†ΠΈΠΏΠΈΠ°Π»ΡŒΠ½Ρ‹Ρ… схСм прСдоставлСны АмСриканским Π½Π°Ρ†ΠΈΠΎΠ½Π°Π»ΡŒΠ½Ρ‹ΠΌ институтом стандартов (ANSI, произносится ΠΊΠ°ΠΊ Β«AN-см.Β») И ΠœΠ΅ΠΆΠ΄ΡƒΠ½Π°Ρ€ΠΎΠ΄Π½ΠΎΠΉ элСктротСхничСской комиссиСй (IEC).ОбС систСмы ΠΎΠ±Ρ‹Ρ‡Π½ΠΎ ΠΈΡΠΏΠΎΠ»ΡŒΠ·ΡƒΡŽΡ‚ΡΡ. ΠœΡ‹ ΠΈΡΠΏΠΎΠ»ΡŒΠ·ΡƒΠ΅ΠΌ стандарт ANSI Π² этом тСкстС для Π΅Π³ΠΎ Π²ΠΈΠ·ΡƒΠ°Π»ΡŒΠ½ΠΎΠ³ΠΎ распознавания, Π½ΠΎ ΠΎΡ‚ΠΌΠ΅Ρ‚ΠΈΠΌ, Ρ‡Ρ‚ΠΎ для Π±ΠΎΠ»Π΅Π΅ ΠΊΡ€ΡƒΠΏΠ½Ρ‹Ρ… ΠΈ слоТных схСм стандарт IEC ΠΌΠΎΠΆΠ΅Ρ‚ ΠΈΠΌΠ΅Ρ‚ΡŒ Π±ΠΎΠ»Π΅Π΅ Ρ‡Π΅Ρ‚ΠΊΠΎΠ΅ прСдставлСниС, Ρ‡Ρ‚ΠΎ ΡƒΠΏΡ€ΠΎΡ‰Π°Π΅Ρ‚ Ρ‡Ρ‚Π΅Π½ΠΈΠ΅.

Π€ΠΈΠ³ΡƒΡ€Π° 9,14 ΠžΠ±ΠΎΠ·Π½Π°Ρ‡Π΅Π½ΠΈΡ рСзистора, ΠΈΡΠΏΠΎΠ»ΡŒΠ·ΡƒΠ΅ΠΌΠΎΠ³ΠΎ Π² ΠΏΡ€ΠΈΠ½Ρ†ΠΈΠΏΠΈΠ°Π»ΡŒΠ½Ρ‹Ρ… схСмах. (Π°) символ ANSI; (b) символ IEC.

Π—Π°Π²ΠΈΡΠΈΠΌΠΎΡΡ‚ΡŒ сопротивлСния ΠΎΡ‚ ΠΌΠ°Ρ‚Π΅Ρ€ΠΈΠ°Π»Π° ΠΈ Ρ„ΠΎΡ€ΠΌΡ‹

РСзистор ΠΌΠΎΠΆΠ½ΠΎ ΡΠΌΠΎΠ΄Π΅Π»ΠΈΡ€ΠΎΠ²Π°Ρ‚ΡŒ ΠΊΠ°ΠΊ Ρ†ΠΈΠ»ΠΈΠ½Π΄Ρ€ с ΠΏΠ»ΠΎΡ‰Π°Π΄ΡŒΡŽ ΠΏΠΎΠΏΠ΅Ρ€Π΅Ρ‡Π½ΠΎΠ³ΠΎ сСчСния A ΠΈ Π΄Π»ΠΈΠ½ΠΎΠΉ L , ΠΈΠ·Π³ΠΎΡ‚ΠΎΠ²Π»Π΅Π½Π½Ρ‹ΠΉ ΠΈΠ· ΠΌΠ°Ρ‚Π΅Ρ€ΠΈΠ°Π»Π° с ΡƒΠ΄Π΅Π»ΡŒΠ½Ρ‹ΠΌ сопротивлСниСм ρρ (Рисунок 9.15). Π‘ΠΎΠΏΡ€ΠΎΡ‚ΠΈΠ²Π»Π΅Π½ΠΈΠ΅ рСзистора R = ρLAR = ρLA.

Π€ΠΈΠ³ΡƒΡ€Π° 9.15 МодСль рСзистора Π² Π²ΠΈΠ΄Π΅ Π΅Π΄ΠΈΠ½ΠΎΠ³ΠΎ Ρ†ΠΈΠ»ΠΈΠ½Π΄Ρ€Π° Π΄Π»ΠΈΠ½ΠΎΠΉ L ΠΈ ΠΏΠ»ΠΎΡ‰Π°Π΄ΡŒΡŽ ΠΏΠΎΠΏΠ΅Ρ€Π΅Ρ‡Π½ΠΎΠ³ΠΎ сСчСния A . Π•Π³ΠΎ сопротивлСниС ΠΏΠΎΡ‚ΠΎΠΊΡƒ Ρ‚ΠΎΠΊΠ° Π°Π½Π°Π»ΠΎΠ³ΠΈΡ‡Π½ΠΎ ΡΠΎΠΏΡ€ΠΎΡ‚ΠΈΠ²Π»Π΅Π½ΠΈΡŽ Ρ‚Ρ€ΡƒΠ±Ρ‹ ΠΏΠΎΡ‚ΠΎΠΊΡƒ Тидкости. Π§Π΅ΠΌ Π΄Π»ΠΈΠ½Π½Π΅Π΅ Ρ†ΠΈΠ»ΠΈΠ½Π΄Ρ€, Ρ‚Π΅ΠΌ большС Π΅Π³ΠΎ сопротивлСниС. Π§Π΅ΠΌ большС Π΅Π³ΠΎ ΠΏΠ»ΠΎΡ‰Π°Π΄ΡŒ ΠΏΠΎΠΏΠ΅Ρ€Π΅Ρ‡Π½ΠΎΠ³ΠΎ сСчСния A , Ρ‚Π΅ΠΌ мСньшС Π΅Π³ΠΎ сопротивлСниС.

Π§Π°Ρ‰Π΅ всСго для изготовлСния рСзистора ΠΈΡΠΏΠΎΠ»ΡŒΠ·ΡƒΠ΅Ρ‚ΡΡ ΡƒΠ³Π»Π΅Ρ€ΠΎΠ΄.УглСродная Π΄ΠΎΡ€ΠΎΠΆΠΊΠ° Π½Π°ΠΌΠΎΡ‚Π°Π½Π° Π½Π° кСрамичСский сСрдСчник, ΠΊ Π½Π΅ΠΌΡƒ ΠΏΡ€ΠΈΠΊΡ€Π΅ΠΏΠ»Π΅Π½Ρ‹ Π΄Π²Π° ΠΌΠ΅Π΄Π½Ρ‹Ρ… ΠΏΡ€ΠΎΠ²ΠΎΠ΄Π°. Π’Ρ‚ΠΎΡ€ΠΎΠΉ Ρ‚ΠΈΠΏ рСзистора — это ΠΌΠ΅Ρ‚Π°Π»Π»ΠΎΠΏΠ»Π΅Π½ΠΎΡ‡Π½Ρ‹ΠΉ рСзистор, ΠΊΠΎΡ‚ΠΎΡ€Ρ‹ΠΉ Ρ‚Π°ΠΊΠΆΠ΅ ΠΈΠΌΠ΅Π΅Ρ‚ кСрамичСский сСрдСчник. Π”ΠΎΡ€ΠΎΠΆΠΊΠ° сдСлана ΠΈΠ· ΠΌΠ°Ρ‚Π΅Ρ€ΠΈΠ°Π»Π° оксида ΠΌΠ΅Ρ‚Π°Π»Π»Π°, ΠΊΠΎΡ‚ΠΎΡ€Ρ‹ΠΉ ΠΈΠΌΠ΅Π΅Ρ‚ ΠΏΠΎΠ»ΡƒΠΏΡ€ΠΎΠ²ΠΎΠ΄Π½ΠΈΠΊΠΎΠ²Ρ‹Π΅ свойства, Π°Π½Π°Π»ΠΎΠ³ΠΈΡ‡Π½Ρ‹Π΅ ΡƒΠ³Π»Π΅Ρ€ΠΎΠ΄Π½Ρ‹ΠΌ. ΠžΠΏΡΡ‚ΡŒ ΠΆΠ΅, Π² ΠΊΠΎΠ½Ρ†Ρ‹ рСзистора Π²ΡΡ‚Π°Π²Π»ΡΡŽΡ‚ΡΡ ΠΌΠ΅Π΄Π½Ρ‹Π΅ ΠΏΡ€ΠΎΠ²ΠΎΠ΄Π°. Π—Π°Ρ‚Π΅ΠΌ рСзистор ΠΎΠΊΡ€Π°ΡˆΠΈΠ²Π°Π΅Ρ‚ΡΡ ΠΈ маркируСтся для ΠΈΠ΄Π΅Π½Ρ‚ΠΈΡ„ΠΈΠΊΠ°Ρ†ΠΈΠΈ. РСзистор ΠΈΠΌΠ΅Π΅Ρ‚ Ρ‡Π΅Ρ‚Ρ‹Ρ€Π΅ Ρ†Π²Π΅Ρ‚Π½Ρ‹Π΅ полосы, ΠΊΠ°ΠΊ ΠΏΠΎΠΊΠ°Π·Π°Π½ΠΎ Π½Π° рисункС 9.16.

Π€ΠΈΠ³ΡƒΡ€Π° 9,16 МногиС рСзисторы ΠΈΠΌΠ΅ΡŽΡ‚ Π²ΠΈΠ΄, ΠΏΠΎΠΊΠ°Π·Π°Π½Π½Ρ‹ΠΉ Π½Π° рисункС Π²Ρ‹ΡˆΠ΅.Π§Π΅Ρ‚Ρ‹Ρ€Π΅ полосы ΠΈΡΠΏΠΎΠ»ΡŒΠ·ΡƒΡŽΡ‚ΡΡ для ΠΈΠ΄Π΅Π½Ρ‚ΠΈΡ„ΠΈΠΊΠ°Ρ†ΠΈΠΈ рСзистора. ΠŸΠ΅Ρ€Π²Ρ‹Π΅ Π΄Π²Π΅ Ρ†Π²Π΅Ρ‚Π½Ρ‹Π΅ полосы ΠΏΡ€Π΅Π΄ΡΡ‚Π°Π²Π»ΡΡŽΡ‚ собой ΠΏΠ΅Ρ€Π²Ρ‹Π΅ Π΄Π²Π΅ Ρ†ΠΈΡ„Ρ€Ρ‹ сопротивлСния рСзистора. Π’Ρ€Π΅Ρ‚ΠΈΠΉ Ρ†Π²Π΅Ρ‚ — ΠΌΠ½ΠΎΠΆΠΈΡ‚Π΅Π»ΡŒ. Π§Π΅Ρ‚Π²Π΅Ρ€Ρ‚Ρ‹ΠΉ Ρ†Π²Π΅Ρ‚ ΠΎΠ±ΠΎΠ·Π½Π°Ρ‡Π°Π΅Ρ‚ допуск рСзистора. ΠŸΠΎΠΊΠ°Π·Π°Π½Π½Ρ‹ΠΉ рСзистор ΠΈΠΌΠ΅Π΅Ρ‚ сопротивлСниС 20 Γ— 105 Ом Β± 10% 20 Γ— 105 Ом Β± 10%.

БопротивлСния Π²Π°Ρ€ΡŒΠΈΡ€ΡƒΡŽΡ‚ΡΡ Π½Π° ΠΌΠ½ΠΎΠ³ΠΎ порядков. НСкоторыС кСрамичСскиС изоляторы, Π½Π°ΠΏΡ€ΠΈΠΌΠ΅Ρ€ Ρ‚Π΅, ΠΊΠΎΡ‚ΠΎΡ€Ρ‹Π΅ ΠΈΡΠΏΠΎΠ»ΡŒΠ·ΡƒΡŽΡ‚ΡΡ для ΠΏΠΎΠ΄Π΄Π΅Ρ€ΠΆΠΊΠΈ Π»ΠΈΠ½ΠΈΠΉ элСктропСрСдач, ΠΈΠΌΠ΅ΡŽΡ‚ сопротивлСниС 1012 Ом 10 12 Ом ΠΈΠ»ΠΈ Π±ΠΎΠ»Π΅Π΅. Π‘ΠΎΠΏΡ€ΠΎΡ‚ΠΈΠ²Π»Π΅Π½ΠΈΠ΅ сухого Ρ‡Π΅Π»ΠΎΠ²Π΅ΠΊΠ° ΠΌΠΎΠΆΠ΅Ρ‚ ΡΠΎΡΡ‚Π°Π²Π»ΡΡ‚ΡŒ 105 Ом 105 Ом, Π² Ρ‚ΠΎ врСмя ΠΊΠ°ΠΊ сопротивлСниС чСловСчСского сСрдца составляСт ΠΎΠΊΠΎΠ»ΠΎ 103 Ом 103 Ом.ΠšΡƒΡΠΎΠΊ ΠΌΠ΅Π΄Π½ΠΎΠ³ΠΎ ΠΏΡ€ΠΎΠ²ΠΎΠ΄Π° большого Π΄ΠΈΠ°ΠΌΠ΅Ρ‚Ρ€Π° Π΄Π»ΠΈΠ½ΠΎΠΉ Π² ΠΌΠ΅Ρ‚Ρ€ ΠΌΠΎΠΆΠ΅Ρ‚ ΠΈΠΌΠ΅Ρ‚ΡŒ сопротивлСниС 10-5 Ом10-5 Ом, Π° свСрхпроводники Π²ΠΎΠΎΠ±Ρ‰Π΅ Π½Π΅ ΠΈΠΌΠ΅ΡŽΡ‚ сопротивлСния ΠΏΡ€ΠΈ Π½ΠΈΠ·ΠΊΠΈΡ… Ρ‚Π΅ΠΌΠΏΠ΅Ρ€Π°Ρ‚ΡƒΡ€Π°Ρ…. Как ΠΌΡ‹ Π²ΠΈΠ΄Π΅Π»ΠΈ, сопротивлСниС связано с Ρ„ΠΎΡ€ΠΌΠΎΠΉ ΠΎΠ±ΡŠΠ΅ΠΊΡ‚Π° ΠΈ ΠΌΠ°Ρ‚Π΅Ρ€ΠΈΠ°Π»ΠΎΠΌ, ΠΈΠ· ΠΊΠΎΡ‚ΠΎΡ€ΠΎΠ³ΠΎ ΠΎΠ½ состоит.

ΠŸΡ€ΠΈΠΌΠ΅Ρ€ 9,5

ΠŸΠ»ΠΎΡ‚Π½ΠΎΡΡ‚ΡŒ Ρ‚ΠΎΠΊΠ°, сопротивлСниС ΠΈ элСктричСскоС ΠΏΠΎΠ»Π΅ для Ρ‚ΠΎΠΊΠΎΠ²Π΅Π΄ΡƒΡ‰Π΅Π³ΠΎ ΠΏΡ€ΠΎΠ²ΠΎΠ΄Π°
РассчитайтС ΠΏΠ»ΠΎΡ‚Π½ΠΎΡΡ‚ΡŒ Ρ‚ΠΎΠΊΠ°, сопротивлСниС ΠΈ элСктричСскоС ΠΏΠΎΠ»Π΅ ΠΌΠ΅Π΄Π½ΠΎΠ³ΠΎ ΠΏΡ€ΠΎΠ²ΠΎΠ΄Π° Π΄Π»ΠΈΠ½ΠΎΠΉ 5 ΠΌ ΠΈ Π΄ΠΈΠ°ΠΌΠ΅Ρ‚Ρ€ΠΎΠΌ 2.053 ΠΌΠΌ (ΠΊΠ°Π»ΠΈΠ±Ρ€ 12), ΠΏΡ€ΠΎΠΏΡƒΡΠΊΠ°ΡŽΡ‰ΠΈΠΉ Ρ‚ΠΎΠΊ I = 10 мА I = 10 мА.
БтратСгия
ΠœΡ‹ ΠΌΠΎΠΆΠ΅ΠΌ Ρ€Π°ΡΡΡ‡ΠΈΡ‚Π°Ρ‚ΡŒ ΠΏΠ»ΠΎΡ‚Π½ΠΎΡΡ‚ΡŒ Ρ‚ΠΎΠΊΠ°, сначала найдя ΠΏΠ»ΠΎΡ‰Π°Π΄ΡŒ ΠΏΠΎΠΏΠ΅Ρ€Π΅Ρ‡Π½ΠΎΠ³ΠΎ сСчСния ΠΏΡ€ΠΎΠ²ΠΎΠ΄Π°, которая составляСт A = 3,31 ΠΌΠΌ2, A = 3,31 ΠΌΠΌ2, ΠΈ ΠΎΠΏΡ€Π΅Π΄Π΅Π»ΠΈΠ² ΠΏΠ»ΠΎΡ‚Π½ΠΎΡΡ‚ΡŒ Ρ‚ΠΎΠΊΠ° J = IAJ = IA. Π‘ΠΎΠΏΡ€ΠΎΡ‚ΠΈΠ²Π»Π΅Π½ΠΈΠ΅ ΠΌΠΎΠΆΠ½ΠΎ Π½Π°ΠΉΡ‚ΠΈ, ΠΈΡΠΏΠΎΠ»ΡŒΠ·ΡƒΡ Π΄Π»ΠΈΠ½Ρƒ ΠΏΡ€ΠΎΠ²ΠΎΠ΄Π° L = 5,00 ΠΌΠ» = 5,00 ΠΌ, ΠΏΠ»ΠΎΡ‰Π°Π΄ΡŒ ΠΈ ΡƒΠ΄Π΅Π»ΡŒΠ½ΠΎΠ΅ сопротивлСниС ΠΌΠ΅Π΄ΠΈ ρ = 1,68 Γ— 10-8 Ом Β· мρ = 1,68 Γ— 10-8 Ом Β· ΠΌ, Π³Π΄Π΅ R = ρLAR = ρLA. УдСльноС сопротивлСниС ΠΈ ΠΏΠ»ΠΎΡ‚Π½ΠΎΡΡ‚ΡŒ Ρ‚ΠΎΠΊΠ° ΠΌΠΎΠΆΠ½ΠΎ ΠΈΡΠΏΠΎΠ»ΡŒΠ·ΠΎΠ²Π°Ρ‚ΡŒ для опрСдСлСния элСктричСского поля.
РСшСниС
Π‘Π½Π°Ρ‡Π°Π»Π° рассчитываСм ΠΏΠ»ΠΎΡ‚Π½ΠΎΡΡ‚ΡŒ Ρ‚ΠΎΠΊΠ°: J = IA = 10 Γ— 10βˆ’3A3.31 Γ— 10βˆ’6m2 = 3,02 Γ— 103Am2.J = IA = 10 Γ— 10βˆ’3A3,31 Γ— 10βˆ’6m2 = 3,02 Γ— 103Am2.

Π‘ΠΎΠΏΡ€ΠΎΡ‚ΠΈΠ²Π»Π΅Π½ΠΈΠ΅ ΠΏΡ€ΠΎΠ²ΠΎΠ΄Π°

R = ρLA = (1,68 Γ— 10–8 Ом Β· ΠΌ) 5,00 ΠΌ3,31 Γ— 10–6 ΠΌ2 = 0,025 Ом. R = ρLA = (1,68 Γ— 10–8 Ом Β· ΠΌ) 5,00 ΠΌ3,31 Γ— 10–6 ΠΌ2 = 0,025 Ом.

НаконСц, ΠΌΡ‹ ΠΌΠΎΠΆΠ΅ΠΌ Π½Π°ΠΉΡ‚ΠΈ элСктричСскоС ΠΏΠΎΠ»Π΅:

E = ρJ = 1,68 Γ— 10–8 Ом Β· ΠΌ (3,02 Γ— 103Am2) = 5,07 Γ— 10–5Vm. E = ρJ = 1,68 Γ— 10–8 Ом Β· ΠΌ (3,02 Γ— 103Am2) = 5,07 Γ— 10–5Vm.
Π—Π½Π°Ρ‡Π΅Π½ΠΈΠ΅
Π˜ΡΡ…ΠΎΠ΄Ρ ΠΈΠ· этих Ρ€Π΅Π·ΡƒΠ»ΡŒΡ‚Π°Ρ‚ΠΎΠ², Π½Π΅ΡƒΠ΄ΠΈΠ²ΠΈΡ‚Π΅Π»ΡŒΠ½ΠΎ, Ρ‡Ρ‚ΠΎ мСдь ΠΈΡΠΏΠΎΠ»ΡŒΠ·ΡƒΠ΅Ρ‚ΡΡ для ΠΏΡ€ΠΎΠ²ΠΎΠ΄ΠΎΠ², проводящих Ρ‚ΠΎΠΊ, ΠΏΠΎΡ‚ΠΎΠΌΡƒ Ρ‡Ρ‚ΠΎ сопротивлСниС довольно ΠΌΠ°Π»ΠΎ. ΠžΠ±Ρ€Π°Ρ‚ΠΈΡ‚Π΅ Π²Π½ΠΈΠΌΠ°Π½ΠΈΠ΅, Ρ‡Ρ‚ΠΎ ΠΏΠ»ΠΎΡ‚Π½ΠΎΡΡ‚ΡŒ Ρ‚ΠΎΠΊΠ° ΠΈ элСктричСскоС ΠΏΠΎΠ»Π΅ Π½Π΅ зависят ΠΎΡ‚ Π΄Π»ΠΈΠ½Ρ‹ ΠΏΡ€ΠΎΠ²ΠΎΠ΄Π°, Π½ΠΎ напряТСниС зависит ΠΎΡ‚ Π΄Π»ΠΈΠ½Ρ‹.

Π‘ΠΎΠΏΡ€ΠΎΡ‚ΠΈΠ²Π»Π΅Π½ΠΈΠ΅ ΠΎΠ±ΡŠΠ΅ΠΊΡ‚Π° Ρ‚Π°ΠΊΠΆΠ΅ зависит ΠΎΡ‚ Ρ‚Π΅ΠΌΠΏΠ΅Ρ€Π°Ρ‚ΡƒΡ€Ρ‹, ΠΏΠΎΡΠΊΠΎΠ»ΡŒΠΊΡƒ R0R0 прямо ΠΏΡ€ΠΎΠΏΠΎΡ€Ρ†ΠΈΠΎΠ½Π°Π»ΡŒΠ½ΠΎ ρ.ρ. Для Ρ†ΠΈΠ»ΠΈΠ½Π΄Ρ€Π° ΠΌΡ‹ Π·Π½Π°Π΅ΠΌ, Ρ‡Ρ‚ΠΎ R = ρLAR = ρLA, поэтому, Ссли L ΠΈ A Π½Π΅ сильно ΠΈΠ·ΠΌΠ΅Π½ΡΡŽΡ‚ΡΡ с Ρ‚Π΅ΠΌΠΏΠ΅Ρ€Π°Ρ‚ΡƒΡ€ΠΎΠΉ, R ΠΈΠΌΠ΅Π΅Ρ‚ Ρ‚Ρƒ ΠΆΠ΅ Ρ‚Π΅ΠΌΠΏΠ΅Ρ€Π°Ρ‚ΡƒΡ€Π½ΡƒΡŽ Π·Π°Π²ΠΈΡΠΈΠΌΠΎΡΡ‚ΡŒ, Ρ‡Ρ‚ΠΎ ΠΈ ρ.ρ. (ИсслСдованиС коэффициСнтов Π»ΠΈΠ½Π΅ΠΉΠ½ΠΎΠ³ΠΎ Ρ€Π°ΡΡˆΠΈΡ€Π΅Π½ΠΈΡ ΠΏΠΎΠΊΠ°Π·Ρ‹Π²Π°Π΅Ρ‚, Ρ‡Ρ‚ΠΎ ΠΎΠ½ΠΈ ΠΏΡ€ΠΈΠΌΠ΅Ρ€Π½ΠΎ Π½Π° Π΄Π²Π° порядка мСньшС Ρ‚ΠΈΠΏΠΈΡ‡Π½Ρ‹Ρ… Ρ‚Π΅ΠΌΠΏΠ΅Ρ€Π°Ρ‚ΡƒΡ€Π½Ρ‹Ρ… коэффициСнтов ΡƒΠ΄Π΅Π»ΡŒΠ½ΠΎΠ³ΠΎ сопротивлСния, поэтому влияниС Ρ‚Π΅ΠΌΠΏΠ΅Ρ€Π°Ρ‚ΡƒΡ€Ρ‹ Π½Π° L ΠΈ A ΠΏΡ€ΠΈΠΌΠ΅Ρ€Π½ΠΎ Π½Π° Π΄Π²Π° порядка мСньшС, Ρ‡Π΅ΠΌ Π½Π° ρ.) ρ.) Π’Π°ΠΊΠΈΠΌ ΠΎΠ±Ρ€Π°Π·ΠΎΠΌ,

R = R0 (1 + Ξ±Ξ”T) R = R0 (1 + Ξ±Ξ”T)

9.10

— это тСмпСратурная Π·Π°Π²ΠΈΡΠΈΠΌΠΎΡΡ‚ΡŒ сопротивлСния ΠΎΠ±ΡŠΠ΅ΠΊΡ‚Π°, Π³Π΄Π΅ R0R0 — исходноС сопротивлСниС (ΠΎΠ±Ρ‹Ρ‡Π½ΠΎ ΠΏΡ€ΠΈΠ½ΠΈΠΌΠ°Π΅ΠΌΠΎΠ΅ Ρ€Π°Π²Π½Ρ‹ΠΌ 20,00 Β° C) 20,00 Β° C), Π° R — сопротивлСниС послС измСнСния Ρ‚Π΅ΠΌΠΏΠ΅Ρ€Π°Ρ‚ΡƒΡ€Ρ‹ Ξ”T.Ξ”T. Π¦Π²Π΅Ρ‚ΠΎΠ²ΠΎΠΉ ΠΊΠΎΠ΄ ΠΏΠΎΠΊΠ°Π·Ρ‹Π²Π°Π΅Ρ‚ сопротивлСниС рСзистора ΠΏΡ€ΠΈ Ρ‚Π΅ΠΌΠΏΠ΅Ρ€Π°Ρ‚ΡƒΡ€Π΅ T = 20,00 Β° CT = 20,00 Β° C.

МногиС Ρ‚Π΅Ρ€ΠΌΠΎΠΌΠ΅Ρ‚Ρ€Ρ‹ основаны Π½Π° влиянии Ρ‚Π΅ΠΌΠΏΠ΅Ρ€Π°Ρ‚ΡƒΡ€Ρ‹ Π½Π° сопротивлСниС (рисунок 9.17). Один ΠΈΠ· Π½Π°ΠΈΠ±ΠΎΠ»Π΅Π΅ распространСнных Ρ‚Π΅Ρ€ΠΌΠΎΠΌΠ΅Ρ‚Ρ€ΠΎΠ² основан Π½Π° тСрмисторС, ΠΏΠΎΠ»ΡƒΠΏΡ€ΠΎΠ²ΠΎΠ΄Π½ΠΈΠΊΠΎΠ²ΠΎΠΌ кристаллС с сильной Ρ‚Π΅ΠΌΠΏΠ΅Ρ€Π°Ρ‚ΡƒΡ€Π½ΠΎΠΉ Π·Π°Π²ΠΈΡΠΈΠΌΠΎΡΡ‚ΡŒΡŽ, сопротивлСниС ΠΊΠΎΡ‚ΠΎΡ€ΠΎΠ³ΠΎ измСряСтся для опрСдСлСния Π΅Π³ΠΎ Ρ‚Π΅ΠΌΠΏΠ΅Ρ€Π°Ρ‚ΡƒΡ€Ρ‹.Устройство нСбольшоС, поэтому быстро ΠΏΡ€ΠΈΡ…ΠΎΠ΄ΠΈΡ‚ Π² Ρ‚Π΅ΠΏΠ»ΠΎΠ²ΠΎΠ΅ равновСсиС с Ρ‚ΠΎΠΉ Ρ‡Π°ΡΡ‚ΡŒΡŽ Ρ‡Π΅Π»ΠΎΠ²Π΅ΠΊΠ°, ΠΊ ΠΊΠΎΡ‚ΠΎΡ€ΠΎΠΉ прикасаСтся.

Π€ΠΈΠ³ΡƒΡ€Π° 9,17 Π­Ρ‚ΠΈ Π·Π½Π°ΠΊΠΎΠΌΡ‹Π΅ Ρ‚Π΅Ρ€ΠΌΠΎΠΌΠ΅Ρ‚Ρ€Ρ‹ основаны Π½Π° автоматичСском ΠΈΠ·ΠΌΠ΅Ρ€Π΅Π½ΠΈΠΈ сопротивлСния тСрмистора Π² зависимости ΠΎΡ‚ Ρ‚Π΅ΠΌΠΏΠ΅Ρ€Π°Ρ‚ΡƒΡ€Ρ‹.

ΠŸΡ€ΠΈΠΌΠ΅Ρ€ 9,6

РасчСт сопротивлСния
Π₯отя слСдуСт ΡΠΎΠ±Π»ΡŽΠ΄Π°Ρ‚ΡŒ ΠΎΡΡ‚ΠΎΡ€ΠΎΠΆΠ½ΠΎΡΡ‚ΡŒ ΠΏΡ€ΠΈ ΠΏΡ€ΠΈΠΌΠ΅Π½Π΅Π½ΠΈΠΈ ρ = ρ0 (1 + Ξ±Ξ”T), ρ = ρ0 (1 + Ξ±Ξ”T) ΠΈ R = R0 (1 + Ξ±Ξ”T) R = R0 (1 + Ξ±Ξ”T) для Ρ‚Π΅ΠΌΠΏΠ΅Ρ€Π°Ρ‚ΡƒΡ€Π½Ρ‹Ρ… ΠΈΠ·ΠΌΠ΅Π½Π΅Π½ΠΈΠΉ Π±ΠΎΠ»Π΅Π΅ 100 Β° C 100 Β° C , для Π²ΠΎΠ»ΡŒΡ„Ρ€Π°ΠΌΠ° уравнСния достаточно Ρ…ΠΎΡ€ΠΎΡˆΠΎ Ρ€Π°Π±ΠΎΡ‚Π°ΡŽΡ‚ ΠΏΡ€ΠΈ ΠΎΡ‡Π΅Π½ΡŒ Π±ΠΎΠ»ΡŒΡˆΠΈΡ… измСнСниях Ρ‚Π΅ΠΌΠΏΠ΅Ρ€Π°Ρ‚ΡƒΡ€Ρ‹.Π’ΠΎΠ»ΡŒΡ„Ρ€Π°ΠΌΠΎΠ²Π°Ρ Π½ΠΈΡ‚ΡŒ Π½Π°ΠΊΠ°Π»Π° ΠΏΡ€ΠΈ 20 Β° C20 Β° C ΠΈΠΌΠ΅Π΅Ρ‚ сопротивлСниС 0,350 Ом 0,350 Ом. Каким Π±ΡƒΠ΄Π΅Ρ‚ сопротивлСниС ΠΏΡ€ΠΈ ΠΏΠΎΠ²Ρ‹ΡˆΠ΅Π½ΠΈΠΈ Ρ‚Π΅ΠΌΠΏΠ΅Ρ€Π°Ρ‚ΡƒΡ€Ρ‹ Π΄ΠΎ 2850 Β° C 2850 Β° C?
БтратСгия
Π­Ρ‚ΠΎ прямоС ΠΏΡ€ΠΈΠΌΠ΅Π½Π΅Π½ΠΈΠ΅ R = R0 (1 + Ξ±Ξ”T) R = R0 (1 + Ξ±Ξ”T), ΠΏΠΎΡΠΊΠΎΠ»ΡŒΠΊΡƒ исходноС сопротивлСниС Π½ΠΈΡ‚ΠΈ Π½Π°ΠΊΠ°Π»Π° задаСтся ΠΊΠ°ΠΊ R0 = 0,350Ξ©R0 = 0,350Ξ©, Π° ΠΈΠ·ΠΌΠ΅Π½Π΅Π½ΠΈΠ΅ Ρ‚Π΅ΠΌΠΏΠ΅Ρ€Π°Ρ‚ΡƒΡ€Ρ‹ составляСт Ξ”T = 2830 Β° CΞ”T. = 2830 Β° Π‘.
РСшСниС
Π‘ΠΎΠΏΡ€ΠΎΡ‚ΠΈΠ²Π»Π΅Π½ΠΈΠ΅ Π±ΠΎΠ»Π΅Π΅ горячСй Π½ΠΈΡ‚ΠΈ Π½Π°ΠΊΠ°Π»Π° R получаСтся ΠΏΡƒΡ‚Π΅ΠΌ Π²Π²ΠΎΠ΄Π° извСстных Π·Π½Π°Ρ‡Π΅Π½ΠΈΠΉ Π² ΠΏΡ€ΠΈΠ²Π΅Π΄Π΅Π½Π½ΠΎΠ΅ Π²Ρ‹ΡˆΠ΅ ΡƒΡ€Π°Π²Π½Π΅Π½ΠΈΠ΅: R = R0 (1 + Ξ±Ξ”T) = (0.350 Ом) [1+ (4,5 Γ— 10βˆ’3 Β° C) (2830 Β° C)] = 4,8 Ом.R = R0 (1 + Ξ±Ξ”T) = (0,350 Ом) [1+ (4,5 Γ— 10βˆ’3 Β° C) ( 2830 Β° C)] = 4,8 Ом.
Π—Π½Π°Ρ‡Π΅Π½ΠΈΠ΅
ΠžΠ±Ρ€Π°Ρ‚ΠΈΡ‚Π΅ Π²Π½ΠΈΠΌΠ°Π½ΠΈΠ΅, Ρ‡Ρ‚ΠΎ сопротивлСниС измСняСтся Π±ΠΎΠ»Π΅Π΅ Ρ‡Π΅ΠΌ Π² 10 Ρ€Π°Π·, ΠΊΠΎΠ³Π΄Π° Π½ΠΈΡ‚ΡŒ Π½Π°ΠΊΠ°Π»Π° нагрСваСтся Π΄ΠΎ высокой Ρ‚Π΅ΠΌΠΏΠ΅Ρ€Π°Ρ‚ΡƒΡ€Ρ‹, Π° Ρ‚ΠΎΠΊ Ρ‡Π΅Ρ€Π΅Π· Π½ΠΈΡ‚ΡŒ зависит ΠΎΡ‚ сопротивлСния Π½ΠΈΡ‚ΠΈ ΠΈ ΠΏΡ€ΠΈΠ»ΠΎΠΆΠ΅Π½Π½ΠΎΠ³ΠΎ напряТСния. Если Π½ΠΈΡ‚ΡŒ накаливания ΠΈΡΠΏΠΎΠ»ΡŒΠ·ΡƒΠ΅Ρ‚ΡΡ Π² Π»Π°ΠΌΠΏΠ΅ накаливания, Π½Π°Ρ‡Π°Π»ΡŒΠ½Ρ‹ΠΉ Ρ‚ΠΎΠΊ Ρ‡Π΅Ρ€Π΅Π· Π½ΠΈΡ‚ΡŒ Π½Π°ΠΊΠ°Π»Π° ΠΏΡ€ΠΈ ΠΏΠ΅Ρ€Π²ΠΎΠΌ Π²ΠΊΠ»ΡŽΡ‡Π΅Π½ΠΈΠΈ Π»Π°ΠΌΠΏΡ‹ Π±ΡƒΠ΄Π΅Ρ‚ Π²Ρ‹ΡˆΠ΅, Ρ‡Π΅ΠΌ Ρ‚ΠΎΠΊ послС Ρ‚ΠΎΠ³ΠΎ, ΠΊΠ°ΠΊ Π½ΠΈΡ‚ΡŒ Π½Π°ΠΊΠ°Π»Π° достигнСт Ρ€Π°Π±ΠΎΡ‡Π΅ΠΉ Ρ‚Π΅ΠΌΠΏΠ΅Ρ€Π°Ρ‚ΡƒΡ€Ρ‹.

ΠŸΡ€ΠΎΠ²Π΅Ρ€ΡŒΡ‚Π΅ своС ΠΏΠΎΠ½ΠΈΠΌΠ°Π½ΠΈΠ΅ 9,6

ΠŸΡ€ΠΎΠ²Π΅Ρ€ΡŒΡ‚Π΅ свои знания Π’Π΅Π½Π·ΠΎΠ΄Π°Ρ‚Ρ‡ΠΈΠΊ — это элСктричСскоС устройство для измСрСния Π΄Π΅Ρ„ΠΎΡ€ΠΌΠ°Ρ†ΠΈΠΈ, ΠΊΠ°ΠΊ ΠΏΠΎΠΊΠ°Π·Π°Π½ΠΎ Π½ΠΈΠΆΠ΅. Он состоит ΠΈΠ· Π³ΠΈΠ±ΠΊΠΎΠΉ ΠΈΠ·ΠΎΠ»ΠΈΡ€ΡƒΡŽΡ‰Π΅ΠΉ основы, ΠΏΠΎΠ΄Π΄Π΅Ρ€ΠΆΠΈΠ²Π°ΡŽΡ‰Π΅ΠΉ рисунок ΠΈΠ· проводящСй Ρ„ΠΎΠ»ΡŒΠ³ΠΈ. Π‘ΠΎΠΏΡ€ΠΎΡ‚ΠΈΠ²Π»Π΅Π½ΠΈΠ΅ Ρ„ΠΎΠ»ΡŒΠ³ΠΈ измСняСтся ΠΏΠΎ ΠΌΠ΅Ρ€Π΅ растяТСния основы. Как мСняСтся сопротивлСниС Ρ‚Π΅Π½Π·ΠΎΠ΄Π°Ρ‚Ρ‡ΠΈΠΊΠ°? ВлияСт Π»ΠΈ Ρ‚Π΅Π½Π·ΠΎΠ΄Π°Ρ‚Ρ‡ΠΈΠΊ Π½Π° ΠΈΠ·ΠΌΠ΅Π½Π΅Π½ΠΈΠ΅ Ρ‚Π΅ΠΌΠΏΠ΅Ρ€Π°Ρ‚ΡƒΡ€Ρ‹?

ΠŸΡ€ΠΈΠΌΠ΅Ρ€ 9,7

Π‘ΠΎΠΏΡ€ΠΎΡ‚ΠΈΠ²Π»Π΅Π½ΠΈΠ΅ коаксиального кабСля
Π”Π»ΠΈΠ½Π½Ρ‹Π΅ ΠΊΠ°Π±Π΅Π»ΠΈ ΠΈΠ½ΠΎΠ³Π΄Π° ΠΌΠΎΠ³ΡƒΡ‚ Π΄Π΅ΠΉΡΡ‚Π²ΠΎΠ²Π°Ρ‚ΡŒ ΠΊΠ°ΠΊ Π°Π½Ρ‚Π΅Π½Π½Ρ‹, улавливая элСктронныС ΡˆΡƒΠΌΡ‹, ΠΊΠΎΡ‚ΠΎΡ€Ρ‹Π΅ ΡΠ²Π»ΡΡŽΡ‚ΡΡ сигналами ΠΎΡ‚ Π΄Ρ€ΡƒΠ³ΠΎΠ³ΠΎ оборудования ΠΈ ΠΏΡ€ΠΈΠ±ΠΎΡ€ΠΎΠ².ΠšΠΎΠ°ΠΊΡΠΈΠ°Π»ΡŒΠ½Ρ‹Π΅ ΠΊΠ°Π±Π΅Π»ΠΈ ΠΈΡΠΏΠΎΠ»ΡŒΠ·ΡƒΡŽΡ‚ΡΡ Π²ΠΎ ΠΌΠ½ΠΎΠ³ΠΈΡ… случаях, ΠΊΠΎΠ³Π΄Π° трСбуСтся устранСниС этого ΡˆΡƒΠΌΠ°. НапримСр, ΠΈΡ… ΠΌΠΎΠΆΠ½ΠΎ Π½Π°ΠΉΡ‚ΠΈ Π΄ΠΎΠΌΠ° Ρ‡Π΅Ρ€Π΅Π· кабСльноС Ρ‚Π΅Π»Π΅Π²ΠΈΠ΄Π΅Π½ΠΈΠ΅ ΠΈΠ»ΠΈ Π΄Ρ€ΡƒΠ³ΠΈΠ΅ Π°ΡƒΠ΄ΠΈΠΎΠ²ΠΈΠ·ΡƒΠ°Π»ΡŒΠ½Ρ‹Π΅ соСдинСния. ΠšΠΎΠ°ΠΊΡΠΈΠ°Π»ΡŒΠ½Ρ‹Π΅ ΠΊΠ°Π±Π΅Π»ΠΈ состоят ΠΈΠ· Π²Π½ΡƒΡ‚Ρ€Π΅Π½Π½Π΅Π³ΠΎ ΠΏΡ€ΠΎΠ²ΠΎΠ΄Π½ΠΈΠΊΠ° с радиусом riri, ΠΎΠΊΡ€ΡƒΠΆΠ΅Π½Π½ΠΎΠ³ΠΎ Π²Ρ‚ΠΎΡ€Ρ‹ΠΌ, внСшним концСнтричСским ΠΏΡ€ΠΎΠ²ΠΎΠ΄Π½ΠΈΠΊΠΎΠΌ с радиусом roro (рисунок 9.18). ΠŸΡ€ΠΎΡΡ‚Ρ€Π°Π½ΡΡ‚Π²ΠΎ ΠΌΠ΅ΠΆΠ΄Ρƒ Π½ΠΈΠΌΠΈ ΠΎΠ±Ρ‹Ρ‡Π½ΠΎ Π·Π°ΠΏΠΎΠ»Π½Π΅Π½ΠΎ изолятором, Π½Π°ΠΏΡ€ΠΈΠΌΠ΅Ρ€ полиэтилСновым пластиком. ΠœΠ΅ΠΆΠ΄Ρƒ двумя ΠΏΡ€ΠΎΠ²ΠΎΠ΄Π½ΠΈΠΊΠ°ΠΌΠΈ Π²ΠΎΠ·Π½ΠΈΠΊΠ°Π΅Ρ‚ нСбольшой Ρ‚ΠΎΠΊ Ρ€Π°Π΄ΠΈΠ°Π»ΡŒΠ½ΠΎΠΉ ΡƒΡ‚Π΅Ρ‡ΠΊΠΈ.ΠžΠΏΡ€Π΅Π΄Π΅Π»ΠΈΡ‚Π΅ сопротивлСниС коаксиального кабСля Π΄Π»ΠΈΠ½ΠΎΠΉ L .

Π€ΠΈΠ³ΡƒΡ€Π° 9,18 ΠšΠΎΠ°ΠΊΡΠΈΠ°Π»ΡŒΠ½Ρ‹Π΅ ΠΊΠ°Π±Π΅Π»ΠΈ состоят ΠΈΠ· Π΄Π²ΡƒΡ… концСнтричСских ΠΆΠΈΠ», Ρ€Π°Π·Π΄Π΅Π»Π΅Π½Π½Ρ‹Ρ… изоляциСй. Они часто ΠΈΡΠΏΠΎΠ»ΡŒΠ·ΡƒΡŽΡ‚ΡΡ Π² кабСльном Ρ‚Π΅Π»Π΅Π²ΠΈΠ΄Π΅Π½ΠΈΠΈ ΠΈΠ»ΠΈ Π΄Ρ€ΡƒΠ³ΠΈΡ… Π°ΡƒΠ΄ΠΈΠΎΠ²ΠΈΠ·ΡƒΠ°Π»ΡŒΠ½Ρ‹Ρ… срСдствах связи.

БтратСгия
ΠœΡ‹ Π½Π΅ ΠΌΠΎΠΆΠ΅ΠΌ Π½Π°ΠΏΡ€ΡΠΌΡƒΡŽ ΠΈΡΠΏΠΎΠ»ΡŒΠ·ΠΎΠ²Π°Ρ‚ΡŒ ΡƒΡ€Π°Π²Π½Π΅Π½ΠΈΠ΅ R = ρLAR = ρLA. ВмСсто этого ΠΌΡ‹ смотрим Π½Π° концСнтричСскиС цилиндричСскиС ΠΎΠ±ΠΎΠ»ΠΎΡ‡ΠΊΠΈ Ρ‚ΠΎΠ»Ρ‰ΠΈΠ½ΠΎΠΉ dr ΠΈ ΠΈΠ½Ρ‚Π΅Π³Ρ€ΠΈΡ€ΡƒΠ΅ΠΌ.
РСшСниС
Π‘Π½Π°Ρ‡Π°Π»Π° ΠΌΡ‹ Π½Π°Ρ…ΠΎΠ΄ΠΈΠΌ Π²Ρ‹Ρ€Π°ΠΆΠ΅Π½ΠΈΠ΅ для dR , Π° Π·Π°Ρ‚Π΅ΠΌ ΠΈΠ½Ρ‚Π΅Π³Ρ€ΠΈΡ€ΡƒΠ΅ΠΌ ΠΎΡ‚ riri Π΄ΠΎ roro, dR = ρAdr = ρ2Ο€rLdr, R = ∫rirodR = ∫riroρ2Ο€rLdr = ρ2Ο€L∫riro1rdr = ρ2Ο€Llnrori.dR = ρAdr = ρ2Ο€rLdr, R = ∫rirodR = ∫riroρ2Ο€rLdr = ρ2Ο€L∫riro1rdr = ρ2Ο€Llnrori.
Π—Π½Π°Ρ‡Π΅Π½ΠΈΠ΅
Π‘ΠΎΠΏΡ€ΠΎΡ‚ΠΈΠ²Π»Π΅Π½ΠΈΠ΅ коаксиального кабСля зависит ΠΎΡ‚ Π΅Π³ΠΎ Π΄Π»ΠΈΠ½Ρ‹, Π²Π½ΡƒΡ‚Ρ€Π΅Π½Π½Π΅Π³ΠΎ ΠΈ внСшнСго радиусов, Π° Ρ‚Π°ΠΊΠΆΠ΅ ΡƒΠ΄Π΅Π»ΡŒΠ½ΠΎΠ³ΠΎ сопротивлСния ΠΌΠ°Ρ‚Π΅Ρ€ΠΈΠ°Π»Π°, Ρ€Π°Π·Π΄Π΅Π»ΡΡŽΡ‰Π΅Π³ΠΎ Π΄Π²Π° ΠΏΡ€ΠΎΠ²ΠΎΠ΄Π½ΠΈΠΊΠ°. ΠŸΠΎΡΠΊΠΎΠ»ΡŒΠΊΡƒ это сопротивлСниС Π½Π΅ бСсконСчно, ΠΌΠ΅ΠΆΠ΄Ρƒ двумя ΠΏΡ€ΠΎΠ²ΠΎΠ΄Π½ΠΈΠΊΠ°ΠΌΠΈ Π²ΠΎΠ·Π½ΠΈΠΊΠ°Π΅Ρ‚ нСбольшой Ρ‚ΠΎΠΊ ΡƒΡ‚Π΅Ρ‡ΠΊΠΈ. Π­Ρ‚ΠΎΡ‚ Ρ‚ΠΎΠΊ ΡƒΡ‚Π΅Ρ‡ΠΊΠΈ ΠΏΡ€ΠΈΠ²ΠΎΠ΄ΠΈΡ‚ ΠΊ ослаблСнию (ΠΈΠ»ΠΈ ослаблСнию) сигнала, ΠΏΠ΅Ρ€Π΅Π΄Π°Π²Π°Π΅ΠΌΠΎΠ³ΠΎ ΠΏΠΎ кабСлю.

Π”ΠΎΠ±Π°Π²ΠΈΡ‚ΡŒ ΠΊΠΎΠΌΠΌΠ΅Π½Ρ‚Π°Ρ€ΠΈΠΉ

Π’Π°Ρˆ адрСс email Π½Π΅ Π±ΡƒΠ΄Π΅Ρ‚ ΠΎΠΏΡƒΠ±Π»ΠΈΠΊΠΎΠ²Π°Π½. ΠžΠ±ΡΠ·Π°Ρ‚Π΅Π»ΡŒΠ½Ρ‹Π΅ поля ΠΏΠΎΠΌΠ΅Ρ‡Π΅Π½Ρ‹ *